Jump to content

Wikipedia:Reference desk/Science: Difference between revisions

From Wikipedia, the free encyclopedia
Content deleted Content added
Franamax (talk | contribs)
→‎Gravity: +answer
Line 785: Line 785:


I'm not sure whether this was only for the Pioneer/Voyager probes and I'm not sure whether it was just one particular battleship, but I know I read it once and I've never been able to find a reference since. Can anyone help me out? [[User:Franamax|Franamax]] ([[User talk:Franamax|talk]]) 20:58, 19 March 2008 (UTC)
I'm not sure whether this was only for the Pioneer/Voyager probes and I'm not sure whether it was just one particular battleship, but I know I read it once and I've never been able to find a reference since. Can anyone help me out? [[User:Franamax|Franamax]] ([[User talk:Franamax|talk]]) 20:58, 19 March 2008 (UTC)
:[[Operation Deadlight]][[Special:Contributions/83.100.183.180|83.100.183.180]] ([[User talk:83.100.183.180|talk]]) 21:15, 19 March 2008 (UTC)


== Wiki Myth Busters... ==
== Wiki Myth Busters... ==

Revision as of 21:15, 19 March 2008

Welcome to the science section
of the Wikipedia reference desk.
Select a section:
Want a faster answer?

Main page: Help searching Wikipedia

   

How can I get my question answered?

  • Select the section of the desk that best fits the general topic of your question (see the navigation column to the right).
  • Post your question to only one section, providing a short header that gives the topic of your question.
  • Type '~~~~' (that is, four tilde characters) at the end – this signs and dates your contribution so we know who wrote what and when.
  • Don't post personal contact information – it will be removed. Any answers will be provided here.
  • Please be as specific as possible, and include all relevant context – the usefulness of answers may depend on the context.
  • Note:
    • We don't answer (and may remove) questions that require medical diagnosis or legal advice.
    • We don't answer requests for opinions, predictions or debate.
    • We don't do your homework for you, though we'll help you past the stuck point.
    • We don't conduct original research or provide a free source of ideas, but we'll help you find information you need.



How do I answer a question?

Main page: Wikipedia:Reference desk/Guidelines

  • The best answers address the question directly, and back up facts with wikilinks and links to sources. Do not edit others' comments and do not give any medical or legal advice.
See also:


March 13

Recent rise in oil prices?

Hi. I'm pretty sure most of you have noticed recent sharp increases in oil prices. In fact, just a few years ago, there were occasional times where some gas stations would give prices of less than 50¢ a litre. At that time, gas prices per litre seldom reached above 90¢ per litre. Prices above $1/l were almost unheard of. Nowadays, prices above $1 are the norm. Prices below 90¢ are now almost unheard of. The news has said that, this spring, prices could reach $1.30. Oil is also constantly above $100 a barrel, if you're American. What's going on? This all started after hurricane Katrina, it seems. Are we reaching global peak oil already? Yes, I know the US and non-OPEC countries have already peaked. Are oil companies doing this on purpose? Or is this permanent and a sign of what's to come? What's next? $2.00 a litre within the next decade? $200 a barrel? Would peak oil cause a spike in prices? Yes I know it will probably run out completely in like, 40 years, so hopefully we switch to non-fossil fuels by then. As oil dwindles but not completely, what will we see? Maybe $10 a litre, $900 a barrel? Thanks. ~A H 1(T C U) 00:03, 13 March 2008 (UTC)[reply]

Your question is asking people to foretell the future. This is a reference desk, not a magic crystal you can rub and hope for a good answer. However, it isn't hard to figure out. Oil supplies are drying up. Oil demand is increasing faster than ever before. What do you think the price should do? -- kainaw 00:13, 13 March 2008 (UTC)[reply]
The present run-up (that is, above US$95 or so) is mostly related to the weak dollar - people want to hedge their investments by moving them to commodities. Daily fluctuations ($2-$3 or thereabouts) usually reflect particular news items, from weather to stocks reports to saber rattling. The basic price (say something like US$85 to $95) is a reflection of supply (near or at capacity right now) and demand (increasing despite the price). Best get used to it. Cheers Geologyguy (talk) 00:20, 13 March 2008 (UTC)[reply]
Any of your scenarios are possible, if not likely, but who can tell the future. For example I've heard serious predictions of $2 - $3 per litre within the next few years in Australia (current average price is about $1.40). And many sensible commentators also suggest that we've already passed worldwide peak oil and are now on the decline. --jjron (talk) 13:49, 13 March 2008 (UTC)[reply]

I find it entertaining seeing Americans complaining about fuel prices when we pay £1 per litre (which is about $8 per gallon) in the UK TheGreatZorko (talk) 14:11, 13 March 2008 (UTC)[reply]

Most of that is tax though. But I will say, complaining is fun, and it allows us british to feel superior knowing that we have much more right to complain than they do. -mattbuck (Talk) 14:30, 13 March 2008 (UTC)[reply]
Speaking as an American who is not complaining (mostly I'm amazed that our prices are not a lot higher), I really shudder to think what the US will be like when reality actually sets in. Cheers Geologyguy (talk) 14:34, 13 March 2008 (UTC)[reply]
I don't complain either because I don't drive an SUV the size of Kansas around. One time I pulled up to the pump after the previous person (who was an owner of a huge SUV) left and almost laughed to death when I saw he had paid almost $100 for gasoline. I haven't been to Europe in about 5 years, but from what I remember they aren't as stupid as we are, and they buy smaller cars unless they need to tow something. Do get me started on driving habits either - everyone I see with a superfluous SUV wastes gas by flooring the pedal the moment the light turns green. And yet they complain, and then they wonder why the world hates America so much. 206.252.74.48 (talk) 19:33, 13 March 2008 (UTC)[reply]

climate modelling simulation map-based continental land-influenced scenario?

Hi. I have a method you might be able to use for calculating the climate of a specific time period. Look at it and tell me what you think and what adjustments might need to be made if it plausibly works. Here are the steps:

map
  • first you need to draw on a piece of paper the shape of your Earth. Make sure you include the whole world in your map shape. You may want to get a huge piece of paper, and maybe several pieces. Now draw a grid for your map. Next, draw the continents. Take note of the elevation. If it is a map far into the past or future by millions of years, make sure you examine all the evidence. Make four maps, one for mid-winter, mid-spring, mid-summer, and mid-autumn. The seasons may not be at the months they are today because of precession, or the days might be longer or shorter, so don't write the month. If this is a period without seas, water, water vapour, sunshine, oxygen, vegetation, volcanoes, or plates, or if it is during a great bombardment, the map will be useless for that era.
  • Examine the plates at the time of the era of your map. If the fossil record shows proof of high mountainous habitats or undersea environments, include it in your map. Do not base the continents on what they look like today. Take note of the sea level. If the sea level was 300 metres below today's, then treat a piece of land that would be at sea level today as if it was 300 metres above sea level, and same if the sea level was or will be higher. If you're doing to future, include as many factors as possible in your simulation of placement of the continents. Draw in any plausible islands. If the mountain ranges that are here today did not exist back then, do not draw them. If it is likely that where there are mountains today were part of the sea floor back then, treat it as the sea floor. Make note of what type of climate it was. If there is evidense of a gulf or bay or inland sea or lake, draw it in. If there are landmasses near the poles there will likely be large glacial sheets.
  • make notice of the rivers and glaciers. If the mountains were high and cold enough, draw in glaciers at that reigon. If it is likely that there were pack ice or large sheet of ice like on Antarctica or Greenland today, draw it in. Make sure you draw these at the correct portion of the grid. If there was a seamount or undersea volcano or likely there was a coral reef or will be and the sea level is low enough to hold them above sea level, draw that in. Calculate sea level carefully, take in the factors. Look for likely positions of rivers and wetlands. If you know that there were deserts or forests in a particular location or will be, make not of that.
jet stream
  • draw 6 seasonal jet streams for both the northern and southern hemispheres, 2 polar, 2 temperate, and 2 tropical. Do this for each season. If the season in the hemisphere is summer, draw the jets closer to the poles, farther in winter, and moderately close tot he poles in spring and moderately far in autumn. Assuming that the Earth is rotation west-east during this era. Draw the polar jet streams with wind from east to west. Draw them from east to west, and make sure they meet on both ends of the map, like if it was at 80 deg at the rightmost egde of the map, make sure it is at 80 deg at the leftmost edge of the map. Cause it to dip south if there is a mountain range or large glacial sheet. If it was a warm period, draw them close to the poles. If it was a cold period, draw them away from the poles. If it was an ice age or there is an ice sheet on land kilometers thick covering the entire polar reigon and then some, draw them from west to east. Make sure that if there is a huge sheet of ice like this, make it so that the jet streams avoid the ice. On the leeward side of large tall mountain ranges that polar jet streams cross, which is the side facing away from the jet streams, cause the streams to dip towards the poles. The streams dip away from the poles if it encounters tundra or desert or ice, toward the poles if it encounters unfrozen water. It will dip towards the poles when it encounters forests in spring or summer, and away form the poles in autumn or winter. It will dip towards the poles if it encounters a coastal area after being over the sea.
  • the temperate jets are at temperate latitudes, from west to east Again draw them according to the season and warmth of the period. If it encounters mountains it will dip toward the equator. If in a warm season it encounters water it wil dip away from the poles and if in a cold season it will dip toward the poles. Rivers don't count as "water" unless it was a bay or gulf hundreds of km wide. Large areas below sea level but are not covered in water get a poleward dip and for other jets as well. Wetlands act like bodies of water if they are large enough. Deserts get poleward dips in spring and summer and equatorward dips in autumn and winter. The dips in spring and autumn are less than in summer and winter. Dip equatorward for any glaciers. For dry salt pans dip equatorward.
  • tropical jest go from east to west. Again draw them according to the global climate at the time. Oceans get a dip toward the equator, but if it just came from the sea and just met land at the coast, dip poleward. Make poleward dips if it just came from a forest. Dip poleward for deserts, and dip equatorward if it just came out of a desert. Do not cross the equator into the other hemisphere. Dip equatorward for mountains and any glaciers. Lakes and wetlands get equatorward dips if crossed.
high and low pressure
  • Label high pressure if an area generally has jet streams circling clockwise in the northern hemisphere and counter-clockwise in the southern hemisphere. For low pressure, the jets should be circling counter-clockwise in the northern hemisphere and clockwise in the southern hemisphere. Label warm and cold fronts around the warm sectors of low pressure systems. The fronts can connect with other low pressure systems but not with high pressure. They should not cross the equator. Create isobars around the high and low pressure. Make the pressure systems especially intense if they are large. Have generally higher pressure at lower areas and lower pressure on mountains and high elevation, including ice as elevation. Make sure you do these for each season, as with below.
prevailing winds
  • Draw wind direction around the pressure systems. Remember which direction the pressure systems turn in. Wind also goes away form high pressure and toward low pressure. Label upwelling and sinking of wind, it rises if it hits mountain ranges, sinks if it hits a valley or the ocean from high elevations, rises at low pressure and sinks at high pressure. Also label upper-level winds which go from upwellings towards sinkings. Wind should be allowed to cross the equator.
ocean currents
  • Draw warm and cold ocean currents. Draw cold ocean currents in cold areas and where the wind goes equatorward. Draw warm ocean currents near tropical reigons and areas where wind goes poleward. Only count surface winds. Do not draw them for lakes and inland seas disconnected from main sea unless they are enormous and deep. Make sure the currents avoid land even if it means going in the opposite direction from prevaling winds. Warm and cold currents should be allowed to converge and cold ocean currents should cross the equator unless something blocks it such as a massive release of freshwater into the oceans. Only the largest rivers should be allowed to nudge existing currrents away from its mouth. Cold currents should not enter shallow tropical gulfs, bays, and seas, and warm currents should not cross polar seas. Currents should be allowed to directly cross under a pressure system if nessecary.
precipitation
  • label wet and dry zones on the map. Areas where wind goes from the unfrozen sea or lake to land should be labelled wet. If warm currents approach land then makes a turn, colour in the areas it was approaching before it turned as wet. Leeward sides of mountains should be labelled dry. Areas directly under high pressure should be labelled dry. When cold ocean currents approach land, label these as dry. If wind from a large already wet place goes into another place, label it wet, and if wind from a large dry place goes to another place, label it wet. Label polar glaciers as dry. The windward side of a mountain, if the wind is from the ocean or a lake or a wet place, it should be labelled wet. Wind going from the land to the sea should be labelled dry. Areas where the jet stream goes from the ocean to land should be labelled wet until it turns around a large high pressure system. Areas ahead of a cold or warm front should be labelled wet, the cold front area larger in summer and warm fromt larger in winter. If the area is especially close to the equator, both spring/autumns should be considered summer. If an upwelling of air occurs near both the sea and the land, that area should be labelled wet. If upper-level winds go from the sea at an upwelling of air towards the land at moderate or high elevations, those areas should be labelled wet. Extremely large supercontinents should be labelled dry near the middle. Create lines denoting areas of equal precipitation for the particular month. Adjust the amount or precipitation of areas between wet and dry based on the global wetness at the era.
temperatures
  • If there is a large polar ice sheet on land, they are always cold, but warmer in summer than in winter. In polar areas, ocean should generally be warmer than land. Dips towards the poles should be warm, dips equatorward should be cooler. Mountains and higher elevation should generally be colder than surroundings. Temperate wet areas should be cooler than surroundings in spring and summer, and warmer in autumn and winter. Tropical wet areas should be cooler, tropical warm areas warmer. High pressure should be cooler if cold and warmer if warm. Low pressure should be warmer if cold, cooler if warm. Downwind from leeward side of mountains should be warmer. Poleward surface winds should be warmer, equatorward cooler. Consider the average global temperatures during that era and which areas are at which temperatures, create isotherms accordingly.
ecosystems and biomes
  • The ecosystems should be drawn on one map. Base it similarly to the modified Koppen system. If the total rainfall is very low for the four seasons, label it desert. Slightly wetter areas semidesert. Polar reigons, if there are glaciers, label it ice, if it’s cold and dry all year, label it tundra. If it’s sometimes warm in polar reigons and moderate precipitation, label it boreal forest. If a tropical area is warm and wet, label it rainforest. Label monsoonal reigons accorgingly and also based on their total precipitation. Label deserts hot and cold. Keep doing this according to both the climate in the areas and the likely biomes in the area.
final thoughts

Well, what do you think? I once tried this method myself (but much, much more simplicified) on the current climate and the near future, and got similar results to the real climate. Does this work, and what are some of the potential modifications tha tmay need to be made? No this is not homework. What would one discover if they tried to base it on global warming of the future? Would northern Europe cool down? Amazonia turn to grassland? Acatama and Namib deserts turn to mediterranean climate? Central North America become desert? Sahel become desert? Lower Tibet become desert? Monsoonal India become drier? North America west coast become drier? California become hotter? Increased El Ninos? SAL across the Atlantic? Little change in Atlantic hurricane activity? Is this similar to some of the climate modeling simulations? Can this be used to predict past and future climates based on continental drift? Or do we have to implement what we already know for the Holocene for it to be accurate? Thanks. ~A H 1(T C U) 00:25, 13 March 2008 (UTC)[reply]

non-medical placebos?

Hi. I know when you apply a treatment and convince the patient it will work and it works, that's a placebo, what what about non-medical, ie. the power of suggestion? Like for example, if you go into a dark house, and someone convinces you it's haunted, then suddenly you start experiencing signs of "ghosts" when it's really your mind playing tricks on you based on the suggestion, what is this effect called? Or, if you convince yourself you're sick, but you're really not, and you begin to experince what you think are "symptoms" of some sickness? Thanks. ~A H 1(T C U) 00:29, 13 March 2008 (UTC)[reply]

Perhaps you are thinking of psychosomatic or conversion disorder --Omnipotence407 (talk) 00:49, 13 March 2008 (UTC)[reply]
See also: nocebo effect, suggestion, hypnotism.
You might also try confirmation bias or hypochondria. - Eldereft ~(s)talk~ 00:53, 16 March 2008 (UTC)[reply]

"Random outbursts give people cancer"?!?

Hi. This is not a request for medical advice. Is this possible? Or was this a sarcastic joke or itself a random outburst? Or is it brain cancer? Is the joke about "if your hand is bigger than your head, you have cancer" solely in order to slam the victim's hand into their face? Or was this question a random outburst? Thanks. ~A H 1(T C U) 00:44, 13 March 2008 (UTC)[reply]

Complete and utter hoax. Wisdom89 (T / C) 00:50, 13 March 2008 (UTC)[reply]
Maybe they meant random outbursts of gamma rays. And yes, the thing about your hand is so that you can hit someone on the nose. That's it. --98.217.18.109 (talk) 00:58, 13 March 2008 (UTC)[reply]

Goatse - how does he do that with his butt?

How did the Goatse man manage to get his rectum so insanely stretchy? Is there a likely medical explanation for it (I saw a guy who could stretch his skin about two feet from his body on tv once, maybe its the same thing), or was it just a "gift" he was born with? How do you go about discovering that you can do that anyway? --62.136.16.236 (talk) 00:58, 13 March 2008 (UTC)[reply]

Well, you can bet he probably started small and worked his way up. If you view the whole series (not recommended), you'll see that he inserts extremely large objects into his rear before doing the classic pose. That's got to help, if that's your goal. --98.217.18.109 (talk) 01:01, 13 March 2008 (UTC)[reply]
It's actually a martial art perfected by the Chinese monk Goa Tse. See here. bibliomaniac 1 5 01:02, 13 March 2008 (UTC)[reply]
Eww!! That's uncyclopedia!!!!!!
As can be seen in slideshow form at http://www.goatsemarathon.com (obviously not worksafe). I've sometimes wondered if his internet antics (legendary they may be) have affected his continence. --Kurt Shaped Box (talk) 01:49, 13 March 2008 (UTC)[reply]
Note to self: When 98.217.18.109 says "not recommended", you don't need to check for yourself. ៛ Bielle (talk) 02:09, 13 March 2008 (UTC)[reply]
It's been years since I've seen it yet I can remember some of the scenes like it was yesterday. It's one of those visual memories you sort of resent having semi-permanently burned into your brain somewhere. Also, re: speculation about his health, I doubt that's a terribly good thing to do for your body and probably has a number of long-term side effects. That's not medical advice though I doubt any doctor would recommend doing that. --98.217.18.109 (talk) 04:38, 13 March 2008 (UTC)[reply]
Men who engage long-term in normal (well, normal for some) receptive anal intercourse (let alone grotesque activities such as fisting) often end up with continence problems because of the weakening of the muscles. What this guy has done to himself ... I'm lost for words. -- JackofOz (talk) 05:20, 13 March 2008 (UTC)[reply]
Jack, being familiar with all matters Down Under, speaks wisely. If this practice hasn't already wrecked him, I'm confident it soon will. I'd wager that traditional medicine would frown on this practice, but holistic medicine would open up to it, perhaps. StuRat (talk) 06:53, 13 March 2008 (UTC)[reply]
I dont know if the Down Under comment was accidental Stu, but its hilarious nevertheless! — Preceding unsigned comment added by 79.76.144.62 (talk) 01:44, 14 March 2008 (UTC)[reply]
Jack, would you be speaking from experience or is it just fistwishful thinking? — Preceding unsigned comment added by 79.76.144.62 (talk) 01:38, 14 March 2008 (UTC)[reply]
Might it cause a Fist ula? Edison (talk) 18:22, 13 March 2008 (UTC)[reply]
Sounds odd - but I think that the internet would probably be a poorer place without the Goatse Man. He's inspired a lot of people to do creative, humourous and culture jamming-related things over the years - just take a look at all those 'tribute to Goatse' and 'Goatse sightings in the real world' pages. Heh, Time Magazine may have even placed a deliberate nod to him on one of their covers... --Kurt Shaped Box (talk) 17:41, 13 March 2008 (UTC)[reply]
Most people believe the goatse man is Kirk Johnson. If you are sick enough to compare his vast collection of photos and videos to the official goatse photo, it is difficult to imagine that Kirk is not the goatse man. So, now that you have a good idea who he is, you search for his email address and ask him about his practices. Who knows, he may have written books or produced do-it-yourself videos. -- kainaw 18:54, 13 March 2008 (UTC)[reply]
I'm pretty sure the goatse man is dead. In fact just Google that phrase (goatse man is dead) and you'll find plenty of links, none of which I'm going to attempt to visit from my workplace computer. --LarryMac | Talk 19:11, 13 March 2008 (UTC)[reply]
If you heard a story about him dying at the age of 70-something, that was from a hoax newspaper article knocked up by the boys at Stileproject years ago. If he died recently, I haven't heard anything about it. --Kurt Shaped Box (talk) 22:50, 13 March 2008 (UTC)[reply]
Years of stretching; you can do it with almost any part of your body. Mac Davis (talk) 23:34, 17 March 2008 (UTC)[reply]

psa levels

can alchohol consumption effect psa levels? — Preceding unsigned comment added by Jon julie (talkcontribs) 02:03, 13 March 2008 (UTC)[reply]

If people didn't drink so much, there wouldn't be so many public service announcements about drunk driving and so forth. I hope this answers your question.--The Fat Man Who Never Came Back (talk) 02:26, 13 March 2008 (UTC)[reply]
To translate, "psa" could refer to anything on this page. Could you please (a) work out which one you're talking about, (b) check the article about it to see if it mentions the effect of alcohol, and (c) if it doesn't, feel free to come back here and clarify what you meant. And the grammar freak in me would like to point out that the word you probably meant to use was affect, although context may prove effect correct. Confusing Manifestation(Say hi!) 02:46, 13 March 2008 (UTC)[reply]
I enjoyed my joke, but he's probably referring to Prostate specific antigen levels.--The Fat Man Who Never Came Back (talk) 02:48, 13 March 2008 (UTC)[reply]
I enjoyed it too, and I agree that that's the likely candidate, but you know what they say about people who assume. Confusing Manifestation(Say hi!) 02:50, 13 March 2008 (UTC)[reply]
Don’t assume that we know what they say. — Knowledge Seeker 09:34, 13 March 2008 (UTC)[reply]
Ass u me--Shniken1 (talk) 10:16, 13 March 2008 (UTC)[reply]
"Well, it is something about a donkey, it is a stupid American phrase!" — Tamfang (talk) 21:03, 19 March 2008 (UTC)[reply]

About Albert Einstien (E=mc^2)

On deriving E=mc², did Einstien also come to know how to travel with the speed of light by converting mass into energy (energy in the form of electromagnetic radiation) and back to mass? i.e, did he know how to travel so fast that one could almost 'dissappear' from one place and 'appear' at another place almost at the same time?59.95.77.132 (talk) 05:34, 13 March 2008 (UTC)[reply]

The short answer is no, certainly not for "everyday" objects - there is usually an irreversible increase in entropy when mass is converted to energy. But see our article on teleportation for recent developments. Gandalf61 (talk) 07:17, 13 March 2008 (UTC)[reply]
E=mc² wasn't a process for extracting energy, it was an upper bound on the energy that might be extracted by any process. Prior to this the only way you could quantify energy was by the amount converted from one form to another. You could extract work from a substance by converting it into something else, but (aside from thermodynamic limitations) it was never clear whether you'd gotten all the work you could get out of it or whether there was some yet-undiscovered way of getting more. Einstein's argument suggested that an object of (inertial) mass m had a total of mc² energy, meaning that that was the most you could ever extract — after that there would be nothing left of the original object. It said nothing about whether there was actually any physical process that could extract that much energy. However at the time people had already noticed that radioactive substances seemed to release an enormous amount of energy — though very slowly — and I think people imagined or hoped that by studying and harnessing this process they could get the full mc² energy out of ordinary matter. It's now known that this isn't correct — radioactivity is just another kind of chemical process, involving nuclear bonds instead of atomic bonds, and just rearranges the constituents of matter without creating or destroying them — but nuclear energy seems to be permanently associated with E=mc² in the public consciousness. It's also now known that you can't turn the basic constituents of matter into useful energy without violating various conservation laws. Actually all of the laws that you need to violate seem to be only approximate, but the situations in which they're violated are pretty extreme, and it's very hard for me to believe that you could ever construct a useful (and safe!) power source this way. You can get plenty of energy from nuclear chemistry alone, and for that you just need the laws of nuclear chemistry, not E=mc². So although E=mc² is of major theoretical importance, it's not relevant to any practical technology. -- BenRG (talk) 13:44, 13 March 2008 (UTC)[reply]
Nuclear fission and radioactivity are not chemical processes. Nuclear chemistry covers different aspects of a nuclear power plant to the actual energy production. AlmostReadytoFly (talk) 14:48, 13 March 2008 (UTC)[reply]
Furthermore, massive particles such as beta particles, neutrinos and antineutrinos are created in radioactive decay. AlmostReadytoFly (talk) 15:02, 13 March 2008 (UTC)[reply]
And, uh, E=mc^2 is relevant to practical technology. There is a real mass differential between U-235 and its fission products, and the energy resulting from the fission reaction (mostly in the form of the kinetic energy of the repelling fission pieces) corresponds exactly to that "missing mass" times the speed of light squared. It comes into play in fusion reactions as well. To say nothing of atomic weapons! It's not an accident or an error that the public associate E=mc^2 with nuclear energy. Fission doesn't violate any conservation laws — total energy of the system is conserved even if some of it is in the form of nuclear bonds and then it becomes kinetic energy, etc. It does violate conservation of mass, but all of SR does that, and that's OK. --98.217.18.109 (talk) 22:14, 13 March 2008 (UTC)[reply]

Tell me if it' snot a good idea....

...to make an air filter based on the design of the human nose. That is, it would consist of several long tubes filled with fibers (nose hairs) extending from the walls, coated with a thick fluid dripping from the walls onto the fibers. Air would be blown through the tubes. The nostrils would need to be occasionally cleaned, perhaps with a silicone corkscrew that could be run through them once a day. Some type of antibacterial solution would need to be used in the drip, to prevent nasties from growing in it. StuRat (talk) 06:39, 13 March 2008 (UTC)[reply]

Snot for me my friend. Who gets to work the corkscrew? Julia Rossi (talk) 07:26, 13 March 2008 (UTC)[reply]
I suppose the high-end models could have this function automated. On the other hand, if the liquid drips through quickly enough, that may clean the nostrils out sufficiently without needing a corkscrew. StuRat (talk) 14:09, 13 March 2008 (UTC)[reply]
Underemployed telephone sanitisers.
Atlant (talk) 13:10, 13 March 2008 (UTC)[reply]
The nose is not designed primarily as a filter, and it doesn't make a particularly good one. I don't think we're gonna see this on Dragons' Den in the near future.--Shantavira|feed me 08:33, 13 March 2008 (UTC)[reply]
Citation please? Seems to me that noses are excellent filters. I don't know about you but whenever I'm inside where the air quality is lower, I'm breathing through my nose to filter our some of the dust etc. And yes the nose is of course an excellent olefactory device, but if I were to posit its main functions, smell would be 1A and getting filtered oxygen to the lungs would be 1B. Vranak (talk) 14:40, 13 March 2008 (UTC)[reply]

This is actually done. Many mechanical air filters are coated with a water-soluable "stickum". They then server their purpose for a while, get dirty, are washed up, recoated, and placed back in service for another cycle. Light oil is also used but requires a detergent or solvent during the washing-up phase. My sports car has just such a filter.

Atlant (talk) 13:12, 13 March 2008 (UTC)[reply]

This wouldn't be much different than blowing air past one of those hanging fly-strips and hoping all the nasties get stuck to it. They don't. You have two things working against you. You need an adhesive that will trap everything - which will be difficult to find. The adhesive cannot dry out as air blows past - which will be difficult to find. -- kainaw 13:16, 13 March 2008 (UTC)[reply]
I don't think any filter removes 100% of the particulates from the air with each pass. As for drying out, my idea was to have a reservoir of liquid which slowly drips over the fibers. To make the fluid last longer, it should be oil-based instead of water-based. Glue traps for mice use such an adhesive, and they stay tacky for quite a long time. To do double duty, the dripping oil could have a nice scent to it, like cinnamon or mint, to provide "aromatherapy" at the same time it cleans the air. StuRat (talk) 14:00, 13 March 2008 (UTC)[reply]
This makes me wonder about bubbling air up through a reservoir of sticky oil. By the time the bubble hits the top, much of the gunk will get stuck. No need to clean anything. Just dump out the reservoir and refill as necessary. -- kainaw 15:03, 13 March 2008 (UTC)[reply]
That might work, or it might just make a thick foam that fills the entire house. :-) StuRat (talk) 15:29, 13 March 2008 (UTC)[reply]
Isn't Kainaw's idea simular to how a bong works? However, those use water and get gunky relatively quickly. Some of our inventions based off nature work quite well (velcro) while others not so well (Ornithopter). But as for this invention...who nose? 206.252.74.48 (talk) 15:40, 13 March 2008 (UTC)[reply]
The difference is that bubbles containing water quickly pop as the water evaporates, but this is not so for bubbles made of oil. StuRat (talk) 16:46, 13 March 2008 (UTC)[reply]
Congratulations you’ve made it onto my list. --S.dedalus (talk) 22:44, 13 March 2008 (UTC)[reply]
Cool. StuRat (talk) 17:05, 16 March 2008 (UTC)[reply]

volume of space

may we say that the space has afinite volume but infinite surface,like Gabriel`s horn???thank youHusseinshimaljasimdini (talk) 11:18, 13 March 2008 (UTC)[reply]

You can say anything you like. It's a free Internet. However, there are existing arguments about what constitutes the "end" of space. Is it infinite? Does it have a boundary? Does it wrap back upon itself? -- kainaw 12:10, 13 March 2008 (UTC)[reply]
If space has infinite volume I would surmise that it has infinite dimension (radius?) and so would come to the conclusion that the outer surface would have infinte area. Nevertheless the concept of an outer surface of infinite radius is a difficult one to consider.87.102.94.198 (talk) 12:51, 13 March 2008 (UTC)[reply]
The general belief is that physical space doesn't have a boundary at all. -- BenRG (talk) 12:56, 13 March 2008 (UTC)[reply]
Husseinshimaljasimdini - I am not clear what you mean by "infinite surface". You seem to be assuming that the Universe has a boundary. As BenRG says, all common models for the topology of the Universe assume that the Universe does not have a boundary (don't be misled by the use of the word "horn" in models such as the Picard horn). On the other hand, you say that "space has a finite volume". Some observations of possible periodicities in the cosmic microwave background hint at a finite universe but this is not certain, and there are still plausible models in which the Universe is infinite. Possibly when you say "space" you may be thinking of the observable universe (which is finite) rather than the whole Universe ? Our article on the shape of the Universe has more details. Gandalf61 (talk) 13:08, 13 March 2008 (UTC).I got it .thank you all.Husseinshimaljasimdini (talk) 14:58, 14 March 2008 (UTC)[reply]
In case it was not clear, the original question referenced Gabriel's Horn, a known mathematical paradox; the unique shape of this figure results in infinite area surrounding a finite volume. I don't think this shape is widely regarded as a good analogy for the universe as we know it. Nimur (talk) 08:13, 16 March 2008 (UTC)[reply]

buisness

what is the importance of buisness to the society?nipet — Preceding unsigned comment added by Nipet (talkcontribs) 12:15, 13 March 2008 (UTC)[reply]

Take a look at The Wealth of Nations. It will point you in the right direction. – Clockwork Soul 13:28, 13 March 2008 (UTC)[reply]

Gases "dissolved" in liquids; fish breathing

Can someone explain to me, in layman's terms, how exactly oxygen is dissolved in sea water and how fish breathe with their gills? Is the oxygen in sea water in the form of tiny bubbles, which the fish's gills suck into their bloodstream in the same way that land animals absorb oxygen through their lungs? Or is the oxygen somehow chemically bound to water or other molecules in sea water as it is to hemoglobin in the blood, in which case the fish's gills would presumably perform a chemical reaction? Or is there some other mechanism involved? Thanks! Marco polo (talk) 13:20, 13 March 2008 (UTC)[reply]

Refer to solubility. To put it in very simple terms, consider water as being composed of many small particles (molecules) of water. These particles nonetheless have a lot of space between them, and molecules of oxygen are able to fill some of that empty space. This is the oxygen dissolving in the water. The way the oxygen then gets transferred into a fish's bloodstream is a similar process to how lungs work. Water is passed over the gills, and the oxygen diffuses into the blood where it is at a lower concentration than in the water. It doesn't require a chemical reaction on the gills to extract the oxygen. --jjron (talk) 14:11, 13 March 2008 (UTC)[reply]
Nitpick: diffusion across a cell membrane, in this case that of the gill's cells, is called osmosis. --Bowlhover 15:04, 13 March 2008 (UTC)
Wouldn't osmosis mean the water is diffusing, not the oxygen? DMacks (talk) 06:30, 14 March 2008 (UTC)[reply]

Well water is made partially of oxygen (It's H2O) but I don't think that is how they do it. I'm sure there is oxygen that is disolved in the water (not in the form of bubbles otherwise fish could breath out of water) but liquid, if that makes sense. I'm not sure how this happens since oxygen has to be REALLY cold before it becomes a liquid on its own. I wouldn't mind hearing an answer to this also. EDIT: Oh hey an answer TheGreatZorko (talk) 14:15, 13 March 2008 (UTC)[reply]

jjron's answer covers that aspect fairly well. As for how the air is initially dissolved in water, there are several mechanisms. Wave action at the surface forces tiny bubbles into the water, which then dissolve into the water. Plants in the water also give off oxygen as a result of photosynthesis, which is then dissolved in the water. Even still water with no plant life will absorb some oxygen at the surface. Ocean currents are critical to the distribution of this dissolved oxygen, which otherwise would be at a much higher concentration near the surface than deep underwater. Note that other gases in air, like nitrogen and carbon dioxide, also dissolve in water. There is an occasional problem where an area of water is low on oxygen, creating a dead zone. Aquariums which lack wave action or sufficient plants often add oxygen to the water by bubbling air through it. StuRat (talk) 14:29, 13 March 2008 (UTC)[reply]
oxygen has to be REALLY cold before it becomes a liquid on its own
Doesn't matter. Salt has to be REALLY hot before it melts into a liquid on its own, but you have no problem with salt dissolving in water, right? — Keenan Pepper 14:49, 13 March 2008 (UTC)[reply]
Check out Lake Nyos for an interesting gas-dissolved-in-water story. --Sean 14:51, 13 March 2008 (UTC)[reply]
I've looked at the Lake Nyos article several times before, and I can't help but wonder at the thought processes and value systems I see. I'd support the "lower the level" plan a lot more cheerfully if all of the planners' children lived downstream. Lowering the pressure, without reducing the gas load, seems, well, unwise. I mean, they already have a vent pipe which works on it's own, powered solely by the rising gases coming out of solution. The vent pretty much proves that we've figured out what we're dealing with. So, given that kind of supersaturation, what happens to the immense pressure in the lower levels if the water level is lowered by the proposed 20m? You are dropping the pressure by about 5 atmospheres, which will, incidentally, allow some of the dissolved gas down there to come out on it's own, and the rising bubbles will probably cause the impending-disaster-to-be-prevented to happen right then and there. I know, take it to the discussion page. -SandyJax (talk) 18:45, 13 March 2008 (UTC)[reply]
The "lower the level" plan has nothing to do with dissolved gas. It's about preventing a landslide and resulting flood that could kill tens of thousands. --Carnildo (talk) 21:56, 13 March 2008 (UTC)[reply]
...and it will create another massive gas outpour, which will....wait for it...kill tens of thousands. It might be a good idea AFTER the pumps have lowered the supersaturation level of the deep water. -SandyJax (talk) 20:42, 14 March 2008 (UTC)[reply]
I saw a movie where a pressurised water suit had a large amount of oxygen disolved in it. Can you breath water if enough oxygen is disolved in it?--155.144.251.120 (talk) 03:32, 14 March 2008 (UTC)[reply]
See liquid breathing. DMacks (talk) 06:32, 14 March 2008 (UTC)[reply]
Water can't absorb nearly enough oxygen to support a mammal. If you're thinking about the film "The Abyss", then they were not breathing water, but some sort of super-oxygenated fluid like perfluorocarbon. 72.10.110.107 (talk) 15:43, 18 March 2008 (UTC)[reply]

Good day, Oxygen is soluble in water. To absorb that oxygen, fishes take a gulp of water and force it out of their gills. Their Gills consists of tiny, fibrous filaments (increase Surface area to volume). These filaments are held up by gill bars which have more tiny individual lamallae which are even smaller (yet again more Surface area to volume ratio). The lamallae is the part of the gills that does the absorbing because it is very thin (few cells thin) so oxygen is easily diffused into the body via a steep concentration gradient difference between O2 outside the lamallae and inside the lamalle, and just right underneath the lamallae are vast networks of capillaries to absorb the oxygen and transport it to the rest of the body where oxygen is needed through the aid of the heart's pumping action. Consequently it is the opposite when Co2 needed to be diffused out through a concentration gradient through the lammale.Hey mrs tee (talk) 09:46, 19 March 2008 (UTC)[reply]

International Space Station

So far the international space station has always housed 3 crew members. When the ISS is finished, will it house more crew members. (I don't believe our article says; I read through it but it is possible I missed it). ike9898 (talk) 16:26, 13 March 2008 (UTC)[reply]

According to this SPACE.com article, the final crew capacity will be six or seven people. --Bowlhover 21:27, 13 March 2008 (UTC)
Sometime during Expedition 19 next year, the crew will increase from 3 to 6. anonymous6494 05:23, 14 March 2008 (UTC)[reply]

IV nurse (medicine)

A question from Germany: What is an "IV nurse"? I have heared it in Grey's Anatomy ("IV nurse to ICU") What is "LR"? - a medical acronym. ("two liter LR running wide open") (Kind of a liquid for i.v. application to substitute a volume deficit.) --84.137.46.213 (talk) 20:28, 13 March 2008 (UTC)[reply]

Isn't a IV nurse a nurse who knows how to do intravenous therapy? --Bowlhover 21:18, 13 March 2008 (UTC)
and also assigned to carry out IV therapy, as opposed to other duties. (If five nurses are on a single hospital floor, they may all know how to do IV therapy, but only one might be assigned to be the IV nurse). - Nunh-huh 02:02, 14 March 2008 (UTC)[reply]
LR is lactated Ringer's solution. TenOfAllTrades(talk) 21:28, 13 March 2008 (UTC)[reply]
Thanks for the fast answers. --84.137.46.213 (talk) 21:45, 13 March 2008 (UTC)[reply]

What are the night-blooming vegetables?

We are interested in a comprehensive list of night-blooming vegetables. — Preceding unsigned comment added by Webstergl (talkcontribs) 21:09, 13 March 2008 (UTC)[reply]

If you include herbs, you might like to look into these: Silver Thyme, 'Alba' or white Eggplant, white pumpkins, Basil, Mint, Oregano according to[1]. Silver or reflective foliage and white fruits seem to be indicators. Bats and moths the agents. Are you thinking of starting an article here? hint hintJulia Rossi (talk) 23:29, 13 March 2008 (UTC)[reply]
If you include fruit, don't forget pineapple. — Keenan Pepper 05:35, 14 March 2008 (UTC)[reply]

if cocaine is just a molecule, why dont peeple synthesize it?

so if cocaine is just a molecule (picture to the right in that article) why don't peeple synthesize it, like they do with meth or lsd, which are also molecules right? why go thru all the trouble of stuffing it up some immigrants butt excuse my french to get it past border control thank you — Preceding unsigned comment added by 79.122.103.78 (talk) 22:10, 13 March 2008 (UTC)[reply]

Because synthesizing it takes time, effort, and some technical know how. Wisdom89 (T / C) 22:18, 13 March 2008 (UTC)[reply]
Even LSD and methamphetamine have starting materials, and those happen to be quite readily available. But as mentioned in Cocain#Synthesis, creating a drug from scratch would be difficult, problematic, and also very expensive. The only answer I can give you is to guess that unlike the other two drugs, there are simply no possible starting materials that occur naturally (or in pharmacies) in sufficient quantity to make synthesis cheaper than smuggling. Someguy1221 (talk) 22:18, 13 March 2008 (UTC)[reply]
(ec) Well, sneaking it past border patrol is actually pretty cost-effective for cocaine, which has a very high street value for its relatively small volumes, unlike LSD and methamphetamines (cocaine costs roughly 10X that for the same volume of meth, if I recall). So you only have to smuggle a small amount to make up for the price of smuggling and the dangers associated. In any case, there is a section on synthesis in the article: Cocaine#Synthesis. It's not cited but it sounds plausible to me: "Synthetic cocaine would be highly desirable to the illegal drug industry, as it would eliminate the high visibility and low reliability of offshore sources and international smuggling, replacing them with clandestine domestic laboratories, as are common for illicit methamphetamine. However, natural cocaine remains the lowest cost and highest quality supply of cocaine. Actual full synthesis of cocaine is rarely done. Formation of inactive enantiomers and synthetic by-products limits the yield and purity."--98.217.18.109 (talk) 22:20, 13 March 2008 (UTC)[reply]
(Of course, I am aware that the cost of the drug is directly related to the availability. But I'm just pointing out that when the price is that high the smuggling isn't all that bad a method, even though, of course, the price is that high in part because smuggling is the method.) --98.217.18.109 (talk) 22:22, 13 March 2008 (UTC)[reply]
The answer to these types of questions is almost always "Because it's not cost effective". If people could make big money synthesizing cocaine, they would presumably already be doing it. Friday (talk) 23:38, 13 March 2008 (UTC)[reply]
An important part of the answer is that setting up a cocaine-synthesis plant would require a lot of up-front investment for an uncertain return, while paying somebody to smuggle it doesn't require much investment at all. --69.134.115.242 (talk) 00:05, 14 March 2008 (UTC)[reply]
That's true, but there is still investment on smuggling. The cocaine doesn't just make itself (or bribe officials, or hide from the Americans, or whatever). --98.217.18.109 (talk) 03:38, 14 March 2008 (UTC)[reply]
Basically same reason as there is no industrial-scale synthesis of caffeine - it can be synthesised, but it is cheaper to extract it from organic sources. Gandalf61 (talk) 09:49, 14 March 2008 (UTC)[reply]
The organic pool is always an atractive source for compounds. The other point is that synthesis requires skilled people who will get less dangerous jobs elsewhere.--Stone (talk) 09:59, 14 March 2008 (UTC)[reply]
Talk:Caffeine#Artificial_caffeine suggests that caffeine as an additive is sometimes or maybe even often/usually synthetic. But Caffeine#Production says (and cites) that it's so easy to extract (and so much is extracted anyway) that it's usually natural. DMacks (talk) 11:10, 14 March 2008 (UTC)[reply]
Cocaine suggests there are problems with current methods. It's likely that these could be solves but as people have already mentioned, existing natural sources are good enough that there is I presume no incentive. Note that the those in charge of the grower side of supply as well as anything else that would be affected by synthesis would obviously not be happy with any threat to their profit and may actively supress any attempts to develop methods to sythesise cocaine if they expect they will be significiantly cut out from the new market Nil Einne (talk) 15:55, 15 March 2008 (UTC)[reply]


March 14

physic

richard fenyman —Preceding unsigned comment added by 71.10.27.69 (talk) 00:14, 14 March 2008 (UTC)[reply]

We just happen to have an article on Richard Feynman, if that's who you're looking for.--VectorPotentialTalk 00:17, 14 March 2008 (UTC)[reply]

Quantum Physics

Why does each atom have different allowed energy levels for its electrons? I understand that each electron is actually a "particle-wave" and its orbits are only the ones allowed so the electron-wave won't cancel itself. So, why are there different orbits allowed for each compound and atom? Every electron is the same so how are there different orbits in each atom that don't make them cancel? For example, in one atom the orbits may be consecutively 2eV then 5eV, but in another, there may be an allowed orbit of 3.5eV where in the other atom this would not have been allowed. Why is this? Many thanks, Zrs 12 (talk) 00:18, 14 March 2008 (UTC)[reply]

The article on atomic orbitals will give you the full answer on this. However, my simplified answer to this is that the charge in the nucleus is different for each type of atom. Consequently, the electric potential energy is different in different atoms. The full quantum mechanical solution for atoms more complex than helium is not trivial and cannot be expressed as a neat exact mathematical formula. Numerical solution techniques must be used. For hydrogen, an exact expression is possible as derived in the Bohr model but this relies on simplifications that cannot be applied to more complex atoms. SpinningSpark 00:47, 14 March 2008 (UTC)[reply]
Mathematical side note: Roughly speaking, the calculation of energy levels involves solving the time-independent Schrödinger equation
where is the energy to be solved, is the potential energy of the electron. The depends on different kind of atom since is the integral of electrostatic force experienced by the electron and is affected by the proton number in the nucleus of different kind of atoms. Therefore, the difference in makes different set of energy levels solved. - Justin545 (talk) 02:05, 14 March 2008 (UTC)[reply]
And atoms with multiple electrons, have the electrons influencing each other. The Pauli exclusion principle applies as electrons are fermions. It forces electrons to adopt different positions in the atom. Graeme Bartlett (talk) 00:53, 14 March 2008 (UTC)[reply]
For a very simplified explanation, see the energy levels article SpinningSpark 01:15, 14 March 2008 (UTC)[reply]

What species of seal or walrus is this?

http://www.youtube.com/watch?v=DDg7kWgs5e0 --Sonjaaa (talk) 02:17, 14 March 2008 (UTC)[reply]

It is definitely an eared seal, meaning either some species of sea lion or - less likely - some species of fur seal. That leaves a choice of about 17 species. Many of the archetypal performing pinnipeds in circuses etc. are California sea lions. --mglg(talk) 16:03, 14 March 2008 (UTC)[reply]

RL circuit

I am working on a physics laboratory project dealing with an RL circuit (the resistor and inductor are in series with a battery). I am trying to find out the work done by the battery over a period of time from when the circuit is connected, and the energy dissipated from the resistor in that same time period. I realize that to find the work done I need to integrate the expression for the power (P=IV) over time, but I'm not sure how I ought to go about doing that. —Preceding unsigned comment added by 134.129.115.87 (talk) 02:19, 14 March 2008 (UTC)[reply]

The work done by the battery is equal to the energy dissipated in the resistor. Is that any help? —Preceding unsigned comment added by SpingMander (talkcontribs) 02:36, 14 March 2008 (UTC)[reply]
I don't think that's right. The battery also did work creating the magnetic field around the inductor; the magnetic field represents potential energy which can be returned to the electrical circuit at a later point in time. Integrated over infinity, your answer is correct, but it's not correct over some discrete, finite portion of time.
Atlant (talk) 12:23, 14 March 2008 (UTC)[reply]

Total work down is given by the expression: (1/T)(integral 0 to T of (P dt)) where P is the power dissipated in the resistor over time t and T is the time period per osscilation of the RL circuit.

Ozone in the interstellar medium

Molecular oxygen has been found in the interstellar medium. Why has ozone not? There is UV radiation in the ISM from nearby stars, surely this would produce ozone? Is it just that it has not yet been discovered. NB: I know that ozone is short lived but if there is O2 and UV radiation continously present, you will get ozone....right? --Shniken1 (talk) 03:38, 14 March 2008 (UTC)[reply]

You also need sufficient density of O2 to generate the ozone, which is unlikely. — Lomn 04:53, 14 March 2008 (UTC)[reply]
Especially, you need a sufficient fraction of the total gas being O2, because else the atomic oxygen produced by photodissociation will also react with other atoms or molecules, and only a small fraction with O2 to form O3. Icek (talk) 22:25, 14 March 2008 (UTC)[reply]
Additionally, the formation of ozone needs a 3-particle collision to conserve the momentum (maybe it could - but with a much lower probability - be conserved by the emission of a photon), whereas the destruction of ozone doesn't; this is another reason for the virtual non-existence of ozone in the low-density interstellar medium. Icek (talk) 06:59, 15 March 2008 (UTC)[reply]

what decides the direction of rotation of diesel engines?

section created AlmostReadytoFly (talk) 10:20, 14 March 2008 (UTC)[reply]

what decides the direction of rotaion of doesel engines--Shoby 2001 (talk) 07:09, 14 March 2008 (UTC)shoby 14 mar 08[reply]

4 stroke or 2 stroke? (just kidding) Seriously it depends on the timing of the injections of fuel into the cyclinders - which in turn must match the arrangement of the crankshaft. Also the starter motor must start in this direction.
If you are still bemused I recommend you read a piece on "how diesel engines work" search for this.. Once you know how they work it the answer should be obvious.
Need more explanation?87.102.83.204 (talk) 11:51, 14 March 2008 (UTC)[reply]
Aren't some supercharged/turbocharged diesel designs valve-less? Assuming you inject the fuel at Top Dead Center, they'll run in either direction. And aren't some marine diesel engines designed to run in both the "forward" and "reverse" directions?
Atlant (talk) 12:29, 14 March 2008 (UTC)[reply]
Your experteese not mine - all I can say is that assuming more than one cylinder I'd still have to control the timing of the fuel injections of the cylinders differently for each rotation direction. eg I'm assuming a 4 cylnder crank has the offsets at 0,90,180,270 degrees?
I'm willing to learn more though, or be wrong.87.102.83.204 (talk) 12:45, 14 March 2008 (UTC)[reply]
(correct myself) If the fuel injections are controlled by tappets on the crank then yes - I think you're right that they would work both ways..
As for 'valveless' there need to be something preventing the diesel gas escaping back out (ie a one way valve) - I don't have full understanding of the exact terminology87.102.83.204 (talk) 12:47, 14 March 2008 (UTC)[reply]
No, that's the interesting thing with a diesel: The only thing that goes in is air and the only thing that goes out is exhaust. (Unlike an Otto engine that forces in an expensive fuel-air mixture.) Because of this, you can arrange the engine so that it simply has valveless exhaust and inlet ports in the cylinder very near the bottom of the stroke. The super/turbocharger (the "one-way device" you mentioned) then rams a whole lotta air in, forcing out the previous cycle's exhaust gas. The piston then rises, closing off the ports and compression begins. It's not the most volumetrically-efficient way to arrange the engine (because 1) lots of extra volume of air is pushed through the engine and 2) because some of the stroke is "wasted" until the piston passes the ports) but it sure is simple and reliable.
Also, injection pumps used to be driven by a gear-train from the crank so that's effectively the same as if they were driven by tappets on the crank.
Atlant (talk) 16:26, 14 March 2008 (UTC)[reply]
Yes. I was thinking in terms of a four stroke 'otto like' diesel - two stroke will work in reverse as you say. So it looks like that the starter motor will determine the rotation in the case of a 2 stroke.
For the reasons you mention I always find it suprising that 2 strokes are more efficeint than 4 strokes. Maybe the 1/2 utilisation of each stroke for power in a 4 stroke tips the balance.87.102.83.204 (talk) 16:42, 14 March 2008 (UTC)[reply]
Ahh, here's one article I was thinking about: Opposed piston engine. I should have thought of it sooner seeing as how I took one of the pictures that graces that article ;-),
Atlant (talk) 23:30, 14 March 2008 (UTC)[reply]
And here's something similar (with animation!) Napier Deltic 87.102.21.171 (talk) 09:57, 15 March 2008 (UTC)[reply]

Knee problems

Does anyone know why traditional Zulu's don't have knee problems? I have searched everywhere and come up with nothing related. Leepylal (talk) 07:53, 14 March 2008 (UTC)[reply]

Where did you hear they don't? - Nunh-huh 08:32, 14 March 2008 (UTC)[reply]

I'm doing a course in exercise science. They make the statement and ask what factors contribute to this phenomenon. —Preceding unsigned comment added by Leepylal (talkcontribs) 10:40, 14 March 2008 (UTC)[reply]

Haven't a clue but people more in line for knee problems (but don't seem to have any to report) are the Maasai for jumping. Julia Rossi (talk) 11:21, 14 March 2008 (UTC)[reply]

try this http://pharyngula.org/index/weblog/print/1572/ specifically comment number 27 ? Does that help87.102.83.204 (talk) 11:47, 14 March 2008 (UTC)[reply]

The idea 'use it or lose it' comes to mind. Perhaps these traditional Zulus use their legs a great deal traversing the veldt, so they are kept in good repair. Vranak (talk) 21:02, 14 March 2008 (UTC)[reply]

Omnipresent God

Does evidence of the superposition of particles that would collapse if observed, such as elelctron difraction, not rule out an omnipresent god, as if he was all seeing and all knowing, he would see mhich gate the electron went through, and therefore destroy the wave diffraction pattern. —Preceding unsigned comment added by 81.140.151.57 (talk) 09:34, 14 March 2008 (UTC)[reply]

It's not quite true that electrons intrinsically know that they are being observed. It's simply the case that all of our detection methods alter the electron's behavior, though I'm sure someone can better describe how this works.
That's right, but please note that it is not the clumsiness of measurement that causes the alteration. It is a necessary consequence of the measurement no matter what method is used. Detection requires energy from the detected particle to be transferred to the detection device. Transfer of energy necessarily means the particle must change state or even be destroyed. If you are trying to detect a train, the change in energy is inconsequential. If you are trying to detect fundamental particles you must find them to be in one of the allowed quantum states, which are usually radically different from one another. SpinningSpark 11:53, 14 March 2008 (UTC)[reply]
When you get into God and logic, there really is no falsifiability as any number of explanations could be used to allow for exceptions. Perhaps God is incorporeal. Perhaps an all knowing God doesn't need to observe the electrons since He already knows its paths. — Ƶ§œš¹ [aɪm ˈfɻɛ̃ⁿdˡi] 09:51, 14 March 2008 (UTC)[reply]
The point the original poster was probably trying to make is that according to mainstream interpretations of various phenomena that are studied in quantum mechanics there is no fact about certain things except that they are in a superposition. And if Bell's Theorem is correct, there cannot be underlying variables which we are either ignorant of or unable to detect due to imprecise equipment.
So the OP wants to know if the universe operates like this, with certain characteristics of matter being in a superposition without clear facts about the certain characteristics other than the fact they are in a superposition, then does that invalidate the notion that a deity knows everything. This conception of the universe is at odds with what most Western religions have thought for quite some time; and it also really perplexes some philosophers of physics. Traditional theology probably defined omniscience as knowing everything that is knowable, so you can't merely object that God can't know false things. This definition seems to evade the worry the OP mentioned since, if there is no fact of the matter in respect to certain characteristics of certain pieces of matter then there is not anything that God would fail to know.--droptone (talk) 16:38, 14 March 2008 (UTC)[reply]
Theology isn't really my subject, but I don't see why omniscience would imply knowing properties of a system that, according to quantum mechanics, the system doesn't even have. What quantum mechanics says is that a system can at a given time only have some properties out of the collection of all properties a similar system might have; having certain properties precludes having certain others. (Very roughly speaking it can have half of all properties at a time.) If you know which properties it has, you can in principle measure the values of those properties without affecting the system. If you try to measure a property that it doesn't have, you're effectively forcing it into a state where it has that property at the expense of some other property it used to have, and it's in that sense that you're altering it. There's nothing that would preclude a deity knowing which properties a system has at any given time, and the values of those properties. -- BenRG (talk) 11:58, 14 March 2008 (UTC)[reply]
Indeed. Asking which slit the electron went through is like asking what colour is Thursday. If a question has no answer then even God cannot answer it. He could change the universe so that the question did have an answer, but He chooses not to. Gandalf61 (talk) 12:40, 14 March 2008 (UTC)[reply]
I see that as a particularly poor example of trying to diminish any respectability to the question, the fact is in two slits experiment if you measure which slit the electron went through, even if you do this by measuring it's "wake" as such, it did go through a slit. But if you dont measure, then the superposition occurs, and patterns of interference with itself are clear. Thursday does not have a colour, even if you try and measure it. If god genuinely saw everything (wether this is a to rigorous definition of omnipresence is another question) then he would know which slit the electron went through and no interference pattern would be made. —Preceding unsigned comment added by 81.140.151.57 (talk) 09:44, 15 March 2008 (UTC)[reply]
Well, anyway, this is the old "Does God play dice with the universe?" question, really. Which has as much to do with your ideas about God as it does what flavor of quantum mechanics you ascribe to. Omniscience is pretty poorly defined, in the sense that its actual physical properties are usually not defined, not to the degree necessary to make sense of quantum mechanical implications . Perhaps He would know the possible future outcomes for all quantum configurations? Is God an "observer" in the physical sense or in some other, supernatural sense? Who knows? It isn't exactly a rigorous set of terms. --98.217.18.109 (talk) 12:45, 14 March 2008 (UTC)[reply]
It really depends on which god you're talking about. There's this Nordic one I just heard about that might be able to do exactly what you're looking for... Imagine Reason (talk) 00:45, 16 March 2008 (UTC)[reply]
Or perhaps God, as it were, could be seen in in the electron...Vance.mcpherson (talk) 20:29, 17 March 2008 (UTC)[reply]
Again, which one? Imagine Reason (talk) 03:50, 18 March 2008 (UTC)[reply]
Hu? How can you have more than one? The God who created the universe obviously. How's it possible to have more than one? Ariel. (talk) 21:12, 19 March 2008 (UTC)[reply]
I myself thought there were two, actually. 66.65.143.85 (talk) 21:02, 20 March 2008 (UTC)[reply]
Because God doesn't watch the electron do anything - God causes the electron to do what it does. Every single motion of every single particle in the universe is individually 'animated' by God. Ariel. (talk) 16:18, 18 March 2008 (UTC)[reply]
No wonder so many children are starving. Imagine Reason (talk) 17:02, 19 March 2008 (UTC)[reply]
Did you post in the wrong section? Ariel. (talk) 21:12, 19 March 2008 (UTC)[reply]

Indian pharmaceutical companies

Which pharmaceutical company is first in India Org ratings & how much turn over per year —Preceding unsigned comment added by 121.247.109.61 (talk) 10:49, 14 March 2008 (UTC)[reply]

Hope you don't mind, I turned down the volume. No caps lock please. Julia Rossi (talk) 11:23, 14 March 2008 (UTC)[reply]
maybe this http://iis-db.stanford.edu/pubs/11893/Smith.pdf will have the answer (p24+?)87.102.83.204 (talk) 11:43, 14 March 2008 (UTC)[reply]
or Category:Pharmaceutical_companies_of_India should help if you do the neccessary research.87.102.83.204 (talk) 11:44, 14 March 2008 (UTC)[reply]
PS what is "India Org"?87.102.83.204 (talk) 12:16, 14 March 2008 (UTC)[reply]

Chemical Process Pipe Labeling

I need the following question answered: When having a mixture of two or more chemicals in a vessel and the mixture is being tranfered to another vessel thru process piping what type of labeling is needed or required? Im not looking for the ansi color code standard, but the exact nomenclature to label the process pipe.

The mixture does consist of the following:

BioDiesel, Methanol, Glycerin, and Sulfuric Acid

So how do I make the correct label in correlation to the most hazardous,the most concentration, specific mixture?

Or do I need to label with all materials within the mixture?

Thank you George Leighlitner CTIBioFuels (email address removed to prevent abuse) Richard Avery (talk) 14:57, 14 March 2008 (UTC) —Preceding unsigned comment added by 66.212.144.50 (talk) 14:30, 14 March 2008 (UTC)[reply]

Right I honestly think you are taking the piss here. I apologise for my frank reply.
http://www.ctibiofuels.com/ which I assume is your company appears to be a legitimate business. We are volunteers and are not paid. I would not expect any professional company to be relying upon the advice of unpaid volunteers to give what is essentially health and safety advice - a matter which most likely is covered and governed by applicable laws in your region.
Typically such safety and safe working practice issues are taken very seriously and failure to comply with them can result in criminal prosecution. As such I would imagine that it is your responsibility to hire or employ someone who is professionally qualified and certified to give you advice on this matter.
Please do not rely on volunteers for this information.87.102.83.204 (talk) 15:33, 14 March 2008 (UTC)[reply]
You should include the words "Run away!" somewhere in your labelling scheme. --Heron (talk) 20:03, 14 March 2008 (UTC)[reply]
Maybe one pipe has Sodium Hydroxide in it instead of Sulfuric acid ? Polypipe Wrangler (talk) 05:56, 15 March 2008 (UTC)[reply]

Vance.mcpherson (talk) 20:40, 17 March 2008 (UTC)[reply]

Echo above, and add that this will differ heavily depending on your province / state / country. And the intended use of the product, and who's going to be using it. Legal implications about this sort of thing are dire. Especially with that mix of joy-in-a-jar. The good news is that these regulations are easy to find and it's usually inexpensive or free to get yourself trained. See Dangerous Goods. I've never seen any reg's about pipes per se, but there may be such regs in some jurisdictions. I wouldn't be using an oxidizable metal to make that pipe, though.

Evolutionary biology and sexual selection in humans

Hi. I remember reading that studies are now showing that a significant percentage of women trick men into raising children who are not theirs, but I can't find any such article now. Is there a reliable source I can go to for the figure? Thanks. —Preceding unsigned comment added by Imagine Reason (talkcontribs) 15:58, 14 March 2008 (UTC)[reply]

Unfortunately, our article on cuckold has no worthwhile references. -- kainaw 16:04, 14 March 2008 (UTC)[reply]
Mmh women who trick their husbands.. I doubt it would ever be possible to get a reliable figure from people who carry out such a deceit.? Also did you specifically want women who 'trick their men' or to include all step-father type upbringings eg accident, drunk, uncontrollable fling, don't know exactly who the father is,,, etc.?87.102.83.204 (talk) 16:55, 14 March 2008 (UTC)[reply]
What the OP is asking about is called "extrapair paternity", and we have a little bit on it at Incidence_of_Monogamy#Incidence_of_genetic_monogamy. --Allen (talk) 17:23, 14 March 2008 (UTC)[reply]
To clarify, "extrapair paternity" doesn't apply to stepfathers, but only to cases where the mother's partner does not know he is not the genetic father. That's because rates of extrapair paternity are used to study how males behave differently (e.g. put less effort into raising their partner's offspring) when they cannot be sure that they are the genetic father. --Allen (talk) 17:33, 14 March 2008 (UTC)[reply]
The studies looking at this looked at DNA (or maybe just blood types, can't remember) of parents and children.. and found very high rates of cuckolding (maybe 1 in 4).. however the methodology was very flawed.. as the "fathers" were a self selecting group who wanted to know if their child was legit.. so unsurprisingly their suspicions were often founded much more often than you'd expect from the general population. The stats were publicized by an agency which specialized in this sort of test. I don't know if any "legitimate" studies have been done in this area. I also don't have any references. —Pengo 04:36, 15 March 2008 (UTC)[reply]
I think that was the study that I read about. A shame, then, that it is not so reliable. Imagine Reason (talk) 15:18, 15 March 2008 (UTC)[reply]
It boggles my mind the bad methodology that one often sees in such sorts of surveys. You'd think someone involved would have seen the very obvious problem with that approach. --98.217.18.109 (talk) 20:31, 15 March 2008 (UTC)[reply]
Maybe the less conscientious scientists do it more often. It certainly gets them into the news. Imagine Reason (talk) 00:47, 16 March 2008 (UTC)[reply]
The conventional rate of "non-paternal events" in a (human) genetic lineage is 10%, though more recent studies suggest a lower rate of about 5%. Even at the lower rate, it is more likely than not that any given chain of 14 generations has been interrupted by an (otherwise unsuspected) non-paternal event. - Nunh-huh 04:57, 15 March 2008 (UTC)[reply]
I trust your numbers, but "non-paternal event" seems to include adoption, stepfather relationships, and other situations where the mother's partner knows he is not the genetic father [2]. So they wouldn't play the same evolutionary role as unknowing extrapair paternity. --Allen (talk) 13:36, 15 March 2008 (UTC)[reply]
No, these are unexpected non-paternal events within a "known" genealogy. These are non-paternal events unsuspected by genealogists, and in most instances it's reasonable to presume also unsuspected by the (falsely) identified father. - Nunh-huh 01:56, 17 March 2008 (UTC)[reply]

3000 Farad "ultracapacitor". Is it a joke?

I've recently seen this [3] on a webpage. Could it be true? A 3000 Farad capacitor? Shouldn't it be larger than the Earth? And needing a Dyson sphere to power it up?

Oops, sorry, I see we have an article about it.. so they must exist for real. So how can it be, that it didn't solved our energy problems? As I know, the storage of power is one of our biggest problems, that's one of the causes why we are trying to develop superconductors. To store solar energy for the night. To store energy in cars and other vehicles. To create man-portable particle beam weapons and railguns. To stop wasting fuel by heating cities at nighttime and cooling them at daytime. --V. Szabolcs (talk) 19:07, 14 March 2008 (UTC)[reply]

It exists, but farads do not equal joules. You can only store a small amount of energy in one of those things, and you can only inject or extract a small amount of power. They have an amazing capacitance per unit volume, but not a very high energy per unit volume or per unit mass. If you look at our energy density article, you will see that a typical ultracapacitor has an ED of about 0.02 MJ/kg, compared to a lead-acid battery with about 0.1 MJ/kg. --Heron (talk) 19:48, 14 March 2008 (UTC)[reply]
It only operates at 2.7 volts. 3000 farads at that voltage can only store 2250 mAh of charge. About the same as a regular AA cell. SpinningSpark 20:33, 14 March 2008 (UTC)[reply]
Yes, but the nice part is that the charge/discharge cycle is substantially more efficient with an ultracapacitor and you can run it hundreds of thousands of times, unlike with rechargeable batteries that do something like 1,000 cycles. For this reason, ultracapacitors are starting to figure into the power trains of hybrid vehicles.
Atlant (talk) 23:40, 14 March 2008 (UTC)[reply]
Point taken, Atlant, but ultracaps haven't yet been economically scaled up for big applications like load-levelling of wind and solar power. (Ultracaps are used in the internal works of some wind turbines, but not for load-levelling, as far as I know.) My virtual money is on flow batteries for really big installations. --Heron (talk) 14:03, 15 March 2008 (UTC)[reply]
Actually, I saw a talk recently about a demonstration project using ultracaps for fluctuation management on a ~70 MW wind farm. Dragons flight (talk) 16:37, 15 March 2008 (UTC)[reply]

Windy metro entrances

Why is there a strong airflow at the ground-level entrances to subway stations? 91.156.148.161 (talk) 19:51, 14 March 2008 (UTC)[reply]

Two reasons come to mind. First, to ensure a constant supply of fresh, clean air at platform level, some stations may use forced ventilation to draw in clean surface air. Second, moving subway trains act as large piston heads within their tunnels, forcing air out of the tunnel ahead of them and drawing in air behind. Both of these factors can mean that a subway station will be at a higher or lower air pressure, resulting in a constant (sometimes strong) airflow. TenOfAllTrades(talk) 19:56, 14 March 2008 (UTC)[reply]
I don't actually know anything about this (other than many years commuting) but from experience the wind at the entrances seems to me to be particularly steady and therefore unlikely to be anything to do with the trains pushing the air along. This is not the case on the platforms where clearly the train is having a marked effect. Could it be that underground heat is forcing the air to rise? The air is certainly very much warmer underground. One can imagine that the air escapes through one entrance and is replaced by colder air coming in through another lower entrance (possibly even at a different station). The constriction at the entrances would magnify the air velocity so even a quite small effect of this sort could produce a strong wind at the entrance. SpinningSpark 20:46, 14 March 2008 (UTC)[reply]
Forced ventilation. There are utility buildings that house huge fans. People ruin "fresh air" pretty fast, between body heat, O2-CO2 respiration, water vapor addition, smoking, body odor, engine exhaust, you name it, we go through air FAST. And, large groups of people go through it faster. So, any restricted area (read as "small cave filled with people") will have massive air turnover systems designed in. And, rather than have three openings (people access, air in, and air out), it is cheaper to have the people opening also serve as one of the air ducts. Next, since the whole purpose is to maintain a constant supply of fresh air, and the people opening is right next to where the bus is idling on the street, belching blue smoke, it is standard to use the people opening as the air exhaust. Go to any subway. You'll note that the wind is coming FROM the subway TO the opening on the sidewalk. Now, how did I get logged out, and where did the SignBot go? -SandyJax (talk) 21:58, 14 March 2008 (UTC)[reply]
Thanks for the explanations! Yeah, the air indeed flows from inside the metro station to the opening. 91.156.148.161 (talk) 07:10, 15 March 2008 (UTC)[reply]
There is another possibility, although from the description of the air always blowing out, I doubt if this is what's happening here. In a large cave system, with only tiny openings to the outside, wind will blow in or out to equalize the pressure whenever the surface barometric pressure changes. At times this can cause strong winds at cave entrances or narrow points inside the cave. I suspect that this effect is slight in the subway, in comparison with the forced air system, due to the large number of entrances to any subway. StuRat (talk) 18:26, 16 March 2008 (UTC)[reply]

Gustatory table of elements

Do molecules, and more specifically elements, have flavours? Could you arange complex molecules in a taste map similar to the colour thing, where they fade into one another? I mean to say, do chemically similar elements taste similar? I think I'm phrasing it badly but that's the best I can do really. 81.96.160.6 (talk) 20:57, 14 March 2008 (UTC)[reply]

Yes, but I believe it would be insufferably difficult to do. However, salts taste salty, acids taste sour, bases taste bitter, most sugars taste sweet, etc..etc.. Wisdom89 (T / C) 21:03, 14 March 2008 (UTC)[reply]
There were informative answers here on a previous question regarding the taste of salts which concluded that all salts do not taste salty. There is some grouping by chemical species but the overall picture is clearly much more complex.
If you were looking for a taste equivalent of colour space it really does not exist. For one thing, human colour vision has only three receptors making the colour space possible to plot graphically. Taste has many more (given that smell forms a large part of taste). Secondly, the receptors are still being discovered and documented and a definitive "map" in terms of the fundamental sensations is not yet possible. And it would need to be multi-dimensional even if all the information were now available. SpinningSpark 22:01, 14 March 2008 (UTC)[reply]
Having said all that, many "flavours" that come to mind when you say the word are actually derived from the same family of molecules -- the terpene family. Among the tasty assortment of delicacies associated with terpenes or terpenoids are citrus (limonene), black pepper (sabinene), ginger (zingiberene and other sesquiterpenoids), mint (pulegone, although menthol is not an example), cloves (eugenol), strawberry (linalool and nerolidol), among many, many others. Our olfactory senses seem to have evolved to be particularly discriminating of of terpenes and terpenoids. Recall that the tongue can only perceive sweet, salty, sour or bitter, and the nose does the rest. Take a peek at [4] for a little fun.Vance.mcpherson (talk) 21:21, 17 March 2008 (UTC)[reply]

Speed of fire ant destruction

How fast can a bunch of fire ants devour something? I guess it depends on how many there are, so let's say a door is opened and a big room full come out into another room; could a person be threatened? This is a a parody story of a soap opera I'm helping a person write, and I'm using an old idea I had years ago as a kid about an Evil Overlord threatening to take over the world using fire ants to blackmail it. Obviously, I don't need to know exactly, but just ballpark figure of what they could do.

As a side note, I recall as a kid, seeing on...maybe That's Incredible on TV, how a colony of fire ants can devour an animal, all but skeleton, in minutes. But, that's pretty vague, too, it was long ago I saw it. Thanks.63.3.19.1 (talk) 21:18, 14 March 2008 (UTC)[reply]

This isn't a science reference but I guess the author researched it – there's an informative scene of fire ant destruction in the Congo in Barbara Kingsolver's novel The Poisonwood Bible though our article is brief and doesn't mention it. Fire ant mentions them in relation to small animals. Julia Rossi (talk) 23:10, 14 March 2008 (UTC)[reply]

Can you look up this physical property for me? I've exhaused my available references

I need to know the surface tension of a saturated solution of magnesium nitrate. Can you help? If so, I also like to know where you found it... Thanks! ike9898 (talk) 21:57, 14 March 2008 (UTC)[reply]

It's possible that someone has measured this - if you can find chemical Abstracts you could look it up there. It's equally possible I think that this has never been measured before.87.102.21.171 (talk) 09:52, 15 March 2008 (UTC)[reply]

Thunderstorms

What's the capacitance of a thunderstorm? --Carnildo (talk) 21:58, 14 March 2008 (UTC)[reply]

Not at all sure that anyone has ever analysed thunderstorms in terms of capacitance. However, to have a go at this question, the thunderstorm article states that an average thunderstorm contains 3.6 x 1013 joules of energy. The lightning article tells me that 109 volts are required to break down air in a 1000 ft lightning strike. Using the equation (see capacitance article);

yields,

Which is a piddling 72 μF. Even a big storm with 100 times the energy only comes out to a surprisingly small 7200 μF. It is turning out this way because the square of a very large voltage is an even more enormous number. Whether anyone has any "real" figures for thunderstorm capacitance I don't know. SpinningSpark 23:51, 14 March 2008 (UTC)[reply]

Um, let me correct that, I was misreading the energy figure from the thunderstorm article which is the energy of the storm, not the electrical energy. The lightning article gives 5x108 joule for the energy of an average lighting strike. So translating that to capacitance yields;
Even smaller. SpinningSpark 00:08, 15 March 2008 (UTC)[reply]
Another possible approach is to calculate the capacitance directly from the geometry of the storm. Probably, the closest model is a spherical storm above a ground plane. But since some pretty wild assumptions are being made, we might as well use the slightly simpler parallel plate model. For a 500m radius storm, 300m above the ground,
putting in results in
I make the result for the spherical/ground plane model;
just over half the parallel plate answer. SpinningSpark 11:26, 15 March 2008 (UTC)[reply]
Being creatively lazy, I just Googled for the answer. The top hit for "thunder cloud capacitance" is an interesting PDF [5] on the subject. It arrives at a figure of 60 nF, so you guys are in the right rounders field.
Digression: That PDF also has an calculation of the power in all the world's thunderstorms, which turns out to be just enough for the electrical needs of one large city. I'll try to remember that fact for the next time someone asks how to harness the power of a thunderstorm. --Heron (talk) 11:55, 15 March 2008 (UTC)[reply]
Actually, the 60 nF calculated in that reference is not the same capacitance. They have calculated 'self-capacitance'. That is, the notional capacitance between the storm and an inifinitely large sphere centred on it. On the other hand, my calcualtion above is the capacitance between the storm and ground. The next paragraph in that paper points out that the important capacitance as far as the discharge of lightning is concerned is the capacitance between the storm and the ionised channel once this has been struck. This capacitance they calculate at 9 nF for the same size storm. SpinningSpark 12:55, 15 March 2008 (UTC)[reply]


March 15

Quantum Physics II

  • How does a wave have a charge (like an electron)?
  • How does light (a wave-particle) and an electron (also a wave-particle) have differing masses if they are both particle-waves?
  • How does a wave have mass at all?

Zrs 12 (talk) 03:39, 15 March 2008 (UTC)[reply]

Mass and energy are equivalent. Anything with energy has mass. An electromagnetic wave does have charge, but positive and negative mixed together so that the result is neutral. Light is a traveling wave, confined to moving at light speed, whereas an electron is a wave packet, something like a standing wave, and therefore can travel at other speeds.
But the 'why' questions dig at the limits of what we know of the universe. — kwami (talk) 03:54, 15 March 2008 (UTC)[reply]
Yes I know , so why does the electron have a mass and photons not have a mass? They both contain energy, no? Zrs 12 (talk) 04:00, 15 March 2008 (UTC)[reply]
A photon has zero rest mass, not zero mass. 'Zero rest mass' is a fiction, another way of saying that it can't be at rest. The mass of a photon is its energy. — kwami (talk) 04:07, 15 March 2008 (UTC)[reply]
The concept of relativistic mass have gone out of use, kwami, rest mass is just an obsolete term for mass. Narayanese (talk) 09:41, 15 March 2008 (UTC)[reply]
Electrons aren't waves. They are electrons. They have wave-like properties, they exhibit wave-like behavior (in some circumstances). But they are not "waves" per se. (When you lose track of the metaphor and the thing-itself then you're heading into the territory of confusion.) --98.217.18.109 (talk) 14:59, 15 March 2008 (UTC)[reply]
According to this resource electrons are particle waves. Is UC Berkeley wrong or am I missing something? Furthermore photons are also wave-particles (photons). What am I missing? Zrs 12 (talk) 20:01, 15 March 2008 (UTC)[reply]
That's a theoretical claim which has not been demonstrated. — kwami (talk) 19:56, 15 March 2008 (UTC)[reply]
Well, don't confuse one professor's lecture at UC Berkeley with UC Berkeley the institution! I haven't the time to figure out where in that hour-long lecture the claim comes up but if I were his teaching fellow (and I have been a teaching fellow in similar classes), I would probably be explaining to my students the basics of the Copenhagen interpretation in response. Electrons are electrons. When we understand them with macroscopic metaphors our understandings will be in part imperfect. It is important to remember this. We can model their behavior like a wave in some situations, and we can model their behavior like a particle in others. (Wave-particle duality). But they are neither waves nor particles. They are electrons. They act like both waves and particles. See Complementarity (physics). There are, of course, other interpretations of QM, but frankly I find Copenhagen to be the most intuitively straightforward once you grasp it, as a non-scientist. It makes your questions unnecessary and removes the confusion of worrying too much about what it would mean for a wave to have a charge—the wave doesn't have the charge, the wave is just another function of what an electron is, just as the charge is part of that. An electric charge is just a fundamental property of the subatomic entity.
If you wanted to point me to the specific time in the lecture I'd be happy to take a look at it. My bet is that the professor is just trying to convey to the non-scientist students that electrons are like both waves and particles. Again, I think a more thorough explanation—if I were doing it, anyway—would be to separate out the epistemology of the electron (what we can know about it, how we can know it) from the ontology of the electron (what the electron really is), which is more in the style of Copenhagen, and in my non-scientist opinion, easier for non-scientists to grasp (personally I think scientists have very little clue what's easier for a non-scientist to grasp or how non-scientists learn, but that's just my own observation, being someone who has long been on the borders between science and the humanities). Regarding it as a "particle wave" is fine shorthand for remembering it has both properties but again, it's a metaphor, it's in a human language, it will have limitations. --98.217.18.109 (talk) 20:19, 15 March 2008 (UTC)[reply]
From 10:42 to about 16:30 he talks about electron waves orbiting the nucleus. This part is what I was talking about. Zrs 12 (talk) 22:21, 15 March 2008 (UTC)[reply]

can we produce 50,000 units of electricity from solar energy

Hi, I want to know wheather i can produce near to 50,000 units of electricity from concentrated type solar panels. I live in a tropical country with ample sun light . Please give me the precice amount of stream needed and the time it needs to be converted into stream.I would also like to know about the suitable dynamo needed with price margine.please help me this is my last resort and this is not an homework question.(e-mail removed to avoid abuse)--Manoj man1990 (talk) 06:21, 15 March 2008 (UTC)[reply]

Find out the amount of units of electricity you can get from a single solar panel in your country and work out how many you would need to get 50,000 units. Better still contact a solar panel supplier and ask them.
What do you mean by stream? is this about a water generation method?
87.102.21.171 (talk) 10:54, 15 March 2008 (UTC)[reply]
What unit? Saying "50000 units of electricity" is like saying there is 1000 units between New York and London. Doesn't say anything at all about the actual distance - the unit could be nanometers or parsecs for all we know! -- Aeluwas (talk) 11:27, 15 March 2008 (UTC)[reply]
I think the OP is referring to a solar thermal concentrator using steam as the heat transfer fluid. There are some links in solar thermal energy that may be of use. If by 'units' you mean watts, then the answer is yes, systems delivering tens of MW have been built. If by 'units' you mean kWh per year, then the answer is still yes, since 50,000 kWh per year is not that much. --Heron (talk) 11:38, 15 March 2008 (UTC)[reply]
Stream is a common (but usually incorrect, depending on language) synonym for "electricity" in many languages. For example, my own native language Dutch has "stroom". User:Krator (t c) 15:12, 15 March 2008 (UTC)[reply]
While anon could be more clear, when it comes to household/consumer electricity, Units of electricity nearly always means kWh. This is in most English speaking countries that I'm aware of including the UK, NZ, Malaysia and apparently the US as well. Household bills and electricity providers will often talk about units rather then kWh although I presume it is defined somewhere in the small print for legal purposes Nil Einne (talk) 15:38, 15 March 2008 (UTC)[reply]
But OP can't mean kWh, because "produce near to 50,000 kWh of electricity from concentrated type solar panels" would be meaningless. 50,000 kWh in how much time? --Heron (talk) 18:23, 15 March 2008 (UTC)[reply]
Unless they say otherwise, I'd assume they mean to produce 50,000 kW per hour. Of course, since solar power only works when the Sun's out, the next question would be how much beyond 50,000 kW must be generated at peak production to allow for it to be stored so 50,000 kW can be drawn continuously. This assumes that a continuous power supply is needed. For some tasks, like pumping water into a water tower, I suppose it's fine to only operate the machinery when the Sun is shining. StuRat (talk) 18:13, 16 March 2008 (UTC)[reply]
Check out [6] -- this may be of some help, although all associated info is in Canadian dollars and presupposes you want to buy the solar panels commercially available in Canada. -- Vance.mcpherson (talk) 21:27, 17 March 2008 (UTC)[reply]

Observes & speed of light

I may have asked this question before, but if I did, I probably didn't understand the answer as it has been bothering me still.

An observer is in a train travelling at 10 m/s on a double track in space. Another train passes on the second track, travelling at 10 m/s as well. Through the side window, the observer sees that train as if it were moving at 20 m/s, causing a certain amount of motion blur etc.

Then, the train starts travelling at 160,000,000 m/s (or: slightly more than half the speed of light), and another train passes at 160,000,000 m/s. What does the observer see? User:Krator (t c) 15:23, 15 March 2008 (UTC)[reply]

I'm a bit confused as to how many trains there are and what you are describing. If one train is going 10 m/s then it can't be passed by another train going the same speed, unless you mean 10 m/s relative to the other train. Also, are talking about trains accelerating (from 20 m/s to 160,000,00 m/s) or moving at constant velocities? Because that changes how it is handled with relativity and etc.; acceleration is much more complicated than constant speeds. In other words, I think you need to clarify a bit of what you are asking. In any case, if what you are asking is will at anytime one observer see a train passing him at the speed of light, then no, he won't—he'll see the speed as being slower, one way or another (the amount of space passed in a given amount of time will never mean their velocity is c or greater—either the time will be measured differently or the amount of space will be measured differently, depending on how you look at it). --98.217.18.109 (talk) 16:16, 15 March 2008 (UTC)[reply]
If an outside observer would see the two trains as moving in opposite directions, both at 1.6×108m/s, then an observer in one of the trains would observe the other as moving at about 2.49×108m/s. See velocity-addition formula. Algebraist 17:58, 15 March 2008 (UTC)[reply]
The OP means that the other train is travelling in the opposite direction. His problem is that if both trains are travelling at more than half C then surely the velocity of one train observed form the other will be more than C? The answer is that no it won't. Velocities do not add linearly, so in this equation where u and v are the velocities of the trains and w is the velocity of one observed from the other;
is only approximately true when u and v are small compared to C. The correct formula for adding velocities under special relativity is;
From which it can be seen that w can never be more than C. The best it can do is equaal C if either u or v are C also. SpinningSpark 17:58, 15 March 2008 (UTC)[reply]
Two trains passing each other at 160 Mm/s will look the same, from the perspective of someone riding one train, as a train at rest being passed by a train going at about 249 Mm/s (all speeds measured with respect to the track). But neither case will look at all like a sped up version of the 10 m/s case. It will look more like this. High-speed physics isn't at all like a faster version of low-speed physics. Fundamentally the reason you can't "add velocities" is that the space of velocities is hyperbolic. When you're dealing with small relative speeds, you're dealing with a small part of the space that's approximately flat, so you can pretend it's exactly flat and use vectors. When you're dealing with large relative speeds this doesn't work any more. Hyperbolic geometry is unfamiliar, but there's a close analogy with spherical geometries like the surface of the Earth. When you're only mapping a small part of the Earth you can pretend it's flat, but you can't make a flat map of a large part of the Earth without distortion. It's the same with velocities. The usual meters-per-second velocities measured with respect to a "reference frame" are actually the Klein model of hyperbolic space, which is a kind of map projection. That article unfortunately has no pictures, but the Poincaré disk model is similar and has some pretty pictures. See also Escher's Circle Limit woodcuts. All of the like-colored shapes in those pictures are the same size; they just look different because of the projection. If you scale the Klein disk so that it has a radius of c, and say that O is the center of the disk, P and Q are points 160 Mm/s from the center in opposite directions, and R is a point 249 Mm/s from the center, then the real distance between P and Q (in the hyperbolic space) is the same as the distance between O and R. That's why those two cases of the passing-train problem are the same. They look different on the map, but that's because it's distorted near the edges. There's nothing deep about the "velocity addition formula"; it's just a side effect of the traditional use of the Klein projection, which is just a historical accident. -- BenRG (talk) 19:30, 16 March 2008 (UTC)[reply]
I'll munch on this answer for the coming month. Thanks! :) I've only done so much physics, but I think I'll be able to understand the above with the help of some physicist friends. User:Krator (t c) 22:42, 16 March 2008 (UTC)[reply]

Hi. This is not homework. So, an apple with a mass of 100 grams dropped from the height of one metre, negligizing air resistance, releases approximately one joule of energy? However, at what downward speed is the apple travelling at by the time it hits the ground? What about from a height of 10 metres? I know acceleration of gravity is just under 10 m/s, but how to calculate the velocity the object is travelling at by the time it hits the ground? If an object is thrown upwards, and say is is travelling at 40 m/s, will it be travelling at 30 m/s upwards after one second, and 20m/s after 2 seconds? Will it start to fall down after 4s, or is acceleration more complicated than that? Say is starts falling down, it will be travelling at 10 m/s after one second, but what about after 2 seconds, will it be travelling at 20m/s? How can we calculate based on height? Also, how does one joule compare to the equivalent of one kg of force? Does the density of the impacted surface usually affect the amount of energy released, such as in asteroid impact simulation given by LPL Arizona? If an object weiging 1 kg is travelling forward without downward interference from gravity, such as pendulum or rolling, and air resistance and friction is negligible, and travelling at 10 m/s, how many joules of kenetic energy? What about 1 kg at 20m/s? Is this calculation linear or exponential? If a (hypopthetical) bullet (average) was shot directly upwards, how high would it reach? If it was shot downwards from the altitude of 1 km from the ground, would it speed up by then because of acceleration or slow down because of air resistance? If you and a ball were falling out of the sky at the same altitude and the same velocity, say at 100m/s, can you safely catch the ball? If an object were to explode upon impact, caused by the impact alone and not some inner explosive force such as a grenade, so it's not a bomb, would the released energy be the same as if they didn't explode? If two bullets hit each other exactly head-on, with exactly the same mass and exactly the same speed, and were travelling exactly horizontal, and they have exactly the same volume and exactly the same density, and air currents and resistance were negligiable, what would happen? If a gamma-ray photon and a radio-wave photon hit each other head on, and didn't miss, would the resulting annilated matter, if any, travel more in the direction the gamma ray was travelling because higher energy, or is it impossible because different wavelengths? Can black holes implode? Have I asked this before? Thanks. ~AH1(TCU) 15:32, 15 March 2008 (UTC)[reply]

  • An apple with a mass of 100 grams dropped from the height of one metre, negligizing air resistance, releases approximately one joule of energy?
Correct.
  • However, at what downward speed is the apple travelling at by the time it hits the ground? What about from a height of 10 metres? I know acceleration of gravity is just under 10 m/s, but how to calculate the velocity the object is travelling at by the time it hits the ground?
It's 10 m/(s^2). velocity=acceleration*time, distance=time^2*acceleration/2. You know acceleration and distance. Solve for time, then velocity.
  • If an object is thrown upwards, and say is is travelling at 40 m/s, will it be travelling at 30 m/s upwards after one second, and 20m/s after 2 seconds? Will it start to fall down after 4s, or is acceleration more complicated than that? Say is starts falling down, it will be travelling at 10 m/s after one second, but what about after 2 seconds, will it be travelling at 20m/s?
That is all correct.
  • How can we calculate based on height?
If you're talking about calculating speed from hight, you already asked that.
  • Also, how does one joule compare to the equivalent of one kg of force?
I assume you mean the weight of a kg, which is about 10N. It's one-tenth of a meter times as much. That is, pushing with a force of 10N over a tenth of a meter uses one J. This is what happens when you mix units.
  • Does the density of the impacted surface usually affect the amount of energy released, such as in asteroid impact simulation given by LPL Arizona?
No, assuming mass, rather than volume, is constant, but it does affect the way in which it is released.
  • If an object weiging 1 kg is travelling forward without downward interference from gravity, such as pendulum or rolling, and air resistance and friction is negligible, and travelling at 10 m/s, how many joules of kenetic energy? What about 1 kg at 20m/s? Is this calculation linear or exponential?
Kinetic energy=mass*velocity^2. It is neither linear nor exponential.
  • If a (hypopthetical) bullet (average) was shot directly upwards, how high would it reach?
I don't know the speed of the average bullet, or the effects of air-resistance, but if you know the former and ignore the latter, distance=time^2*acceleration/2. Air resistance would be far from negligible.
  • If it was shot downwards from the altitude of 1 km from the ground, would it speed up by then because of acceleration or slow down because of air resistance?
Terminal velocity for a bullet is much slower than what speed it's fired at. It would slow down, and approach terminal velocity.
  • If you and a ball were falling out of the sky at the same altitude and the same velocity, say at 100m/s, can you safely catch the ball?
Air resistance would make a much larger difference for the ball, then for you, but if you ignore that, you'd both be in free fall and the ball would seem to hover, making it both safe (insomuch as falling out of the sky is safe), and extremely easy.
  • If an object were to explode upon impact, caused by the impact alone and not some inner explosive force such as a grenade, so it's not a bomb, would the released energy be the same as if they didn't explode?
The energy remains constant. Unless there's some way it's storing the energy, the amount released will be the same. Whether or not it explodes will make a difference in how the energy is released, and thus its destructive power.
  • If two bullets hit each other exactly head-on, with exactly the same mass and exactly the same speed, and were travelling exactly horizontal, and they have exactly the same volume and exactly the same density, and air currents and resistance were negligiable, what would happen?
They'd either bounce off of each other or fuse. I've seen a picture of two bullets that collided in midair and fused, so I know that one's possible. The other is probably also possible. I'd say it depends on the bullet.
  • If a gamma-ray photon and a radio-wave photon hit each other head on, and didn't miss, would the resulting annilated matter, if any, travel more in the direction the gamma ray was travelling because higher energy, or is it impossible because different wavelengths?
From some point of reference, they're the same wavelength. I think they can annihilate each other, but I don't know if they have to. If they do, momentum will be conserved, so the resulting matter/antimatter or whatever gets released will be moving more in the direction of the gamma ray.
  • Can black holes implode?
They're as imploded as it gets. If our current physics are wrong, and they can implode more, there'd be no way to tell, as no information about the internal structure can leave the event horizon. — DanielLC 18:02, 15 March 2008 (UTC)[reply]
AH1, are you taking the piss? SpinningSpark 18:09, 15 March 2008 (UTC)[reply]
What?!? This? *confusion*. ~AH1(TCU) 18:48, 15 March 2008 (UTC)[reply]
Never new that was Hiberno-English. Amazing what you learn on Wikipedia! SpinningSpark 19:08, 15 March 2008 (UTC)[reply]
It may be more common in Ireland, but it's plenty common in south England. Algebraist 20:56, 15 March 2008 (UTC)[reply]
And in New Zealand, too. Not an expression to use when your among Americans, who are likely to interpret it as a golden shower. StuRat (talk) 18:04, 16 March 2008 (UTC)[reply]
You never know your luck, though (if that's the sort of thing you like). You might have randomised yourself into the path of a fellow ... er, urinee, and if you don't put it out there, you might miss a "golden" opportunity. -- JackofOz (talk) 18:35, 16 March 2008 (UTC)[reply]
Hi. By the way, great answer to my question. However, what did you mean by asking if I am taking the piss?! I still don't get it, so before I list some ridiculous possibilities, can you explain what you meant originally by this so I can solve this mystery that has been plauging me for the past 90,000 seconds? Thanks. ~AH1(TCU) 19:32, 16 March 2008 (UTC)[reply]
It means something like "Are you teasing us ?". In the context of this post I would guess they think you may be asking so many question just to annoy us. You might want to ask fewer questions at a time, to avoid this type of reaction. StuRat (talk) 22:47, 16 March 2008 (UTC)[reply]
The best American translation for "taking the piss" is "pulling (my) leg", I think. —Tamfang (talk) 22:23, 19 March 2008 (UTC)[reply]

Effect of carbon dioxide on photosynthesis

Hi all have a question I couldn't find the answer to. I know that the carbon dioxide concentration in air is usually about 0.035% but plants can benefit from much higher concentrations, up to 0.5% I believe. If the concentration is increased further I have heard it has a detrimental effect on photosynthesis causing the stomata to close. My question is why does a large concentration of carbon dioxide in the air cause the stomata to close? Surely they would benefit from the carbon dioxide. Any help would be much appreiciated. —Preceding unsigned comment added by 172.201.120.93 (talk) 15:45, 15 March 2008 (UTC)[reply]

This is a very complicated problem; see, for example, abstract of this paper. I am afraid there is no simple answer to this one. Sorry. --Dr Dima (talk) 16:08, 15 March 2008 (UTC)[reply]

Wikimania - is it an academic conference?

Is Wikimania considered an academic conference? Are publications presented there published in peer reviewed outlets? --Piotr Konieczny aka Prokonsul Piotrus| talk 16:22, 15 March 2008 (UTC)[reply]

Not routinely, though some presenters will be academics who also choose to publish their work in academic journals. Dragons flight (talk) 16:26, 15 March 2008 (UTC)[reply]

Metabolism and conservation of energy

For people or creatures that have a fast metabolism, where is the additional energy they consume going? (Mostly I care about people, but one would think that general principles would be the same across species.) I know certain studies have found that people who fidget generally are skinnier, so maybe some of it is mechanical energy that is used in gross motion. Besides that it seems like you would either need to be passing improperly digested food or translating that energy into heat somewhere. But do people with high metabolisms really have higher body temperatures? (Or have dilated surface blood vessels so that they give off heat more quickly, or something like that?) Any insight into this would be welcome. Thanks! Mangostar (talk) 17:51, 15 March 2008 (UTC) (I just noticed I wasn't that precise. I understand that technically someone who fidgets more and is skinny because of it doesn't really have a faster "metabolism". Still, my general question is the same. I guess there are three categories that I'm seeing initially--dissipation of energy through motion, poor digestion (seems implausible), and various internal processes/metabolism that would generate heat... Mangostar (talk) 17:57, 15 March 2008 (UTC))[reply]

I could be thinking about this the wrong way myself, but I don't think there's such a thing as dissipation of energy through fidgeting per se... I think that has to be heat dissipation as well. Say you're moving your foot from side to side... when you start a leftward stroke, you're turning your body's chemical energy into kinetic energy (and heat, because of the inefficiency of muscles). But then you have to decelerate your foot (if you want to keep it), so you're turning that kinetic energy into... heat, as far as I can figure. It could be a bit of air motion, but it doesn't seem like that would account for much. So unless the air motion is more important than I'm guessing, then it does seem like people with faster metabolisms must have higher body temperature. But I'm not a physicist; I'm probably mixed up somewhere here. --Allen (talk) 20:48, 15 March 2008 (UTC)[reply]
As for the dissipation thing I just meant that there immediately you are creating mechanical energy. Obviously that is somehow transformed into mechanical energy in the air around you as well as heat in the air and in your muscles etc. I just meant to distinguish that from heat that is generated from chemical reactions in your body. Mangostar (talk) 22:45, 15 March 2008 (UTC)[reply]
Mangostar's original thought is on the right track - conservation of energy. Basically, the energy expended plus the energy stored by an organism must be equal to the energy ingested. In practice, estimating those quantities is difficult; for instance, the energy expended will include both mechanical energy and heat energy (as Amcbride noted, life here on earth has not yet developed muscles that can work in reverse by converting kinetic energy into chemical energy). As a general principle, if an organism uses more energy, it must consume more energy, so the short answer to your question is that people who have faster "metabolisms" must eat more food, but it's really more complicated than that. And I don't know about the body temperature thing, but it looks like the article Warm-blooded has some good info on that topic. --Bmk (talk) 20:58, 15 March 2008 (UTC)[reply]
I guess my big question here is, are people who are naturally skinny normally just hotter than people who are naturally fat (assuming they eat the same amount of food and move the same amount)? To the extent it's true that some people are "naturally" thin in that their bodies are just extremely inefficient at energy (i.e. fat) storage, where is all that energy going? Mangostar (talk) 22:45, 15 March 2008 (UTC)[reply]
I'm not sure if that has anything to do with your question but when I eat cheese or fatty meat I sweat. So does my brother. People sometimes ask me if there is something wrong so maybe it is not that common. I don't know. 200.127.59.151 (talk) 01:19, 16 March 2008 (UTC)[reply]
I don't know the answer, but I would like to add that this is a really good question.
It's well known that different people have higher and lower metabolic rates. People with lower metabolic rates are in a sense "more efficient" at utilizing their food, in that they waste less of the energy derived from their food and therefore either need less of it or put on weight faster. There's also the terribly unfair result that if you're overweight, and go on a diet and eat less food, your body decides that you're "starving" it, and decreases its metabolic rate further, nullifying your efforts.
But Mangostar is absolutely right: that energy has to go somewhere. And it pretty much has to be released as waste heat -- that's generally the end result of any energy-transfer process.
I'm not aware that people with higher metabolisms have higher temperatures, but if we think about it, that couldn't be the answer anyway; if the excess energy just went into heating you up, you'd get hotter and hotter until you incinerated. So the heat has to be lost to the environment. (Now, it's true, given a constant ambient temperature, hotter objects lose heat to the environment faster, but I can't see a difference of a degree or two in human body temperature making much of a difference here.)
Perhaps people with a higher metabolic rate breathe faster, and thus lose more heat in the warmed air they exhale. —Steve Summit (talk) 15:09, 16 March 2008 (UTC)[reply]
This is just ignorant speculation, but maybe we are looking at this from the wrong way around. Thinner people have less fat to provide insulation, and hence it seems likely that they will dissipate internal heat more rapidly. Maybe skinny people need to burn energy more rapidly than the rest of us simply to maintain the same body temperature. Dragons flight (talk) 17:19, 16 March 2008 (UTC)[reply]
I had a grandmother who must have had a migh metabolic rate, because she was a "ball of energy". She would vaccuum every day, go to the grocery store every day, etc. I imagine she burnt the energy as heat which was dissipated as she rapidly moved around the home, to the store, etc. A person in motion should transfer heat to the air more quickly than one who stays still. If they sweat then evaporative cooling will lower the temp, so there the energy is also going into vaporized water, just like a flame under a pot of boiling water. StuRat (talk) 17:56, 16 March 2008 (UTC)[reply]

Skeleton

File:Skelotor.JPG
What could this be
Could it be one of theses

Hi All.
I came across this skeleton at the Middle Head Fortifications in Mosman, New South Wales. Does anyone know what this could be. Cheers. --User:Adam.J.W.C. (talk) (talk) 23:36, 15 March 2008 (UTC)[reply]

It might be a bat, though that little "hand" would have to be a foot of that were the case. Has more teeth than most species of bat, I reckon. --98.217.18.109 (talk) 00:17, 16 March 2008 (UTC)[reply]
Apart from the head, I though the rest of it looked quite human, I suspected that it could be a bat due to the fact that there is some kind of black colored skin or something wrapped around the skeleton. The only animal that I have come across in abundance in that area are hairs and rabbits. Cheers. --User:Adam.J.W.C. (talk) (talk) 00:31, 16 March 2008 (UTC)[reply]
The forelimb is absolutely not bat-like. It would be easier if there was a better view of the head. Check that. Rather, the front teeth on the bottom jaw (which is pointing straight down) look rather rodent-like. But that's a bit of a guess. -- Flyguy649 talk 01:31, 16 March 2008 (UTC)[reply]
It might not be a forelimb. Imagine it is a hind limb. It's not unlikely that the skeleton is jumbled up. I thought it looked rather rodent like, but I've never seen rodents with long arms like that. However bat legs can be quite long. (I am no mammal expert, though.) --98.217.18.109 (talk) 04:10, 16 March 2008 (UTC)[reply]

I guess one should not discount any one of a small number of Australian animals that have the general appearance of rodents - bandicoots, possums, gliders, potoroos etc., and with which we are not familiar in the skeletal state. Definitely not a bat with a 'hand' like that. Richard Avery (talk) 08:31, 16 March 2008 (UTC)[reply]

Possum is a real possibility. Bandicoot, potoroo, no way. Glider, can't tell, not enough to compare with. You can google things like "Possum skull" and see if it looks anything like the skull show there, it's an easy way for ruling things out (doesn't always rule things in, though). The more I look at it, the more I think the top jaw is broken—it is lacking symmetry with the bottom one and would have a tremendous underbite as is. --98.217.8.46 (talk) 15:44, 16 March 2008 (UTC)[reply]

The Great Gazoo ? :-) StuRat (talk) 17:36, 16 March 2008 (UTC)[reply]

The lower incisors tempt one to say it's a rodent, but there is no diastema (dentistry). This question was posted in various places, and I think the most clueful reply is at Talk:Rat#Who_is_skelotor. William Avery (talk) 19:21, 16 March 2008 (UTC)[reply]

I'm pretty sure its a sugar glider Petaurus breviceps, might be another species. I tell by the broad hands (all four much the same) with parallel fingers bunched into fists. I recognise the anatomy from removing them from water tanks through the bottom 25mm tap hole. This is why you must always cover the overflow pipe with netting. Polypipe Wrangler (talk) 23:27, 16 March 2008 (UTC)[reply]
Mmmm... I don't know. The skull doesn't look right. Compare with this skeleton of a sugar glider (from one of my favorite stores—The Bone Room in Berkeley). The body type looks right though—maybe some other species of glider? --98.217.8.46 (talk) 04:17, 17 March 2008 (UTC)[reply]
Have a look at this possum skull [7]and see what you think, especially the lower jaw dentition. Richard Avery (talk) 08:34, 17 March 2008 (UTC)[reply]
That looks extremely plausible. --Captain Ref Desk (talk) 14:35, 17 March 2008 (UTC)[reply]
Just asking, but can a foetus develop a skeleton? --Zacharycrimsonwolf 14:24, 20 March 2008 (UTC)[reply]
Ever see a newborn without bones? —Tamfang (talk) 17:15, 2 April 2008 (UTC)[reply]

March 16

Toxicity of non-stick pan lining?

I know it's not the recommended methd of cleaning them, but I often use rough scrubbing pads on my cheap no stick pans. The coating does flake off slowly. If I am ingesting this material in small quantities, should I be worried?--68.237.2.101 (talk) 03:15, 16 March 2008 (UTC)[reply]

See Polytetrafluoroethylene. Basically if it's Teflon and hasn't been overheated it is chemically inert and should pass straight through your body. --antilivedT | C | G 03:42, 16 March 2008 (UTC)[reply]

I personally develop a bad skin condition whenever I eat something cooked in Teflon, so it can't be entirely harmless. And no overheating is not the issue. Dupont has a vested interest in having you believe their stuff is safe if used properly, so what they tell you cannot be trusted.

If I were you I'd err on the side of caution and get some regular cookware -- stainless steel, cast iron, copper if you're wealthy. But it really depends on how concerned you are about your health and whether doing a little extra scrubbing to clean normal cookware is something you'd be willing to do. If you aren't a big-time health nut it's probably not something you need to worry about. Vranak (talk) 15:20, 16 March 2008 (UTC)[reply]

This is a contentious issue, and as far as I know there isn't a definitive answer yet. As Vranek notes, the manufacturer has a vested interest, so don't believe everything you hear.
Here are the facts as I understand and observe them:
  • Teflon and related compounds are quite biologically inert; they'll pass through you like a rock.
  • "Burnt" Teflon is reasonably nasty, and is to be avoided.
  • The non-stick coating on non-stick pans does flake off. More so with cheap pans.
  • The non-stick coating is much more likely to flake off it it's been overheated, i.e. burned. Danger, Will Robinson.
  • The non-stick coating is easily damaged (and made much more prone to both burning and flaking) by the use of metal implements, either while cooking or cleaning.
My own conclusion is that the potential toxicity of burnt-and-flaked-off Teflon is suitably worrisome that it's worth avoiding.
If you want to clean your cookware so vigorously that the non-stick coating flakes off, pretty soon it isn't non-stick any more, and you might as well use non-non-stick cookware, instead. Then you don't have to worry.
Personally, I don't like non-stick cookware, because I can't be bothered to be careful of it. (Plastic spatulas are teh pitz; they melt if you leave them in.) I find that with proper cooking techniques, my ordinary (non-non-stick) cookware doesn't stick badly enough to worry about. YMMV. —Steve Summit (talk) 15:36, 16 March 2008 (UTC)[reply]
Also, steel wool is absolutely indispensible for regular cookware. Vranak (talk) 17:00, 16 March 2008 (UTC)[reply]
Who uses plastic spatulas? Silicone or wood are much better Nil Einne (talk) 07:30, 18 March 2008 (UTC)[reply]
And why would you use steel wool on a non-stick pan ? It's guaranteed to destroy it. You might as well polish your car with steel wool. StuRat (talk) 17:31, 16 March 2008 (UTC)[reply]
I said it's good for regular cookware. i.e. non-non-stick cookware. Vranak (talk) 19:33, 16 March 2008 (UTC)[reply]
Right. I wasn't replying to you; please note the indentation. Apparently the OP is also using steel wool, or something just as bad, on non-stick pans. StuRat (talk) 22:39, 16 March 2008 (UTC)[reply]
Well, a rough scrubbing pad could mean many things. What matters more than the roughness is the abrasiveness. If the grit in the scrubbing pad is too hard, it will scratch the teflon. The OP may want to invest a non-abrasive nylon scrubbing pad. Or switch to Cast-iron cookware, which has the benefit of improved heat retention. Dforest (talk) 21:44, 17 March 2008 (UTC)[reply]


(edit conflict)Hi. I've watched infomercials on TV way too often, and on one of them, there is a solution to all your problems: SmartWare. Try Googling it or something. Remember, this offer is not avalible in stores. Don't delay, order your SmartWare today! Call within the next 30 seconds and recieve my special bundt cake pan, as well as our cake stencil: Turn a ho-hum cake into a yum-yum cake! You should have at least two to three bowel movements a day. Less than one bowel movement a day is irregular, and can lead to constipation. (Dr. Ho) Hope this helps. Thanks. ~AH1(TCU) 19:39, 16 March 2008 (UTC)[reply]
Yes, it seems as if infomercials are full of Ho's selling crap. StuRat (talk) 22:41, 16 March 2008 (UTC)[reply]

Horizontal search functions (in visual perception)

Does anybody have a clue what could be meant by the expression horizontal search functions in the following sentence: "Targets that yield horizontal search functions are assumed to reflect visual primitives, the basic building blocks of perception." (PS this is a sentence in my textbook, not a sentence found in Wikipedia.) Lova Falk (talk) 15:11, 16 March 2008 (UTC)[reply]

It's just a guess, but they may refer to a heirarchy of objects, like the following example:
  Humans
  |    |
Men  Women
     |   |
   Barb  Sue
So, a horizontal search would be a search at the same hierarchy level. For example, "Is that Barb ?", "No", "Then is that Sue ?", etc. StuRat (talk) 17:26, 16 March 2008 (UTC)[reply]
This sounds very close to a classification project I was working on. In my project the term "horizontal" was used literally to suggest the conjunctive sense of "and" when a glyph was used as a modifier of another glyph. "Vertical" on the other hand was used to suggest the conjunctive sense of "or". Thus a word is considered a "horizontal" set of glyphs and a sentence is then a horizontal set of words, whereas alternate glyphs, words, phrases, and sentences are deemed "vertical".

It would help if you gave us a little more background on the textbook and the section in question. One isolated sentence from any textbook is going to be hard to make sense of without context. Give us the whole paragraph and the title of the book, at least. Otherwise people are going to be just guessing blindly. --98.217.8.46 (talk) 18:12, 16 March 2008 (UTC)[reply]

Is Energy relative?

This is perhaps a very silly question, but nevertheless, it has troubled my from time to time. What's wrong with the following line of reasoning:

  • Velocity is relative (depends on the reference frame)
  • Therefore, its time derivative, acceleration is relative
  • Force, which is acceleration times mass, is relative (or, we can say, momentum and its rate of change are relative)
  • Thus, quantities like kinetic energy (m*v^2/2) and work (force x displacement) are also dependent on reference frame.

Is this last proposition correct? deeptrivia (talk) 16:02, 16 March 2008 (UTC)[reply]

Yes, but the second and third aren't. Acceleration isn't inherently a frame-dependent thing. A speedometer can only tell you the relative speed between two objects (for example, your car and the road), but an accelerometer can tell you its own intrinsic acceleration without reference to anything else. Inertial navigation is based on this principle. -- BenRG (talk) 18:37, 16 March 2008 (UTC)[reply]
So, there is such a thing as an absolute acceleration? If so, how can we ever find it? For example, the earth itself accelerates as it revolves around the sun, and the sun might be accelerating around the center of the galaxy. Is it that these effects are small, and in principle measurable? Do the sensors we use include all these accelerations? deeptrivia (talk) 21:16, 16 March 2008 (UTC)[reply]
Yes these small accelerations are absolute and measurable. The accelerations ,that are a direct consequence of the gravitational forces in the universe, are making the changes in velocity that make astronomical bodies orbit each other, rather than move in a straight line as they would do if no force operated on them. GameKeeper (talk) 23:16, 16 March 2008 (UTC)[reply]
It's a key tenet of general relativity that accelerations are not relative except that they can't be distinguished from gravitational fields. You can use an accelerometer to measure accelerations; the most simple of which is a cup of coffee. Start to accelerate it, just a little—you'll start to spill it. But if you have it in an inertial frame, no matter how fast you are going (say, 67,000 miles an hour), it'll sit there placidly.
Einstein used to comment that theory of relativity was something of a misnomer: what's important is not what is relative, but the very few things which are not relative, like the speed of light. It is from finding the few things which are not relative—and the implications of that—that the genius of his theory comes. --98.217.8.46 (talk) 00:59, 17 March 2008 (UTC)[reply]
KE and momentum both depend on the reference frame —Preceding unsigned comment added by 79.76.144.62 (talk) 02:19, 17 March 2008 (UTC)[reply]

DBasing the money supply

Wouldn't it be possible to enter the serial numbers that are already on US currency into a database, record the serial numbers of any currency that is stolen in the DB, scan all bills whenever they are tendered at a business and run a database check, and thus instantly know when any stolen money is tendered ? The weak point in the chain seems to be getting businesses to pay for the devices, but I imagine some tax incentive could be offered to get them to agree. StuRat (talk) 17:21, 16 March 2008 (UTC)[reply]

Such a system could easily be built based not on a piece of new equipment but merely on the upload of a digital picture of a bill using a regular scanner. This would also help everyone including the Secret Service to determining which bills were counterfeit. If yours turned out to be counterfeit then of course you would be stuck with the loss unless you could spend it quickly somewhere a scanner was not in use and the system did not require your name and to turn in the bill if it was found not to be legitimate. —Preceding unsigned comment added by 71.100.174.10 (talk) 17:43, 16 March 2008 (UTC)[reply]
I don't quite understand what you mean, StuRat. What is preventing the counterfeiter from copying legitimate serial numbers onto fake bank notes? --Bowlhover 18:01, 16 March 2008 (UTC)
Serial numbers are unique and associated with other components of a bill which can not be duplicated exactly. Pictures of every bill issued already exist. Of two bills with the same serial number figuring out which one is real and which one is not is a relatively insignificant problem. —Preceding unsigned comment added by 71.100.174.10 (talk) 18:11, 16 March 2008 (UTC)[reply]
I was talking about the theft of valid money, not countefeit money. However, the system could be somewhat useful in apprehending counterfeiters, too. If, for example, somebody tries to spend a large amount of currency and several of the serial numbers are listed as being in the possession of other banks and businesses, it might look suspicious and security might want to detain the big spender until the police arrive. StuRat (talk) 18:38, 16 March 2008 (UTC)[reply]
The potential cost of maintaining and using a database and a system that would hold information for all of the billions of US bills in circulation would be prohibitive and probably outweigh whatever is lost in counterfeiting. The labor required to enter in every bill one received and, presumably, all of the bills that one gave out (how else would you know which ones were stolen?) would be unreasonably large. Is it possible? Yes, sure. But wildly impractical. There's no way the gains would be worth the costs. --98.217.8.46 (talk) 18:02, 16 March 2008 (UTC)[reply]
And just to illustrate: say I run a business, even one that deals with a relatively small amount of money, like a convenience store. Let's pretend I have an automated machine that'll scan all of the money I put into the till, and registers it in some sort of giganormous Treasury database which always works (despite store millions of new transaction entries a day from around the country) and is miraculously fast enough to not make every bill check take as long as a credit card takes to process (about 10 seconds a card swipe or so, so if you paid for a $5 item in ones then you've got to sit around for a minute). (And note: yes, the business WOULD need to check with the centralized database every time they received a bill for change, because they too are also in the process of looking for "stolen" money from other businesses, right?) OK—best case scenario so far?
Now a customer comes, and I have to take the money they give me, scan it, and then take the money I give them, and scan it. Because I don't keep track of what I've taken out of the till, when my store gets robbed, I won't be able to distinguish between "valid" money that I gave out as a change and "stolen" money. Let's even pretend that this was somehow practical, and the system says that a mile away, someone put one of my five dollar bills into a till at a grocery store. The police are called, the suspect kindly waits for them to arrive, and everybody has a nice conversation about where that five dollar bill came from. Maybe, in an ideal world, the suspect matches a perfect description from the robbed cashier, they put him in a line-up, they identify him, they convict him. Bravo. But what if he doesn't match? What if, say, he got the five dollar bill from someone else? OK, now we've got a complicated situation here—maybe we try to shake him down and find out where he got that bill from. And so on and so on. The question is: how many times do you have to "catch" the wrong person for 1. people to think this is a waste of time, 2. businesses to think this is a waste of resources, 3. law enforcement to think this is a waste of resources? Not many. --98.217.8.46 (talk) 18:07, 16 March 2008 (UTC)[reply]
Okay you want special equipment. NCR can build a cash register with a bill scanner inside above the drawer. You put the bill in the drawer and the cash register does the rest. As for a waste of time... such a system has the potential to waste a crook's time rather than the police by simply going the extra step of requiring validation of the bill before it can be spent. Bill gets stolen the system is notified by pressing the "theft" key and poof the bill is no longer valid until re-certified by the police. —Preceding unsigned comment added by 71.100.174.10 (talk) 18:16, 16 March 2008 (UTC)[reply]
There is the voluntary project, http://wheresgeorge.com -- kainaw 18:17, 16 March 2008 (UTC)[reply]
Too many holes to be worth the effort. —Preceding unsigned comment added by 71.100.174.10 (talk) 18:23, 16 March 2008 (UTC)[reply]
Still a ridiculous suggestion. Do you think the police want to spend their time becoming "bill re-certifiers"? No law enforcement agency in the world is going to want to take on that task. The best way to catch criminals involves talking to other people, looking at specific forensic, not trying to track down each and every physical dollar. --98.217.8.46 (talk) 20:28, 16 March 2008 (UTC)[reply]
Thanks for confirming everyone's concept of the police as worthless lazy donut eating taxpayer supported freeloaders and do nothings until the opportunity arises to write a parking ticket or a ticket for driving near enough to the speed limit that in their minds it might as well be speeding. Not to mention being self-righteous oppressors of Berkley student freedoms. —Preceding unsigned comment added by 71.100.174.10 (talk) 20:57, 16 March 2008 (UTC)[reply]
I tend to think of police that way myself, as the only interaction I seem to ever have with them is when they extort money from me via tickets. Since I am neither a pretty woman good at flirting nor another policeman or government official immune to traffic laws, I never get out of a ticket, either. When anyone is driving like a fool and actually endangering lives, no police are ever to be found. I even saw one instance where police apparently put up a barracade and stopped traffic with the sole purpose of ticketing those who drove around it. StuRat (talk) 22:30, 16 March 2008 (UTC)[reply]
Many of the limitations listed are only current limitations, which can hopefully be solved soon. I've never understood why credit card approval takes so long, and increasing bandwidth, etc., should make this process quicker soon. Also, rather than "dialing in" for each bill, something like a DSL system can be always on and ready for a check. Looking up a unique serial number in a database is one of the quickest DB operations there is. Initially, perhaps only banks would record the location and distibution of currency, with businesses merely running a check, not updating the database. A few years later on, businesses could update the records, too. Note that the full system could also be used as an anti-terrorism tool, as terrorists often need to move large sums of money around. If they go to a bank and withdraw large sums of money, the serial numbers could be tracked and we would know, say, if they used it to buy weapons and explosives. It would also be interesting to know if large sums of money given to Islamic charities are being used to purchase weapons. StuRat (talk) 18:50, 16 March 2008 (UTC)[reply]
The whole thing is totally impractical with paper money. And at least in the United States, no arms dealers are going to start using such a system—a lot of the transactions would be "off the grid", especially for those involving potentially scandalous or illegal activities. There are better ways to check how businesses and charities spend their money if one is inclined to do so.
It doesn't matter whether reading the index of a bill is a quick operation—even a relatively quick operation needs to be able to scale. And every complex system is going to have major downtimes, bugs, etc. in it.
My money (haw haw) would be on using electronic money and getting rid of paper money long before any of this would ever become practical, and I still think that even if you did have a system of tracking paper money it would still be wildly impractical. Certainly more impractical than just making everyone switch to purely electronic currency and skipping the paper step altogether. If countries make universal IDs mandatory (which I suspect they will in the next decade or so), that would make it all the easier. --98.217.8.46 (talk) 20:24, 16 March 2008 (UTC)[reply]
Certain "high-risk" businesses, like gun shops, pawn shops, etc., could be forced to comply by law. It would be easy enough to verify, just have an undercover cop go in with a "bad bill" occasionally, and, if they don't report it and refuse to sell the gun, give them a whopping huge fine. This method would allow the police to make a nice profit, ensuring that they would actually enforce the law. StuRat (talk) 22:21, 16 March 2008 (UTC)[reply]
(And if you're worried about people buying guns, why not make them register the guns? There are far less guns than dollars, and they're much larger, much more worth the effort. If you think people don't want you to track their gun-buying habits, or that they can find ways around that check, why would you assume money would be any easier?) --98.217.8.46 (talk) 20:30, 16 March 2008 (UTC)[reply]
People don't usually leave guns around after a crime. And, if they want to get around the tracking, it's a lot easier to remove a serial number from a gun than from every bill you ever have. StuRat (talk) 22:12, 16 March 2008 (UTC)[reply]
What has been learned from previous experiments to track money is that criminals create their own from of money and find other ways to thwart tracking like through the construction, purchase and sale of real estate. A "company" buys a lot for twice what it is worth, builds on it and sells the lot and the building for eight times the going price of any adjacent property. People are so open to a free market under capitalism they do not even bat an eye. Money gets laundered like this around here all of the time and no even cares.
This might help track down the laundering operations, too. If an armored car is robbed, then, a year later, somebody buys a $100,000 home for $200,000 in cash, and many of the bills are those stolen from the armored car, the buyer would have some explaining to do. StuRat (talk) 22:15, 16 March 2008 (UTC)[reply]
Why would you assume that they'd be using that money anyway? If a ridiculous tracking system went into effect, what's to prevent someone going outside of the country to launder it? You severely overestimate the technical abilities needed to do such a thing while at the same time you underestimate how easy it would be to get outside of the grid. Again, there's nothing your system would offer that a generalized electronic money system would not, except that yours would cost billions more to produce and maintain in both infrastructure costs and operation time, and because you still were trying to rely on physical money you make it exceptionally easy to evade your proposed system. In any case, armored cars are not robbed all that often and the companies that run them are insured anyway (you're using a very clunky hammer to try and hit a pin of a problem). --98.217.8.46 (talk) 00:55, 17 March 2008 (UTC)[reply]
Moving currency offshore in large quantities has it's risks, too, as anyone repeatedly smuggling large sums of cash is likely to be caught. The armored car was just an example, it doesn't really matter where the currency was obtained illegally. And, whether they are insured or not is quite irrelevant, it only means the insurance company now wants the cash recovered instead of the original target. StuRat (talk) 01:07, 17 March 2008 (UTC)[reply]

Interesting question, StuRat, but lets take the suggestion even a step further:

  • Instead of carrying paper currency around, why don't we store the serial numbers of the currency that each person owns in a (verifiable and secure) database.
  • Then instead of presenting the paper bills to (say) a gun shop, one only needs to provide a secure ID and the database is remotely updated to transfer ownership records of the requisite amount of paper currency.
  • As a last step, clearly we don't need to store the actual "serial numbers" of the notional "paper" money, but only the amount of money that each person possesses at any particular point of time.

Looked upon this way, your proposed system is just an insufficiently ambitious version of an all electronic financial system, which (in developed countries at least) accounts for (guesstimate) 99.9...% of all financial transactions (once we account for all financial activities, i.e., not only retail consumer purchases).
Aside: We may well be living in the the last-few-decades of the physical currency era, and future generations may look back with amusement and disbelief at our primitiveness, just as we consider a barter economy obsolete and limited.
PS: The only relevant wikipedia article I could find is the one on electronic commerce but the scope of that is too narrow for our discussion. Abecedare (talk) 02:22, 17 March 2008 (UTC)[reply]

I don't agree with 99.9% of all transactions being electronic. Perhaps if measured by percentage of total transaction value, that would be correct, but many small transactions, like fast food restaurants, paying the paper boy, etc., still are largely done with cash. In a society without cash, bums would have a hard time getting a hand-out, too (I picture them carrying a credit card scanner to accept donations :-)). I, for one, use cash whenever possible, as there is no possibility of ID theft when I hand the cashier cash (and if a cashier doesn't take cash, do they become a "creditier" ?). StuRat (talk) 05:31, 17 March 2008 (UTC)[reply]
  • Yes, I meant 99.9...% by value (I'd be curious to know the true stats).
  • The bum wouldn't have to carry credit card scanners. All you'll need to do is note down their Database ID number (perhaps, their ID card will communicate wirelessly with your "mobile phone") and transfer funds electronically to their account, which they can then access instantaneously. Note that this perfectly implementable using today's technology, although clearly there are sociological, economic, ID theft and other non-technological issues. IIRC, on several university campuses one can pay petty expenses, such as vending machines, laundry etc using the student ID card and that may be a first step towards a cashless society with all its benefits and ills.
Cheers. Abecedare (talk) 06:01, 17 March 2008 (UTC)[reply]
An interesting effect of a cashless society would be that it might force many back to a barter economy. An illegal immigrant likely can't use a credit card, so, without cash, they would need to be paid in room and board and maybe pesos, if they weren't also eliminated. StuRat (talk) 19:31, 17 March 2008 (UTC)[reply]
Again hardened criminals and just ordinary people who want to avoid paying income and sales taxes barter all of the time without cash or debit cards getting involved. You sell a house to someone for $950,000 no one is going to require you to show where they actually paid or where you actually deposited the cash. Instead the house can be a payment in reality rather than a sale. You can avoid the grid without going offshore and by staying right here. Everyone knows how to work the system and how the system works. —Preceding unsigned comment added by 71.100.174.10 (talk) 09:30, 17 March 2008 (UTC)[reply]

artificially macerated food

As an informal experiment I have been macerating my meals using a food processor. The consistency of maceration is about the same as bean dip used for tacos but includes meats, vegetables and most components of a regular meal including condiments. I consume a beverage with the macerated meal. Not only do I become full with only half a meal but have been able to reduce the number of meals to one per day with occasional in between unmacerated light snacks. Not only do I seem to have more stamina for excercise but the pounds are beginning to come off. Where can I find scientific or medical studies which discuss artificially macerated food? —Preceding unsigned comment added by 71.100.174.10 (talk) 17:34, 16 March 2008 (UTC)[reply]

The only way I can see that helping is if the food is less palatble in that form, so you eat less of it. StuRat (talk) 18:54, 16 March 2008 (UTC)[reply]
It is somewhat less palatable but not to a degree that it is something one can not get use to in light of other benefits. What I need is a list of studies which have explored the question in depth. —Preceding unsigned comment added by 71.100.174.10 (talk) 21:14, 16 March 2008 (UTC)[reply]
I'm not aware of any studies. But what exactly would be the mechanism by which you would desire to eat less, if not that the food is unpalatable ? I suppose that might still be better than diet/weight loss methods like Alli, which apparently causes weight loss because people who take it crap their pants unless they stop eating fat. Talk about unpalatable ! StuRat (talk) 22:03, 16 March 2008 (UTC)[reply]
...you know perhaps NASA has a study. I just thought of that. Even Stanley Kubrick included it (pre-macerated food) in his film. —Preceding unsigned comment added by 71.100.174.10 (talk) 09:13, 17 March 2008 (UTC)[reply]
Check with aged-care studies, where macerated food (often reset in more attractive moulds) is used.Polypipe Wrangler (talk) —Preceding comment was added at 23:34, 16 March 2008 (UTC)[reply]
This question sounds strikingly familiar. --The Fat Man Who Never Came Back (talk) 23:58, 16 March 2008 (UTC):-)[reply]
Weathermen are notorious for earning good livings by giving different answers to the same questions every day. —Preceding unsigned comment added by 71.100.174.10 (talk) 08:55, 17 March 2008 (UTC)[reply]

How much human evolution is yet to come?

Is there any evidence regarding how close human evolution is to reaching an end state where it will stay? How much improvement is left to be made over the next few million years, assuming the planet remains habitable that long? NeonMerlin 18:22, 16 March 2008 (UTC)[reply]

That's easy. Whatever will make the human physically and mentally more efficient. An adaptation to the ever increasing abundance of pornography, for instance, might be a third hand growing out of one's thigh. ;D —Preceding unsigned comment added by 71.100.174.10 (talk) 18:25, 16 March 2008 (UTC)[reply]
As long as the environment changes, evolution will cause change in the species that survive. Even if the environment on Earth becomes static (which is not likely), humans are trying to go to other planets, which will lead to evolution to match the foreign environment. -- kainaw 18:41, 16 March 2008 (UTC)[reply]
Unfortunately, human evolution is likely to take a nasty turn, where those who irresponsibly produce the most children and abandon them are likely to pass on the most genes. Historically, the abandoned children would die, but in modern society, they are taken care of by the state. In the long run, however, the state will no longer be able to care for the expanding pool of abandoned children, and they will no longer survive. Or, perhaps before we get to that point, some fairly radical solutions, like forced sterilization, will become acceptable. StuRat (talk) 19:00, 16 March 2008 (UTC)[reply]
By chance, Stu, have you seen Idiocracy? Someguy1221 (talk) 20:36, 16 March 2008 (UTC)[reply]
No, I haven't, since doing so won't tend to increase the number of times I reproduce. :-) StuRat (talk) 21:53, 16 March 2008 (UTC)[reply]
And spending time on Wikipedia does? o_O Someguy1221 (talk) 23:38, 16 March 2008 (UTC)[reply]
Irresponsibility is not a genetic trait. There's no reason to assume those particular children are going to be any stupider than others. The complaint about "abandoned children" (or the mentally disabled, or the physically disabled) overburdening the state has been a nice scare fantasy of eugenicists for over 100 years now; the state seems to be doing just fine in that respect. It is pretty unlikely that compulsory sterilization will ever become in vogue again in Western countries. --98.217.8.46 (talk) 20:20, 16 March 2008 (UTC)[reply]
I don't agree that irresponsibility isn't a genetic trait (the Kennedy family may make my point for me). But, even if we say it isn't, children who are abandoned are also less likely to grow up to be responsible parents due to the lack of a good role model. As for abandoned children being cared for "just fine", this certainly isn't the case. In Romania, for example, they have a high fatality rate in underfunded and often abusive state-run institutions. The kids are often misdiagnosed as having severe mental and physical problems, which, while not true initially, becomes a self-fulfilling prophecy the longer they remain there. For example, since they are judged incapable of learning, they aren't given access to education, which, in turn, leaves them illiterate. The story isn't much better in the US, where children are often shuffled from foster home to home, some of which are abusive. Also, 100 years is an insignificant amount of time on the scale of human evolution, so I'm not surprised that the negative aspects haven't yet become overwhelmingly apparent. StuRat (talk) 21:47, 16 March 2008 (UTC)[reply]
I say "just fine" from the standpoint of the species and the standpoint of the survival of the state. This is a conversation about evolution, right? Abandoned children make up an insignificant part of the human gene pool, as does all of Romania. Even the United States doesn't stand for a whole lot on a biological level. If you want to talk about biology, talk about biology. If you want to talk about society, okay, talk about society. But don't pretend you are talking about one thing when you're talking about the other. For all of your talk about the effects of modern society on the long-term prospects of the human species, your views are tailored to very small issues and completely absent are any considerations that would take into account, say, the most populous societies on the planet. Don't wrap your ideology up in scientific terms and pretend it is science—that's another bad legacy of the eugenicists for you to avoid. --98.217.8.46 (talk) 00:51, 17 March 2008 (UTC)[reply]
When did I say Romania and the US were the only countries having trouble caring for abandoned children ? It's a problem in many countries. If you want to wait until we get to the point where nearly all abandoned children die to see it as a problem, then it will be too late to avoid the worst consequences by then. StuRat (talk) 01:22, 17 March 2008 (UTC)[reply]
You might be interested in the higher evolution.--Shantavira|feed me 19:23, 16 March 2008 (UTC)[reply]
I won't argue StuRat on intellectual grounds but I may say that his view strikes me as a touch pessimistic.
As to the original question, a quote from Hagakure:
A certain swordsman in his declining years said the following:
In one's life, there are levels in the pursuit of study. In the lowest level, a person studies but nothing comes of it, and he feels that both he and others are unskillful. At this point he is worthless. In the middle level he is still useless but is aware of his own insufficiencies and can also see the insufficiencies of others. In a higher level he has pride concerning his own ability, rejoices in praise from others, and laments the lack of ability in his fellows. This man has worth. In the highest level a man has the look of knowing nothing.
These are the levels in general. But there is one transcending level, and this is the most excellent of all. This person is aware of the endlessness of entering deeply into a certain Way arid never thinks of himself as having finished. He truly knows his own insufficiencies and never in his whole life thinks that he has succeeded. He has no thoughts of pride but with self-abasement knows the Way to the end. It is said that Master Yagyu once remarked, "I do not know the way to defeat others, but the way to defeat myself.
Throughout your life advance daily, becoming more skillful than yesterday, more skillful than today. This is never-ending.
I believe this to be true at the level of the human race as well as the individual. So, to answer the question with this in mind: there is no end-point in human evolution. Vranak (talk) 19:31, 16 March 2008 (UTC)[reply]
The question as to whether humans will keep evolving in a meaningful way (that is, having more than just incidental circulation of gene frequencies, etc.) and whether that will be for the better or the worst has been asked since the 19th-century. There isn't really a single right answer for it. Human selective pressures have been relatively weak on average for a long time now—we don't tend to let people die off who might otherwise, and no individuals tend to have trouble reproducing if that's what they want to do. Human genetics and human society are both sufficiently complex that the Social Darwinist beliefs that those with money/power/affluence must necessarily be genetically "superior" to those without them have long been shown to be very flawed. There is also the big question of human genetic engineering, which could lead into a variety of different possible directions. Lee Silver's book Remaking Eden suggests a number of interesting possibilities in it without making terribly large assumptions about the scientific advances needed. --98.217.8.46 (talk) 20:20, 16 March 2008 (UTC)[reply]
Assuming machines keep on tending to make life easier and humans do not suddenly come under some sort of drastic survival pressure, then any changes that occur will not be considered improvements by most of us. For instance, lack of toes will not instantly result in death or the inability to reproduce. It might have been so once upon a time, but not now. What is going to stop evolution getting rid of toes if they have no survival benefit? Or take our precious brains, very expensive in energy to maintain, but no survival value now that we have machines to do all our thinking for us. Probably first thing to go if evolution needs to save on energy for any reason. We will only be saved from this terrible future if survival becomes more difficult, or cosmetic genetic engineering becomes legal and cheap. SpinningSpark 20:33, 16 March 2008 (UTC)[reply]
If there is no selection for a trait then it's not going to propagate. And I don't know about you but my machines haven't learned to do any thinking for us; so far, all predictions of human intelligence being useless have proved pretty baseless. At the moment, for all of your fears of brains being cosmetic, most of our top paid professions require big brains, ergo the difficulty of getting into and succeeding in law school, business school, medical school, etc. If anything has characterized human society in the last hundred years it has been the increased importance of expertise and the slow shuffling towards something of a meritocracy (as opposed to the centuries of hard and fast rule by aristocracy). I wouldn't be all that worried.
As for toes, if there is no strong selective pressure for it, it won't catch on. You might think there is not selective pressure against it, but I'm betting that most toe-less individuals would probably disagree to some extent. And in any case, sometimes there is a benefit to missing a few. ;-)
Evolution and genetics is much more complicated than the "toes" example, in any case. Evolution in the face of heavy selective pressures can be quick in small populations, but in a very large population with a lack of selective pressures I think you're going to find mostly a regression towards the mean on the whole, e.g. not much change in either direction over the course of the population as a whole. (Ironically, that is what the founder of eugenics, Francis Galton, thought too. He thought this was stagnation. I consider it just to be stability.) --98.217.8.46 (talk) 20:41, 16 March 2008 (UTC)[reply]
Humans and the demands of their environments are quite mobile. Adaptation often takes the form of simply moving to a place where your deficiencies become meaningless and your talents are irreplaceable. Hence, Lovette goes to jail, Hilary and Barack hit the campaign trail and McCain heads for Iraq. —Preceding unsigned comment added by 71.100.174.10 (talk) 21:27, 16 March 2008 (UTC)[reply]
Yes it's true that brains are still valued in law, science and medicine etc. but my point is that this makes no difference to an individuals abililty to procreate. Hence, it is irrelevant to evolution. The unemployed make just as many children as brain surgeons, possibly more. SpinningSpark 21:45, 16 March 2008 (UTC)[reply]
Money appears to be the deciding factor when it comes to procreation even for the lowest class rather than brains unless brains translate into money. —Preceding unsigned comment added by 71.100.174.10 (talk) 21:56, 16 March 2008 (UTC)[reply]

Biological evolution is an ever ongoing incessant process. It occurred in the past, is occurring at the present, and will continue to do so in the future as long as life exists on the planet. Wisdom89 (T / C) 22:22, 16 March 2008 (UTC)[reply]

The only end state for evolution of humanity will be extinction or immortality. If you agree with the Doomsday argument the former may be quite soon. GameKeeper (talk) 23:28, 16 March 2008 (UTC)[reply]

Extinction is the only eventual possibility, the question is when and why. Personally I'm of the "we're never going to get off of this hunk of rock" school of thought, and would be highly surprised if we made it through the next century, judging by how the last one went. --Fastfission (talk) 23:38, 16 March 2008 (UTC)[reply]

One recent study found that human evolution had recently accelerated (though "recently here" is from 50,000 BC to 10,000 BC or so). 140.247.254.4 (talk) 01:03, 17 March 2008 (UTC)[reply]

We could also be in a state of de-evolution now that computers are handling so many of our former jobs.
Assuming a distant stable future and an extreme mastery of technology and genetic manipulations by the descendents of humans, we can imagine a world where the idea of human evolution as become obsolete. An almost total ability to genetically engineer and transform organisms breaks the barriers between what we call robots and living organism. The notion of 'humanity' has lost its meaning as it has become only a narrow range of genetic possibilities in a large spectrum of possible configurations that are all part of society, or maybe the term's meaning has broaden to include all sentient beings. So the idea of human evolution will disappear in time to be replaced by genetic engineering (and maybe we'll all look like manga and disney characters?). Keria (talk) 13:30, 17 March 2008 (UTC)[reply]
It bears mention that we haven't really been around as a species for all that long. The numbers are hard to imagine as they are -- 2.5 Million Years for humans and 3.8 billion years for the cooled planet -- but consider the first land masses on earth as having formed around midnight of January 1, and we are approaching midnight of Dec 31 a year later. That puts humans on the planet for the past 4 ½ hrs -- during the first two hours of which we seemed to coexist quite peacefully with the Neandertal, and possibly others. All of recorded history has taken place in the past 2 minutes. (The dinosaurs died out on Christmas Eve). The jellyfish, in contrast, have been around for a couple of months, relatively unchanged. In the face of these kind of numbers, it's easy to see where the question gets murky. If we are to quantifiably evolve, it's going to take tens of thousands of years, just like every other organism on the planet, which means that we've got to survive long enough to do it. And if we're talking about a speciation event, then by definition "we" will no longer be...Vance.mcpherson (talk) 21:42, 17 March 2008 (UTC)[reply]

Winter Solstice

In the year 1990, did the winter solstice fall on December 22nd? Or the 21st? 71.174.25.42 (talk)Winter

The winter solstice falls on the 21st during leap years and the 22nd during regular years. 1990 was not a leap year. Therefore, December 22 was the winter solstice. --Bowlhover 21:40, 16 March 2008 (UTC)
Unfortunately, it's not quite that simple: "Depending on the shift of the calendar, the event of the Winter solstice occurs some time between December 20 and December 23 each year in the Northern hemisphere." See Winter Solstice#Date, especially the graph on the left. You also need to keep in mind that the solstice proper is a single moment, which happens simultaneously in all time zones. Depending on where on the planet you are, it may be late in the evening on one day, or early in the morning on the next. BTW, [8] (found through Google) gives the 1990 winter solstice happening on Dec 22 at 03:10 UTC. Note that this is actually 9:10 pm on Dec 21 in Chicago. (Also note that the Winter Solstice happens in June for those people south of the equator, due to the inversion of the seasons:) -- 128.104.112.85 (talk) 22:49, 16 March 2008 (UTC)[reply]
For some reason, I thought the winter solstice always fell on December 22 UTC time or December 21 UTC time. Of course, that's not true; the winter solstice article states that it occurs on December 20 this year and December 21 for a few years afterwards. --Bowlhover 04:22, 17 March 2008 (UTC)

Radiography in Thailand?

Does anyone have any information on working as a radiographer in Thailand? I am studying it at the moment in Scotland and would quite like to do an elective placement there. 172.142.123.129 (talk) 23:02, 16 March 2008 (UTC)[reply]

I had a look, and curiously we don't have a Healthcare in Thailand article. Perhaps the best place to start would be the Thai Embassy in London which has a list of hospitals in Bangkok. Sorry, I couldn't find out if there was a consulate in Edinburgh. Astronaut (talk) 04:21, 18 March 2008 (UTC)[reply]

what is the most effective toothpaste?

which brand of toothpaste is the most effective? —Preceding unsigned comment added by 79.122.42.52 (talk) 23:40, 16 March 2008 (UTC)[reply]

The brand you actually use. —Preceding unsigned comment added by 71.100.174.10 (talk) 23:44, 16 March 2008 (UTC)[reply]
That would depend on what you want it to do. There is cavity prevention, tartar control, tooth whitening, fresh breath, etc. Depending on what particular problem(s) you have, a different formulation may help. StuRat (talk) 23:53, 16 March 2008 (UTC)[reply]
The one that claimed to be able to split water molecules during brushing in order to release 'activated oxygen' into the mouth for a deeper clean sounded pretty interesting. Shame they stopped advertising it... --Kurt Shaped Box (talk) 23:58, 16 March 2008 (UTC)[reply]
All you need to do that is brush with hydrogen peroxide. —Preceding unsigned comment added by 71.100.174.10 (talk) 00:02, 17 March 2008 (UTC)[reply]

I'm not sure what brands are available in Hungary, but you might want to check out an independent product reviewer like ConsumerReports.org, or even a website like Epinions.--The Fat Man Who Never Came Back (talk) 23:54, 16 March 2008 (UTC)[reply]

All of the same major ones are available here as the US. Which is the best? (Did you read independent reviews?)

As you haven't said exactly what you mean by "most effective", you might do well to read the Consumer Reports' article on various aspects of toothpastes and the claims made about them here [9]. ៛ Bielle (talk) 01:35, 17 March 2008 (UTC)[reply]
I had a small travel pack of Colgate sensitive (clinically proven sensitivity relief) promising all kinds of protection and relief from pain, which I used to clean the grunge off the baked enamel stove top -- surprisingly effective. No need for harsh scrubbiing. I'm still waiting to hear from the cooker about its new found confidence, but I'm keeping it handy for next time when nothing else works. Julia Rossi (talk) 05:23, 17 March 2008 (UTC)[reply]

please remember that the most effective toothpaste always has most number of active chemicals which means you may.. may..and not definitely.. damage your teeth. So, I always prefer the least effective from a trusted company. -Balaji —Preceding unsigned comment added by 59.92.126.151 (talk) 13:58, 18 March 2008 (UTC)[reply]

Speed in space(not the drug)

I was discussing with a friend about the age of the universe. A subject way over my head. I brought up that a quasar had been found that was 28 billion light years away from us and gaining. Yet the universe is only 13.73 billion years old. So that would mean that the quasar is traveling faster than the speed of light. It has moved farther than light could in that time. Disance in time = speed.(I know that we are also moving, so some math would have to be done) That is how I and the traffic cops understand speed. But he said that it realy was not moving that fast. The space in between us was actualy growing. My response was Duh, you don't say. That is what distance is. Anyway, he was serrious. He said I would have to understand general reletivity to understand that objects in space are not moving at speed but that space was growing. Can you tell me if he is an idiot or not? And if he is not can you explain this in common language? thanks.

cris —Preceding unsigned comment added by 68.209.69.103 (talk) 23:59, 16 March 2008 (UTC)[reply]

Metric expansion of space, yes, space is in fact getting bigger. A common analogy used in popular science it to imagine a universe that is confined to the surface of a balloon. The balloon obviously has a volume to it, but everything, including light, is permanently stuck to travelling along the surface. If you draw two dots on the balloon to represent, say, you and a quasar, and then you blow up the balloon, you two will get farther and farther apart. Each of you can look around and accurately say, "Everything around me is moving away from me, and I seem to be standing still." So who's right, and who's moving? Or is it merely space that's changing? I'm not a general relativity expert, so that's as close as I'm going to get to explaining this to you. Someguy1221 (talk) 00:08, 17 March 2008 (UTC)[reply]

OK. I understand the baloon analogy. But the two objects are still gaining distance from one another. Wouldn't that still asign speed to them? Are all objects in space spreading apart? Are they doing this equaly or are some moving faster away than others? —Preceding unsigned comment added by Loach (talkcontribs) 01:18, 17 March 2008 (UTC)[reply]

I removed the spaces from before your answer, because they put your text in an unformatted grey box ...
like this.
If you want to indent text, use colons (these things - :::::) instead. Now, on to your question. Yes you can assign a speed to the separation between two objects, but there are two caveats: (1) you can only say that they're moving relative to each other, and (2) you do so based on the frame of reference you are observing from. The "relativity" in Einstein's theory of relativity refers to the fact that as long as you keep track of things properly, no frame of reference is better than another - and most importantly, there's no "true" frame of reference, so you can't just say "that object is moving at this velocity" unless you state with respect to what. As to the rest of your questions, yes most of the observable universe appears to be expanding at a fairly uniform rate, with differences due mostly to gravitational effects (if you and I are standing on the balloon, and holding on to each other, then we won't be pulled apart as much as the balloon expands). In fact, the uniform expansion of space is what allows us to equate how far away things are with how fast they're moving away from us, proportional to Hubble's constant. And that linear relationship means that, yes, the furthest objects appear to recede faster than the speed of light. Confusing Manifestation(Say hi!) 03:03, 17 March 2008 (UTC)[reply]
There are no inertial reference frames at cosmological scales; the symmetry is broken by the overall curvature. Length contraction, time dilation and the relativity of simultaneity are meaningless in cosmology. There is a privileged time coordinate (cosmological time, the distance to the south pole on the globe below) and a privileged spatial distance (comoving distance, the difference in longitude on the globe), which can be used to define a privileged recession velocity between two objects. Hubble's law says that the (privileged) recession velocity is roughly proportional to the (privileged) distance. There is no privileged position or absolute velocity, though (i.e. no center to the expansion). -- BenRG (talk) 21:36, 17 March 2008 (UTC)[reply]

OK. Sorry for the grey box. I do not know how I did that. One last question. Does the universe have a central starting point from which the BB started? I've heard both yes and no. Do we know where it is? And could a reference on speed be made from there. thank you for your help. —Preceding unsigned comment added by 68.209.69.103 (talk) 03:29, 17 March 2008 (UTC)[reply]

You're probably hitting TAB or something before typing your questions. Anyway, there was no central starting point. If you were to run the universe backwards, and watch it with a camera in any inertial reference frame you please, you would see the density of the universe climb higher and higher, approaching infinity at the big bang. Once again, this would happen no matter where your camera was or what reference frame your camera was in. So every position in space has equal claim to having been the "central starting place." And depending on what the shape of the universe is, the universe may well have always had infinite volume and no boundaries; so it may actually be geometrically impossible to define a center of the universe. Someguy1221 (talk) 03:39, 17 March 2008 (UTC)[reply]
See my response above—no inertial reference frames. Also, it doesn't make sense to say that something is in a particular inertial frame, since every inertial frame assigns coordinates to every object. Also also, the universe doesn't look the same at different speeds (it makes a difference whether you're moving with respect to the CMBR). But it's true that the camera will see the density go to infinity regardless of its state of motion. -- BenRG (talk) 21:36, 17 March 2008 (UTC)[reply]
It's not just that objects are moving (which they are); the very fabric of space is expanding as well. Imagine two ants on a blanket, walking away from each other. But the blanket itself is also stretching wider at the same time, so the net distance that they are from each other increases very quickly. What's impressive is that this particular effect—the expansion of space—is pretty much the only way that something can end up going faster than the speed of light relative to anything else. The objects themselves are not moving faster than the speed of light, but the distance between them is growing at a rate that they end up distances apart that would require going faster than light to actually have done had the space not been expanding. Crazy, no? --98.217.8.46 (talk) 14:18, 17 March 2008 (UTC)[reply]
How can we see something farther than n light years away when the universe is only n years old? That's easy: it's simply not true that light travels a distance ct in a time t. It's never been true, thousands of confident assertions in books notwithstanding.
It's easiest to visualize what's going on in the case of a closed universe which expands, reaches a maximum size, and then recollapses. For simplicity suppose there's just one spatial dimension, which is all we need for this example. You can imagine this universe as the surface of a globe, with the south pole being the big bang, the equator being the time of maximum size, and the north pole being the big crunch. (Note that this is different from the "expanding balloon" analogy—this is a fixed-size balloon which shows the whole history of the universe at once.) We can draw lines of latitude (x axes, purple) and lines of longitude (t axes, cyan) on this globe. We'll draw a very large number of them so that the whole globe is covered fairly densely with intersecting latitude and longitude lines. What "light travels at a constant speed" really means is that light crosses all of these intersecting axes at the same angle. Usually the units are chosen so that the angle is 45°. A curve with this property is known as a loxodrome, and an example is shown in white in the image to the right. Unfortunately the angle here is nowhere near 45°, but try to imagine a similar curve with a 45° angle. Say that the present era is 30° from the south pole. The spatial size of the universe in this era is 2πR sin 30° = πR, and the elapsed time since the big bang is πR/6, where R is the radius of the globe. The first question is: how far away in this era can two objects be that started out in the same place at the big bang, and that have moved slower than light since then? The answer is obviously "arbitrarily far". In particular, they can be on opposite sides of the universe (πR/2 light years apart) even though it has only been πR/6 years since the big bang. The second question is: how far away is the most distant object we can see? The answer is again "arbitrarily far", because the loxodrome that represents the path of a light beam has circled the entire universe in the time since the big bang. In fact it has circled it infinitely many times, so we will see infinitely many images of everything in the universe at progressively earlier stages in their evolution. Note that the distances involved here have nothing to do with the speed c—they're a consequence of the large-scale geometry. The distance ct, where t is the time since the big bang, is geometrically meaningless. In the real world, the current distance to the south pole (along a line of longitude) is 13.7 billion years, and the current distance to the most distant known quasar (along a line of latitude) is 28 billion light years. Nothing in the geometry of the real world relates the latter to c times the former.
The globe model differs from the real FLRW metric in a few ways. First, it's missing two spatial dimensions, but that doesn't matter here. Second, the scale factor is wrong: a(t) ~ sin t isn't even a solution of the Friedmann equations, much less a solution approximating the world we see. Third, there's no reason to think that the real universe wraps around spatially like that. You can imagine "unrolling" the globe such that circling around it actually takes you to a new place (technically this is taking the universal cover of the sphere with the poles removed). Fourth, the early universe was opaque, so we should actually cut off the loxodrome some distance short of the south pole (i.e. when it hits Antarctica). But the basic geometry is right. This is why we can see things farther than ct away. -- BenRG (talk) 21:18, 17 March 2008 (UTC)[reply]
Hi. I'm not really an expert at this but is it possible that the light left the quasar, say, 12 billion years ago, and travelled to us now, so the image is really only 12 billion light-years away, but people calculated it based on the redshift of the quasar, and perhaps estimated that the quasar is currently 28 billion light-years away from us, as the expansion of space can travel faster than light in all those dimentions but matter itelf relative to itself can't? Thanks. ~AH1(TCU) 23:13, 17 March 2008 (UTC)[reply]
I'm not sure I understand your question. There is no image 12 billion light years away. As I said above (and below), the travel time times c is a completely meaningless distance. It has no physical significance at all. Light really does not travel 12 billion light years in 12 billion years. -- BenRG (talk) 23:57, 18 March 2008 (UTC)[reply]
Okay, I got inspired and made some numerically accurate images. These are two views of a surface that's just like the globe above except that the shape is calculated from the ΛCDM model and the WMAP five-year data. Click on either image for a larger version, and on the first one for some mathematical details of the embedding. The small circle at the "bottom" is 700 million years after the big bang. (Nothing earlier can be embedded with this particular choice of embedding parameters.) The large circle at the "top" is 18 billion years after the big bang, where you can start to see the accelerating expansion that eventually dominates in this model. The lines of latitude (purple) are 1 billion years apart. The lines of longitude (cyan) are one billion present-era light years apart. They're closer together than that in the past and farther apart in the future; in all eras the physical distance is proportional to the Euclidean distance measured along the lines of latitude. The cyan lines are also the worldlines of objects moving with the Hubble flow. The brown line is the worldline of the Milky Way and the yellow line is the worldline of the most distant known quasar with redshift z ~ 6.4 (I've assumed both are moving with the Hubble flow). The red line is the path of light from the quasar to present-day Earth, and the orange line shows the quasar's distance from Earth at the current cosmological time. You can verify by counting grid lines that the present-era distance to the quasar is about 28 billion light years and the light travel time is about 12 billion years. You should also be able to see that the light beam always makes a 45° angle with the nearby grid lines. (In the second image it kind of looks like it doesn't, but that's because of the perspective foreshortening.)
Every light beam currently reaching the earth follows the same path as the red beam, except possibly with an earlier or later starting time or from a different direction. So the region of spacetime we can see right now—that is, Earth's past light cone—is the subsurface traced out by the red line and its counterparts in other directions (most of which can't be drawn on this surface since two dimensions of space are missing).
Another interesting thing to notice is that the (real, metric) distance between the light beam and the Milky Way increases for the first few billion years before reaching a maximum and decreasing again. This is related to the odd fact that the angular diameter of extremely distant objects increases with distance—that is, the rules of perspective are reversed and more distant objects look larger. The cosmological time at which objects look smallest is the same as the time at which the light beam's distance is largest. The reason for this is left as an exercise for the reader (there's a simple geometrical answer).
I deliberately cut off the embedding short of a full circle to emphasize that space doesn't loop back on itself. The cutoff distance doesn't have any physical significance, and neither does the circumference of the full circle. It's an adjustable parameter of the embedding and I just picked a value that looked good.
All four of the colored lines are loxodromes (if that word is appropriate for a surface that's this much unlike a globe). The brown, red, and yellow lines are also spacetime geodesics, but the orange line isn't. N.B.: most geodesics are not loxodromes and most loxodromes are not geodesics. In fact they only coincide when v=0 or v=c, which happen to be the two cases that appear here. Also note that the spacetime geodesics (based on the real, Lorentzian geometry) are not the same as the Euclidean geodesics on this embedded surface, unfortunately. But I still think this kind of embedding is a useful way of understanding the expansion of the universe. It might be nice to have these images or similar ones in metric expansion of space. The expanding raisin bread model leaves something to be desired. -- BenRG (talk) 23:57, 18 March 2008 (UTC)[reply]
Holey-moley! Excellent detail, I'm bookmarking this for the future. I sure hope you're right, I'll be reading this for awhile :) Thanks for the effort! Franamax (talk) 00:17, 19 March 2008 (UTC)[reply]

March 17

Brushing teeth with bleach?

This question has been removed. Per the reference desk guidelines, the reference desk is not an appropriate place to request medical, legal or other professional advice, including any kind of medical diagnosis, prognosis, or treatment recommendations. For such advice, please see a qualified professional. If you don't believe this is such a request, please explain what you meant to ask, either here or on the Reference Desk's talk page.
This question has been removed. Per the reference desk guidelines, the reference desk is not an appropriate place to request medical, legal or other professional advice, including any kind of medical diagnosis or prognosis, or treatment recommendations. For such advice, please see a qualified professional. If you don't believe this is such a request, please explain what you meant to ask, either here or on the Reference Desk's talk page. --~~~~
--S.dedalus (talk) 02:39, 17 March 2008 (UTC)[reply]
I am not seeking medical advice. HYENASTE 02:48, 17 March 2008 (UTC)[reply]
Good because you're not going to get it. This is a stupid question. Theresa Knott | The otter sank 11:07, 17 March 2008 (UTC)[reply]
What does it say on the label on your bleach bottle? Read the part where it states the health risks. Why expect someone here to know more than the manufacturers who had lawyers work up the warning label? -- kainaw 02:38, 17 March 2008 (UTC)[reply]
It may be worth pointing out that the questioner seems to have been misled by the notion of "bleaching" teeth. When teeth are "bleached", it is done with specific preparations designed to whiten teeth, none of which contain household bleach (sodium hypochlorite). You don't bleach teeth with bleach. - Nunh-huh 14:21, 17 March 2008 (UTC)[reply]

what is the plant in this picture?

The large bushy plants after the grass in the foreground.

I was walking in the park with my boyfriend today and we could not identify the round bushy plants. They are in the background of this pic...Are they called pampa grass or somthing?W-i-k-i-l-o-v-e-r-1-7 (talk) 00:58, 17 March 2008 (UTC)[reply]

What part of the world is this in? -GTBacchus(talk) 03:23, 17 March 2008 (UTC)[reply]

Pacifica, California. I this Pampas Grass?W-i-k-i-l-o-v-e-r-1-7 (talk) 03:40, 17 March 2008 (UTC)[reply]

Hi, there's an illustration here[10] of purple needlegrass or it could be a kind of para grass in California. It looks like the lower green grass has grown through and up around the dieback. Julia Rossi (talk) 05:44, 17 March 2008 (UTC) (Sorry, not since it's blue-green and belongs to your main plant.)[reply]
I think you are right, that this is pampas grass (not a native plant, but common in that area). You are seeing this years fresh green growth coming up through the broken and tangled dry remains of last years leaves and stems. --169.230.94.28 (talk) 16:23, 17 March 2008 (UTC)[reply]

Did the Big Bang go bang

My understanding of the BB is that before it there was nothing. A split second after the BB was hydrogen and maybe some hellium. And that the "Big Bang" was an explosion. Like they said on the History chanel, "the Universe" exploded into existence. So I mentioned this to a friend and said that the Big Bang was an explosion. And that before that there was nothing. He said I was wrong on both accounts. I all but called him a morron. He is not an ID. He is saying that the BB was not an explosion. My Q: If it was not an explosion then what was it? Q: what was it that exploded? What was there before the BB? thanks cris —Preceding unsigned comment added by Loach (talkcontribs) 01:11, 17 March 2008 (UTC)[reply]

The Big Bang was an explosion, in the sense that it was a rapid expansion of energy and matter. The tricky bit about it is that it wasn't just a bunch of compacted stuff exploding into the surrounding space, it was an explosion of the space itself. And, at the same time that space came into being, so did time, so it doesn't really make sense to ask "what came before the Big Bang", since there was no before - a similar question in spatial co-ordinates would be "what's North of the North Pole?" Take a look at Big Bang and Timeline of the Big Bang for more details. Confusing Manifestation(Say hi!) 02:55, 17 March 2008 (UTC)[reply]


Thank you. That is more or less what I thought. Again, thank you for the help. —Preceding unsigned comment added by 68.209.69.103 (talk) 03:20, 17 March 2008 (UTC)[reply]

I could have been your friend. The terminology is very confusing. "Big bang cosmology" is a description of the evolution of the universe over the last 13.7 billion years. It's not a description of how the universe started. The model has a mathematical singularity called "the big bang singularity" or just "the big bang", which you can think of as a limiting state of infinite density, but nobody takes it seriously; it's seen as a sign that the model breaks down there. Among people who speculate about what should replace the singularity, the most popular models involve a universe with no particular beginning of time (that is, there's nothing inherent in the models that suggests a beginning of time, in contrast to the big bang model, which has a singularity at a finite time in the past). See also Age of the universe#Explanation. As to whether the expansion of the universe should be called an explosion, that's a matter of terminology, but I tend to think that it doesn't fit the dictionary definition of an explosion very well. For one thing, one normally thinks of an explosion as expanding outward into the surrounding space, and there's no space surrounding the universe. It's homogeneous. Note that if it was an explosion then the explosion is still going on; there's no transition point at which it stopped exploding and started just expanding. -- BenRG (talk) 12:05, 17 March 2008 (UTC)[reply]
A fun little bit of history -- the "Big Bang" was a term used first by Prof. Sir Fred Hoyle, one of the Big Bang's most outspoken critics. He had used the term disparagingly, and as a contrast to his "Steady State" model, but (assumedly to his chagrin) the phrase caught on first among the general public, and eventually in the scientific community. Bill Bryson's excellent book "A Short History of Nearly Everything" tells the tale entertainingly. Also see the obituary at [11].Vance.mcpherson (talk) 19:53, 17 March 2008 (UTC)[reply]

eggs

Do all eggs contain all of the amino acids, i.e., if forced to live in a survival situation with nothing but eggs how long could you survive with no other food? —Preceding unsigned comment added by 71.100.174.10 (talk) 01:55, 17 March 2008 (UTC)[reply]

Assuming you are referring to chicken eggs, they contain very little vitamin C. Without vitamin C, scurvy is very likely to occur. -- kainaw 02:36, 17 March 2008 (UTC)[reply]
Which is true, and pertinent to the second part of the question. As to the first part, yes, eggs contain all of the essential amino acids, which is the reason they are considered a "high quality" source of protein. But as Kainaw points out, the essential amino acids alone aren't enough to support health; there are other essential nutrients that are not amino acids. (Another problem is that eggs contain avidin, so a diet consisting entirely of eggs is likely to cause biotin deficiency. - Nunh-huh 03:02, 17 March 2008 (UTC)[reply]
As a followup question what basic items contain all of the nutients essential for indefinite survival? —Preceding unsigned comment added by 71.100.174.10 (talk) 04:56, 17 March 2008 (UTC)[reply]
I asked a question before about human milk, which, after all, is designed to provide for all human nutritional needs, at least for a baby. However, the nutritional label on regular cow's milk seems to indicate it is lacking in many nutrients we need. I forget if this discrepancy was ever explained. StuRat (talk) 05:57, 17 March 2008 (UTC)[reply]
(Not sarcasm here.) And that's why it's cow's milk and not us-milk, and is made up with supplements for human infants. There's a nice section about comparative milks here[12]. Julia Rossi (talk) 06:06, 17 March 2008 (UTC)[reply]
But human milk doesn't contain all the nutrients we need, like iron and folic acid: [13]. So how do babies survive on it ? StuRat (talk) 17:29, 17 March 2008 (UTC)[reply]
Because they're babies and have different nutritional needs to us. That's why, apart from the convenience factor, they have to be weaned as they grow up and start developing different nutritional needs. And why you can't feed a tiny baby puréed adult food instead of milk. Skittle (talk) 13:19, 18 March 2008 (UTC)[reply]
Babies do have different nutritional requiremnets, yes, like needing more fat and cholesterol to support rapid growth. However, they certainly must require iron for growth, as well. Don't babies have hemoglobin in their blood ? StuRat (talk) 14:30, 18 March 2008 (UTC)[reply]

It sounds like you saying that human milk and supplemented cow's milk provide complete nutrition for indefinite survival. What I am really looking for are combinations of other foods that one might search for and find in a survival situation like egg of all types for protein and pine needles for Vit C. I'm sure there must be lots of combinations so I am trying to find out what they are. Is there a chart somewhere perhaps that shows all of the required essentals with overlays of the portions provided by various foods? —Preceding unsigned comment added by 71.100.174.10 (talk) 07:43, 17 March 2008 (UTC)[reply]

Adventurers seem to know what food to take that doesn't weigh much and keeps them going for days (eg beef jerky, dried fruits, nuts and chocolate), but I don't know what that would be listed under. About milk, my take is that it isn't the complete food. I looked at space food but there's no information about nutrition just mechanics. Julia Rossi (talk) 08:53, 17 March 2008 (UTC)[reply]
Space food has some interesting information but I'm not looking for food which is pre-prepared but rather items which one might look for in the great outdoors, i.e., jungle, coastline, desert, etc. I'm not necessarily looking for foods you can take with you but rather foods that you can find. —Preceding unsigned comment added by 71.100.174.10 (talk) 15:35, 17 March 2008 (UTC)[reply]
You may be interested in some of the references of the Survival skills article. What food sources you might encounter in a wilderness would of course depend entirely on what biotope you are in – looking for mussels, for example, would be a great idea on the coast but less useful in an inland desert... So where exactly are you planning to get stranded? Also note that you can survive for weeks without any food at all, and for months or years on an incomplete diet. Are we talking about a permanent situation here, or a would crawling back towards inhabited areas be an option? --169.230.94.28 (talk) 16:13, 17 March 2008 (UTC)[reply]
In my youth when I ran away from home my inability to find a hamburger, French fries and a soda usually brought me to my senses and led me to the nearest pavement an on to the nearest fast food store. Now that I am older I'm curious just how long I can go out there somewhere nobody else is without need for finding pavement and inevitably French fried onion rings. —Preceding unsigned comment added by 71.100.171.236 (talk) 22:32, 17 March 2008 (UTC)[reply]

If you supplemented your eggs with fruits and vegetables I think you'd do pretty well. If you knew your edible plants better than how to steal eggs (you might decimate the local population rather fast depending on where you are eating only eggs), you could also do well combining grains with beans or nuts for your protein. Variety would be the key, some meat, some fruit, etc., since there's no one food you can healthily live on forever. — Laura Scudder 16:31, 17 March 2008 (UTC)[reply]

Sodium-potassium pumps

What percentage of the ATP that is used in the body is used by the brain to run the sodium potassium pumps? Schmultz (talk) 03:05, 17 March 2008 (UTC)schmultz[reply]

25%-30%, but don't reference me in your thesis. Mac Davis (talk) 04:46, 17 March 2008 (UTC)[reply]

Phosphodiester bonds

I want to know whether a molecule of water is removed when forming a phosphodiester bond between 2 DNA neucleotides. —Preceding unsigned comment added by 124.43.61.81 (talk) 06:42, 17 March 2008 (UTC)[reply]

Yes. Someguy1221 (talk) 09:24, 17 March 2008 (UTC)[reply]
See condensation reaction. Wisdom89 (T / C) 19:03, 17 March 2008 (UTC)[reply]

infrared raman spectroscopy

can infrared raman spectroscopy be used to detect bones in in vivo detection Neel shah556 (talk) 08:22, 17 March 2008 (UTC)[reply]

If the bones are covered by flesh, no. Infrared raman spec is used mostly on thin tissue samples or single cells. In these samples it can detect differences in the concentrations of proteins, lipids, carbohydrates and nucleic acids.--Shniken1 (talk) 11:11, 17 March 2008 (UTC)[reply]
Good answer ("that's using the old noodle"). :-) StuRat (talk) 14:48, 18 March 2008 (UTC)[reply]

Minimum entropy

Can whatever proceeded the Big Bang be defined as the state of minimum entropy?

There is no accepted physical theory on what preceded the big bang. In fact, accepted physics only deals with what happened after the very moment of the big bang. So there's nothing you can say about it. Someguy1221 (talk) 09:30, 17 March 2008 (UTC)[reply]
It is reasonable to say that some point in the early or very early universe there was a point of least entropy, and that entropy has increased since then, giving a direction to the cosmological arrow of time. The problem lies in trying to pin down just when this point of least entropy was. One big difficulty is that we have no agreed model for the physics of the very early universe (up to the end of the inflationary epoch) so we can't be sure that the second law of thermodynamics even applied then. One theory is that inflation itself drove entropy down, so that the point of least entropy is at the end of inflation, and the universe has been climbing out of this entropy "hole" ever since. Gandalf61 (talk) 12:15, 17 March 2008 (UTC)[reply]

swing??

why can't bernoulli's theorem be used to explain swing in cricket balls? --scoobydoo (talk) 14:19, 17 March 2008 (UTC)[reply]

because cricket is not a series of tubes? —Preceding unsigned comment added by 79.122.42.52 (talk) 14:59, 17 March 2008 (UTC)[reply]

I've read through the article and I suppose it might involve Bernoulli's Principle, but since I'm American half the words are complete Greek to me. The article doesn't really explain what swing does, can someone fix that? 206.252.74.48 (talk) 19:05, 17 March 2008 (UTC)[reply]
The Swing bowling article might help, but it is still full of cricket jargon. Perhaps the best explanation for american ears is to think of it like pitching a curveball, but using a different principle to get the ball to curve. Astronaut (talk) 03:36, 18 March 2008 (UTC)[reply]
Most ball sports players, can use Bernoulli's Theorem to cause the ball to curve in flight. I've seen it in football, golf, tennis, table tennis, baseball. However, I think Bernoulli's Theorem assumes the surface is of constant roughness. In cricket, bowlers will polish one side of the ball (by polishing it on their trousers), and occasionally cheat by roughening the other side (by stepping on the ball with their spiked shoes or scratching the surface with coins or dirt). Having one smooth side and one rough side affects the airflow around the ball and causes it to curve in flight - towards the rough side (I think :-)) where there is more aerodynamic drag. If the bowler has also put some spin on the ball, the overall effect can be very complicated. Astronaut (talk) 03:36, 18 March 2008 (UTC)[reply]

temp & fire

does temperature & fire have any relation? i mean, can i put out the fire of a burning body by cooling it?--scoobydoo (talk) 14:23, 17 March 2008 (UTC)[reply]

You might be able to - for instance if you manage to freeze it.
Also some things just won't burn below a certain temperature.87.102.13.144 (talk) 14:56, 17 March 2008 (UTC)[reply]

See our article on fire triangle. Fires do need heat. Rapidly cooling a fire (by adding lots of water for example) does put it out. Theresa Knott | The otter sank 15:25, 17 March 2008 (UTC)[reply]

NASA has a page here that shows how to snuff a candle by drawing the heat away from the wick with a coil of bare copper wire. --Milkbreath (talk) 16:47, 17 March 2008 (UTC)[reply]
There are basically three ways to extinguish a fire: deprive it of heat, fuel, or oxygen. Spraying water on a fire removes heat because the water absorbs heat when it is vaporized to form steam. Since water is inexpensive and readily available, this makes a good choice for most fires. It's not a good choice for some fires, like electrical fires (because water conducts electricity and could electrocute the firefighters) and oil fires (because water can splatter burning oil). In such cases they normally try to deprive the fire of oxygen by using halon or foam, or just wait until it burns itself out by using up the available fuel supply. StuRat (talk) 14:42, 18 March 2008 (UTC)[reply]

Speed of ...

Say you have a rigid object, such as a meter long titanium pole. When you pull one end, the other end presumably doesn't move instantly, because the force would have to move faster than the speed of light. It must be determined by the flexibility between the bonds of the titanium atoms/molecules. So ignoring the impracticality of its weight and size, if you were to pull a light-year, or some similar length, long titanium pole, the other end wouldn't notice it's been pulled until all the bonds are at their maximum pulling length? At that point, it still can't be instant if it's pulled further? Could anyone elaborate on what goes on? -- MacAddct  1984 (talk • contribs) 15:16, 17 March 2008 (UTC)[reply]

I'm not sure what you are asking but yes, it is impossible to pull all of it instantaniously. The rod will deform elastically and the pull will travel down it at the speed of sound in that material. (much slower than the speed of light). Theresa Knott | The otter sank 15:28, 17 March 2008 (UTC)[reply]
This is a Reference Desk Frequently-Asked Question but I think you've already sussed-out the answer: What we consider to be the "structural strength" of solid materials is actually the electromagnetic interaction of the electron shells of the constituent atoms. And these electromagnetic interactions can never propagate faster than the speed of light. So if you pull or push on that light-year-long titanium rod, a compression or expansion wave propagates through the material. It certainly doesn't go faster than the speed of light and probably only travels at the speed of sound in that material.
Atlant (talk) 15:31, 17 March 2008 (UTC)[reply]
Thanks! Yeah, that's about what I was thinking. It's hard to think in terms of familiar objects moving in ways we're not familiar with.
As for it being FAQ , is there an FAQ (official or unofficial) for the Reference Desk? If not, there really should be one started... -- MacAddct  1984 (talk • contribs) 15:53, 17 March 2008 (UTC)[reply]
There is a FAQ page, at Wikipedia:Reference_desk/FAQ. It is embryonic and underused, because it is hidden. We should link to it from the main RD page, and maybe mention it in the Before asking a question/Search first section at the top of each RD page. --169.230.94.28 (talk) 19:04, 17 March 2008 (UTC)[reply]
So what happens if you move one end faster than the speed of sound ? StuRat (talk) 17:36, 17 March 2008 (UTC)[reply]
You'd be breaking the rod. By definition you would be moving the atoms faster than they could convey that movement on to the atoms next to it; the rod wouldn't be structurally stable if you could do that, by definition. Remember it's the speed of sound in that material, not the speed of sound in air, which is normally what we think the speed of sound as being. --Captain Ref Desk (talk) 18:08, 17 March 2008 (UTC)[reply]
Or, if you are moving in a compressive direction, and buckling doesn't occur, you may create a shock wave. --169.230.94.28 (talk) 19:04, 17 March 2008 (UTC)[reply]
It's not speed that would break or buckle the rod, but acceleration 196.2.113.148 (talk) 22:07, 17 March 2008 (UTC)[reply]

I've been told that in the disease of Diabetes damage to the body is done by excessive amounts of either sugar or insulin molecules. Exactly what type of damage is done by excess amounts of either molecule besides oppressing the production/utilization of the other molecule that would cause the loss of toes? Is it damage to cell chemistry or structure, and if so of what does the damage consist? —Preceding unsigned comment added by 71.100.174.10 (talk) 15:47, 17 March 2008 (UTC)[reply]

Wikipedia has a very detailed article on diabetes, signs and symptoms, the different types, etc... -- MacAddct  1984 (talk • contribs) 15:56, 17 March 2008 (UTC)[reply]
If you're losing toes, you should go see the doctor. -- MacAddct  1984 (talk • contribs) 15:58, 17 March 2008 (UTC)[reply]
LOL... I'm hoping to learn what the sugar and the insulin do to the body that causes reports to be written of poeple with Diabeties to loose toes. Mine remain quit tasty, thanks, according to the wife. —Preceding unsigned comment added by 71.100.171.236 (talk) 21:40, 17 March 2008 (UTC)[reply]
Nothing in the post suggests that the poster is losing toes.
More specifically this is probably the section of the article you need. AlmostReadytoFly (talk) 16:52, 17 March 2008 (UTC)[reply]
this is much more what I'm looking for. Thanks.

It brings tears to my eyes...

...to waste part of an onion, but are the green sprouts in the center edible ? That is, should I make the best of them or make compost of them ? StuRat (talk) 18:07, 17 March 2008 (UTC)[reply]

They are edible, but you are only getting them as your onions are starting to grow. Try keeping your onions in a dark cool place, if in a hot country the fridge will do. This will stop the central green bit appearing. Taste wise the green bit will be slightly more tough than the cooked down flesh, more similar to a spring onion. I would not use it in a Burre Blanc sauce but it would be fine in a stir fry or stew. GameKeeper (talk) 19:23, 17 March 2008 (UTC)[reply]
Thx. StuRat (talk) 20:06, 17 March 2008 (UTC)[reply]
1138? 206.252.74.48 (talk) 16:01, 18 March 2008 (UTC)[reply]

1-Octen-3-ol CAS numbers?

I'm trying to track down the CAS numbers for 1-octen-3-ol. The best I could find is this page from a book about coffee. It lists the numbers [3391-86-4], (±) [50999-79-6], (R)-(-) [3687-48-7], (S)-(+) [24587-53-9]. The latter two clearly refer to the two enantiomers. My question is: what's the difference between 3391-86-4 and 50999-79-6? Both appear to refer to a mixture of the two enantiomers. Thanks, AxelBoldt (talk) 18:43, 17 March 2008 (UTC)[reply]

[3391-86-4] clearly states "matsutake alcohol FEMA 2805" almost certainly "Flavor and Extract Manufacturers Association".
The most probable explanation is that the natural product 'matsutake alcohol' was known before it was recognised to be indentical to 1-octen-3-ol, hence the different numbers.
The simplest way if possible would be to examine the papers referenced by [3391-86-4] and see exactly what they describe.87.102.13.144 (talk) 19:32, 17 March 2008 (UTC)[reply]
Also [3391-86-4] might refer to a naturally obtained product - if in the future this product is shown to contain other components then the distinction will be useful (for example conside oil of almond which is almost all benzaldhehyde - but not 100%)
Take a look at the papers cited for both from chemical abstracts - this might give you a clue as to what's going on eg does the use of one of the numbers cease after a certain date, or is the term 'matsutake' specific to certain countries ?87.102.13.144 (talk) 19:40, 17 March 2008 (UTC)[reply]
According to CAS, [50999-79-6] is a deleted CAS number and [3391-86-4] should be used. -- Ed (Edgar181) 19:44, 17 March 2008 (UTC)[reply]

Magical self cleaning wound

Yesterday I cut myself in my finger whilst outside. It had some dirt in it so I attempted to clean it to the best of my ability. However some small about of debris was still stuck inside. After a while I stopped trying to get the debris out, thinking it would most likely do more harm than good to go out of my way to pick out the last bit of dirt. Come next morning my wound was magically clean. And when I say clean I mean, perfect pink/reddish skin with not a hint of dirt. I'm wondering, where did this dirt go? Is there some mechanism by which wounds are able to expel small amounts of debris? Did it just fall out whilst I was sleeping even though I couldn't get it out myself? PvT (talk) 18:58, 17 March 2008 (UTC)[reply]

Blood would be the obvious agent to pour out of a wound and cleanse it. I've also noticed a clear fluid flow out of wounds not deep enough to bleed. I suspect that this is the blood plasma without the red blood cells, which need a larger opening to escape. Pus can also flow out, but only after the wound becomes infected. StuRat (talk) 19:13, 17 March 2008 (UTC)[reply]

Well the skin literally grows from inside to out - so it's unlikely to trap any dirt.. Once formed the new skin layer will be drier than a wound making it less sticky - and so any remaining dirt just fell off. Probably.87.102.13.144 (talk) 19:15, 17 March 2008 (UTC)[reply]

There is also the possibility that while you were asleep your unconcious mind went to work scraping the wound against the bed sheets or instructing other parts of your body to scratch or rub the wound. Sleep study video reveals all sorts of directed movements as if we were actually or part awake.

I wouldn't be so sure wounds are self-cleaning. When I was a child, I managed to stab myself in the hand with a newly sharpened pencil. Decades later, I still have a small grey dot under the skin. Astronaut (talk) 02:47, 18 March 2008 (UTC)[reply]
And I have a 20 year old splinter that hasn't managed to work it's way out, yet. StuRat (talk) 04:41, 18 March 2008 (UTC)[reply]
I too have a mark from a pencil stab (a "friend" did this to me in primary school). I think these are different from cuts as they are deep puncture wounds. Also, the graphite is organically neutral meaning that it won't be attacked by the immune system. I once had a thorn stuck in a deep cut, and it became an infected boil, and the thorn was eventually ejected along with puss. -- Q Chris (talk) 10:10, 18 March 2008 (UTC)[reply]
In my case I believe the wooden splinter eventually dissolved, leaving the dark brown wood stain which once coated the splinter behind as a type of tattoo. StuRat (talk) 14:19, 18 March 2008 (UTC)[reply]

Searching for massively-multiplayer synonyms in PubMed

So "tight junction protein 1" has a variety of names; how am I supposed to search effectively in PubMed? Is there some way of referring to this gene/protein in such a way that all synonyms are searched for at once?

I might be wrong, but a little further down on that page it has all of the names in a list box ("Search for TJP1"). Hold down shift and you can select all of them at once. Change the boolean term to OR so that you get publications with any of those names in it (AND would get publications that contain each and every one of them). Hit search. Many results abound. --Captain Ref Desk (talk) 20:25, 17 March 2008 (UTC)[reply]
Have you ever worked with the MeSH function?
Change the drop down to "MeSH" and search for, say, tjp 1. The result shows zonula occludens-2 protein, and it the page lists the synonyms. On the same line of the substance name, all the way to the right, click on "Link", then on "Pubmed". Hopefully this will give you good results. 128.163.116.64 (talk) 22:18, 17 March 2008 (UTC)[reply]

Chem Acid Stoichimetry problem

Suppose I was given Ba(OH)2 at 0.5 M/L and was asked to find it's pOH. Since it's diprotic, I have to multiply the concentration by 2 right? If so, why do I multiply the concentration by 2? Why not the number of moles?


It will dissociate into Ba + 2OH right? 99.240.177.206 (talk) 21:04, 17 March 2008 (UTC)[reply]

Yes, Ba(OH)2 will dissociate into Ba2+ and two OH- (note charges). It's not really diprotic, as it has two hydroxyls, not two protons (diprotic refers to acids, not bases). The reason you multiply by two is that each mole of Ba(OH)2 contains two moles of OH-, so in a sense, 2 is the number of moles. BTW, this analysis only holds if the barium hydroxide is completely dissolved and completely dissociated. If it's a weak base it may stick around as dissolved Ba(OH)2 or possibly Ba(OH)+ (Sorry, I don't know the pKas of Ba(OH)2, so I can't give you a definitive answer.) Also, I don't think the solubilities work out - the barium hydroxide article lists the solubility as ~60 g/L (for the octahydrate), whereas a 0.5 M solution would be ~150 g/L (for the octahydrate). Lastly be aware that the capitol M already means moles per liter (molar). You don't have to divide through by liters again, unless you mean moles per liter per liter. -- 128.104.112.85 (talk) 21:46, 17 March 2008 (UTC)[reply]

Part of the USS Pampanito's hull

The USS Pampanito is the submarine pictured in this photo. Is there a term for the black part of its hull that tapers from the bow? Or is it still the hull? --BrokenSphereMsg me 21:08, 17 March 2008 (UTC)[reply]
If you mean the part that bulges around the waterline, I'd say tapers to. That said, it's certainly part of the hull -- or at least the light hull. On similar-era U-boats, those structures were "saddle tanks" -- auxiliary ballast tanks. Such tanks could also be designed to carry fuel, but the Germans left their diesel tanks inside the pressure hull to improve depth charge survivability. — Lomn 21:23, 17 March 2008 (UTC)[reply]
Whaddaya know? Saddle tank (submarine). — Lomn 21:27, 17 March 2008 (UTC)[reply]

Paraffin/aluminium foil

Years ago at a ski-lodge as after-dinner entertainment one of the guests carried out a trick involving a bit of chemistry, and I wonder whether anyone here knows how it was done. A piece of foil - the type found in some cigarette packs - had something done to it using paraffin/kerosene. The foil was then crumpled into a tight ball and placed in the palm of the hand. Within a few seconds it became too hot to hold. How was it done? —Preceding unsigned comment added by 196.2.113.148 (talk) 21:56, 17 March 2008 (UTC)[reply]

Probably caustic soda......Rotational (talk) 16:18, 18 March 2008 (UTC)[reply]

March 18

More quantum physics...

I need to know, in depth, all aspects of the behavior of electrons in crystaline structures (more specifically, quantum dots). If there is an article on this, please let me know. Thanks! Zrs 12 (talk) 03:01, 18 March 2008 (UTC)[reply]

Quantum dot seems like a good place to start. DMacks (talk) 04:07, 18 March 2008 (UTC)[reply]
Read it. I need to know in depth. I'm having to do a project over these things and apparently electrons act differently in them than in single atoms. (That probably sounds like I'm being short or rude, but I don't mean it like that). Zrs 12 (talk) 04:41, 18 March 2008 (UTC)[reply]
No problemo...wasn't clear from your question what if anything you already knew/had read:) By spacially constraining the electrons, the energy levels are changed, so the spectra of a quantum dot that are based on electronic transitions are different than for free atoms/molecules. Quantum well has some detail. What level and sort of project are we talking about here? "In depth" and "all aspects" sound either contradictory or unlimited in scope without knowing what kind of project and what academic level. The refs cited in quantum dot, quantum well, and potential well are good ones. DMacks (talk) 05:12, 18 March 2008 (UTC)[reply]
Thanks for your help. Well, I am in 9th grade. However, I have been reading alot about the subject and am beginning to grasp some of the more in depth concepts such as the mathematical concepts in some areas of quantum physics. This is also a project to be in the state science fair, so it probably must be done at a higher that 9th grade level to have any chance. By the way, the specific information I seek about the electrons in quantum dots is; Why are the electrons confined to bands rather than descrete energy levels where they can have partial quanta of energy (right?). And, how do the electrons behave. They bounce back and forth, but between what? Where? Are they confined to a specific area? How and why are the energy levels changed by spatial restriction? To how big an area does the Pauli exculsion principle apply? To a single atom? A molecule? What do the matrices of objects such as represent?Bra-ket notation Is it correct to say that, for the photoelectric effect to take place in which Eg is the energy of the bandgap, h is Planck's constant, and f is the frequency of the light?
Once again, thanks very much for your help. Zrs 12 (talk) 05:45, 18 March 2008 (UTC)[reply]
As the Pauli Exclusion Principle article states, it says that no two fermions may occupy the same quantum state simultaneously, fermions are a type of elementary particle, described by The Standard Model. I recommend purchasing the book Hacking Matter at the local bookstore, about quantum dot, wells, and artificial atoms. I suggest taking up your discussion at Physicsforums's physics section— they will be very helpful to you.[14] Mac Davis (talk) 10:49, 19 March 2008 (UTC)[reply]

Infrared raman spectroscopy

Infrared rays having best penetration in skin and flesh, why can't be utilized for in-vivo non-invasive probing? Can't these rays yield good signature of bones? Neel shah556 (talk) 04:00, 18 March 2008 (UTC)[reply]

Infrared is strongly absorbed by many of the components in skin and flesh. Water, lipids, proteins etc. Infrared therefore does not have good penetration of skin and flesh. X-rays on the other hand are not absorbed (much) by flesh and therefore penetrate the skin and flesh to the bones, thus x-rays are used to look at bones in-vivo, not infrared.--Shniken1 (talk) 12:40, 18 March 2008 (UTC)[reply]

global warming

Will rise in sea level be even across the world? Or will the rise be more in regions near to arctic and antarctic and less in regions near equator? —Preceding unsigned comment added by 59.92.113.83 (talk) 06:46, 18 March 2008 (UTC)[reply]

Sea level changes are not uniform, but it's not as simple as greater rises at higher latitudes; contributing factors are tides, density due to salinity, expansion and contraction due to heat, flows around landmasses, etcetera. Most models show that the greatest rise will be seen in the arctic (due to increased runoffs causing a drop in salinity) and the lowest rise will be seen in the antarctic. FiggyBee (talk) 07:34, 18 March 2008 (UTC)[reply]
Dumb question then - why wouldn't the higher sea level water flow down to the lower level until the levels are equal? Franamax (talk) 08:12, 18 March 2008 (UTC)[reply]
It does, but not fast enough to overcome all the other factors (which are all ongoing processes, rather than one-off starting conditions). FiggyBee (talk) 08:19, 18 March 2008 (UTC)[reply]
Thanks! I knew it was a dumb question. :) Flow varies with square-root of water head, right? It's just hard to imagine that the Arctic Ocean could fill up with water faster than it could empty out. I forgot the factor of time. Franamax (talk) 08:39, 18 March 2008 (UTC)[reply]
I agree with you Franamax. While some difference seems inevitable I have a hard time imagining a significant difference. Are we talking about fractions of an inch here ? StuRat (talk) 14:24, 18 March 2008 (UTC)[reply]
From Sea level : "Mean sea level does not remain constant over the surface of the entire earth. For instance, mean sea level at the Pacific end of the Panama Canal stands 20 cm higher than at the Atlantic end." 72.10.110.107 (talk) 15:22, 18 March 2008 (UTC)[reply]
The earth's spin alone accounts for the height of the Pacific being slightly elevated relative to the Atlantic, as I understand it, and that difference is more pronounced towards the Pacific's West end. Any of us who have spent time on bodies as small as the Great Lakes know that water level can seem to rise or fall significantly due to changes in wind direction. And, for that matter, the Great Lakes do have a flow -- a molecule of water at the West end of Lake Superior takes 15,000 years to reach the Atlantic. So water, as it were, doesn't settle and "even out" quite so quickly as we might suppose.Vance.mcpherson (talk) 15:41, 18 March 2008 (UTC)[reply]
But, of course, the Great Lakes are far more restricted in their flow to the ocean than the oceans are to each other. If the oceans were only connected via narrow channels, it would be easier to understand a difference in elevations. The tides will also cause some areas to have far higher elevations than others at any given time. However, none of this explains why the relative elevations would significantly change when more water is added to the system via global warming. StuRat (talk) 22:15, 18 March 2008 (UTC)[reply]
The Arctic Ocean may be the exception though, it only has two outlets, and some pretty big rivers flowing in, Mackenzie, Ob-Irtysh, Yenisey (I think). Plus thermal expansion as the ocean warms up, so it's plausible. I would be interested to see the models though. Rising sea levels are certainly already a problem in the Arctic. Most interesting... Franamax (talk) 01:28, 19 March 2008 (UTC)[reply]
A rise in sea level is not necessarily even around the world. As discussed above, mean sea level (MSL) involves complex measurement and it is attempted to accurately determine an MSL using the geoid as a level reference surface. A rise in sea level, however, would expand the geoid and mean sea level. Isostatic changes will also do this. Mac Davis (talk) 10:42, 19 March 2008 (UTC)[reply]
Hi. Well, I read in a book that a recent satellite measurement of the world's oceans that the mean sea level excluding waves and other anomalies in a given area was actually greatly affected by the sea bottom elevation: the surface water was generally lower near a trench and higher near an undersea mountain range. Also, a shift in sea currents could affect the amount of heat in a given area and thus the sea level. Erosion will also cause the now-undersea landforms to erode, allowing sea levels farther inland. Given the right (or should I say wrong) type of rock, the sea can either flow into the groundwater and flood a nearby low basin or create waves that enter the basin. For now, many regions in the arctic are still experiencing the shoreline recede, as the rising of the land since the last ice age is currently faster than the sea level rise, at least until the Greenland and Antarctic ice sheets start collapsing. Also, if you try to use the flood maps at flood.firetree.net, they are largely innacurate (see their disclaimer). Constant flooding in an area may also allow sea levels to creep up. If there is a river, the sea can easily flood farther inland bacause of the eroded lowlands formed by the river. Hope this helps. Thanks. ~AH1(TCU) 21:11, 19 March 2008 (UTC)[reply]

MSM expiry

This question has been removed. Per the reference desk guidelines, the reference desk is not an appropriate place to request medical, legal or other professional advice, including any kind of medical diagnosis, prognosis, or treatment recommendations. For such advice, please see a qualified professional. If you don't believe this is such a request, please explain what you meant to ask, either here or on the Reference Desk's talk page.
This question has been removed. Per the reference desk guidelines, the reference desk is not an appropriate place to request medical, legal or other professional advice, including any kind of medical diagnosis or prognosis, or treatment recommendations. For such advice, please see a qualified professional. If you don't believe this is such a request, please explain what you meant to ask, either here or on the Reference Desk's talk page. --~~~~
TenOfAllTrades(talk) 12:55, 18 March 2008 (UTC)[reply]
You should ask a pharmacist and your doctor about this. They know much more than we do. Franamax (talk) 08:00, 18 March 2008 (UTC)[reply]

Doctors and pharmacists are Wikipedia editors as well.

This is not about pharmacy or medicine. The question is: Is MSM oxidized or reduced in household storage (or does it decompose)? [household chemistry]

MSM is not pharmacy; It is a mineral (sulfur) nutrient, much like calcium or magnesium. But the question is not so much about nutrition as it is about msm decomposition in household storage. Please leave this for discussion. —Preceding unsigned comment added by Zaqry (talkcontribs) 19:42, 18 March 2008 (UTC)[reply]

Is MSM oxidized or reduced in household storage (or does it decompose)? —Preceding unsigned comment added by Zaqry (talkcontribs) 22:39, 18 March 2008 (UTC)[reply]

Does MSM decompose in household storage and how (oxidation or reduction reaction for example)? —Preceding unsigned comment added by Zaqry (talkcontribs) 18:00, 19 March 2008 (UTC)[reply]

Radio transmitter power

A 50,000-watt radio station is transmitting at full power.

a) Is there at least 50KVA (50KVA plus the heat coming out of the building) running through the power lines into the station at all times?
b) Does it matter whether anyone is talking over the radio?
c) Is the answer different whether it's AM or FM, or VHF or UHF television?

Thanks for the help! Franamax (talk) 08:08, 18 March 2008 (UTC)[reply]

My understanding is

a) Yes for FM. See c
b) No for FM, but for AM it will increase and decrease as someone talks. There is a variation of AM known as Single-sideband modulation (SSB) that only uses power when there is some sound.
c) For FM and AM yes. For TV, I am not sure but I think that US digital TV uses ATSC Standards a type of SSB. I would imagine that in video there are no "silent" pauses like in audio, so the power load will be more constant. European digital TV uses Orthogonal frequency-division multiplexing, which I am unsure about. Analogue TV uses FM, and will therefore have constant power. -- Q Chris (talk) 10:02, 18 March 2008 (UTC)[reply]
For frequency modulation, phase modulation, and ordinary analog amplitude modulation (of 100% or less modulation depth), the average power of the carrier wave remains constant so the power inflow to the building will always be at least 50 KW. For single-sideband modulation, there's no transmitted power when no one is talking so the average power of the carrier wave varies with the average volume of the modulating signal. Analog TV uses vestigial sideband modulation (a hybrid of AM and SSB) for the video and frequency modulation for the audio.
Atlant (talk) 12:04, 19 March 2008 (UTC)[reply]

Atomic Radio clocks in Bulgaria

I live in Bulgaria. How do I find out whether an atomic radio clock will work in Bulgaria? Do I have to obtain one from a specific provider? —Preceding unsigned comment added by ENiklaus (talkcontribs) 12:06, 18 March 2008 (UTC)[reply]

Bulgaria seems to be at the edge of the region where the German DCF77 signal can be received. According to the PTB, the range is 2000 km (unfortunately that's probably not very helpful, but maybe interesting). Icek (talk) 12:54, 18 March 2008 (UTC)[reply]
(edit conflict)
There are long-wave time transmissions from Switzerland and Germany which might reach Bulgaria (depending on mountains etc.) They broadcast on slightly different frequencies, so, yes, you do need to purchase the correct receiver, and there is no guarantee that it will work where you live. I have found that the UK transmission can be picked up in some parts of buildings, but not in other parts. Mountains and steel frames cause reception problems. See Radio clock for more details, but the price of radio clocks is now quite low, so if they are not available for sale in your area, then it is probably because there is not a good radio signal. A world-wide time transmission can be picked up by a GPS clock (built into most GPS units) but you need to be either outdoors or near to a large window to pick up the satellite signals. dbfirs 13:08, 18 March 2008 (UTC)[reply]

Genetic probabilities

In a problem I'm trying to solve, there are 2 alleles, A and a. a is a defective allele, so the genotype aa induces disease. Both parents are carriers: Aa.
1) What is the probability that all three children are of normal phenotype? The way I see it, the chance of the normal phenotype in one child is 3/4, the result of a simple Aa x Aa Punnett square, so (3/4)3 = 27/64.
2) What is the probability that one or more of the children have the disease? Shouldn't it be 1/4 + 1/4 + 1/4 = 3/4? But 27/64 + 3/4 ≠ 1 ... help! Thanks, anon. —Preceding unsigned comment added by 70.23.85.120 (talk) 16:48, 18 March 2008 (UTC)[reply]

You can only add probabilities when they are probabilities of mutually exclusive (i.e. incompatible) events. Child A has the disease with probability 1/4, same for B, but the probability that A or B or both have the disease is not 1/4 + 1/4 because you are double-counting the case where both A and B have it, which has probability 1/16. So the probability for two children would be 7/16, which equals 1 - 9/16 that you could compute from the probability that both were normal. With three children it is easier to start with the probability that all three are normal, and you have that already. --Anonymous, 16:58 UTC, March 18, 2008.
As an exercise to help understand the probabilities you might want to list every possible combo. Each child has 4 possible combos (AA, Aa, aA, or aa). While Aa and aA both mean they are carriers of this recessive disorder, it's probably easier to list them separately as equal probability than combine them together. So, that gives you 4^3 or 64 possible combos. List them out and record how many children have the disease or are carriers for each combo. StuRat (talk) 22:05, 18 March 2008 (UTC)[reply]
Thanks so much :-), anon. —Preceding unsigned comment added by 141.155.38.85 (talk) 17:17, 19 March 2008 (UTC)[reply]

300,000 Elephants in Chad?

In the article excepted below [and in many secondary & tertiary sources] the assertion is made that “--the Chad population was over 300,000 animals as recently as 1970 and has been reduced to approximately 10,000 as of 2006.” I have looked carefully at the publications of the African Elephant Specialist Group of the IUCN for confirmation of these figures [latest report 2007]. The IUCN folks report no data on Chad elephants before 1985. They report “no evidence” for 1981 and no figures before that time. Where did this 300,000 figure originate and who started it?

2006 Zakouma elephant slaughter refers to a series of poaching massacres of African elephants in the vicinity of Zakouma National Park in southeastern Chad. These killings have been documented in aerial surveys conducted from May through August 2006 and total at least 100 animals.[1] This region has a four decade history of illegal killing of this species; in fact, the Chad population was over 300,000 animals as recently as 1970 and has been reduced to approximately 10,000 as of 2006. The African elephant nominally has Chadian governmental protection, but the implementation practices of the government (backed with certain EU help) have been insufficient to stem the slaughter by poachers.[1] The species African Bush Elephant (Loxodonta africana) occurs in several countries of Eastern Africa. —Preceding unsigned comment added by Lynmil (talkcontribs) 19:45, 18 March 2008 (UTC)[reply]

Paramagnetic metal that do not become magnetized.

The question is what are metals called that are paramagnetic,but do not become magnetized (they are attracted by magnets, but when removed from magnetic infuences, are not magnetic)?

What metals are these? How do I reference the subject on WikiPedia?

Thank You, My Email address (removed for your protection)—Preceding unsigned comment added by 151.202.62.236 (talk) 19:55, 18 March 2008 (UTC)[reply]

"soft magnets", see magnet also remanence and follow the links in the 'see also' section.
The remanence of a material is the extent to which the material retains the magnetic field - you are obviously asking for 'low remanence'. Please ask for furhter info.87.102.47.176 (talk) 20:15, 18 March 2008 (UTC)[reply]
You may want to have a look at [[15]] for a little fun. I saw a nifty powerpoint by a fellow from University of Victoria on paramagnetism some six years ago, and I'll be damned if I know how to find him. If memory serves, and the link I provide confirms, paramagnetism isn't so much about what metal is paramagnetic as what you did to the metal, i.e. what else is it bonded to. Gets into the realm of organic chemistry.Vance.mcpherson (talk) 20:15, 19 March 2008 (UTC)[reply]

What is cyclooctatetraene?

What is cyclooctatetraene? 66.81.43.129 (talk) 21:59, 18 March 2008 (UTC)[reply]

Take a look at our article, Cyclooctatetraene. It appears to have a lot of info. -- Flyguy649 talk 22:02, 18 March 2008 (UTC)[reply]

March 19

Insanity...

Is the capability to display behavior patterns indicative of insanity in any way proof that the insane entity had a mind of its own to lose in the first place?

Just musing to myself about those neglected, stir-crazy, obsessive-compulsive, self-mutilating parrots at this late hour... --Kurt Shaped Box (talk) 02:42, 19 March 2008 (UTC)[reply]

While it certainly is possible for any creature with a brain to suffer a brain malfunction, I'd expect that complex minds are more likely to have complex malfunctions. So, strange obsessive-compulsive behavior may indeed be a sign of a fairly complex mind. A simpler mind would likely suffer from a more basic malfunction, like walking in circles instead of straight. I suggest you get the parrot in question a companion (or at least a cuttlebone to cuddle with). :-) StuRat (talk) 05:45, 19 March 2008 (UTC)[reply]

Biology: What are these caterpillars doing?

What are the caterpillars doing in this video? - Pureblade | Θ 04:46, 19 March 2008 (UTC)[reply]

I would speculate that they are exhibiting a defensive behaviour by co-ordinated action to give the impression of a larger creature. They may also be releasing body hairs containing a very powerful irritant. It is entirely possible that they sense the presence of the video operator and are reacting appropriately. I have experienced a similar effect with pine processional moths in Spain, when I blew on them gently they all jerked simultaneously. This seems to me to be an analogous behaviour. What is slightly odd, if these are pine processional moths, is that they are out feeding in the daylight. Of course they could be a related species demonstrating similar behaviour. Richard Avery (talk) 08:32, 19 March 2008 (UTC)[reply]

Science in my gloves

We have to wear latex gloves in the cleanroom that I work in. Under these gloves we can wear a nylon glove liner. It's just a loose fitting liner so the glove isn't directly on your skin. I've noticed that sometimes as the night goes on, stains will develop inside my glove on the liner. The stains are blue like a bit of ink has found its way into the glove but the liners are white and stitched with white thread. There isn't any obvious source for the blue color. Some of the liners can be yellowed, almost as if they have been sitting in the sun, so I tried one of those to see if maybe the blue was due to a reaction between my sweat and whatever was yellowing the liners. I've only tested this once so far but the yellowed liner produced the blue stains while a whiter liner on my other hand did not. So what could be producing the blue color? Dismas|(talk) 08:16, 19 March 2008 (UTC)[reply]

an off-the-wall guess.... povidone/iodine (from gloves or handwashing) + starch --> blue stain. - Nunh-huh 08:53, 19 March 2008 (UTC)[reply]
I guess I should have mentioned this but we are required to wash our hands using this white foamy soap before putting the gloves on. Dismas|(talk) 09:01, 19 March 2008 (UTC)[reply]

Two questions

What proportion of the scientific community believes the earth to be less than 100,000 years old and approximatly how accurate is carbon dating?--193.120.116.177 (talk) 11:02, 19 March 2008 (UTC)[reply]

And how old does the pope beleive the earth to be?--193.120.116.177 (talk) 11:18, 19 March 2008 (UTC)[reply]
Age of the Earth might help.
Zain Ebrahim (talk) 11:25, 19 March 2008 (UTC)[reply]
The Roman Catholic church does not have any theological issues with the earth being 4-6 billion years old. See Evolution and the Roman Catholic Church for related information. Specifically note what Cardinal Ratzinger (now Pope Benedict) said in a report: "According to the widely accepted scientific account, the universe erupted 15 billion years ago in an explosion called the 'Big Bang' and has been expanding and cooling ever since. Later there gradually emerged the conditions necessary for the formation of atoms, still later the condensation of galaxies and stars, and about 10 billion years later the formation of planets. In our own solar system and on earth (formed about 4.5 billion years ago), the conditions have been favorable to the emergence of life." -- 14:38, 19 March 2008 (UTC) —Preceding unsigned comment added by 128.104.112.85 (talk)
Do you have a reference for this?Zain Ebrahim (talk) 14:43, 19 March 2008 (UTC)[reply]
Found one: [16]. Zain Ebrahim (talk) 14:47, 19 March 2008 (UTC)[reply]
Carbon_dating#Calibration regarding the accuracy of carbon dating and radiometric_dating#Limitation_of_techniques for accuracy in dating the age of the earth. Lanfear's Bane | t 11:37, 19 March 2008 (UTC)[reply]
Carbon dating is just one kind of radiometric dating. It isn't used for determining the age of the Earth—it is only useful for dating things in smaller time periods than that. Uranium-lead dating is more common for things as old as the Earth.
As for how much of the scientific community believing the earth is less than 100,000 years old, practically none—a tiny number, insignificant when compared to the number of all scientists, and of those small number I imagine even smaller of them are active participants in a field of scientific inquiry that actually informs such assessments, or in what are considered to be real scientific institutions (not the Institute of Creation Research, for example).
There are quite a number of people outside the scientific community who believe that, but there is simply no compelling scientific evidence that the Earth is so "young" and much compelling evidence against the idea. I don't think it's an overstatement to say that the only people who believe the Earth is less than 100,000 years old are people who have a predisposition towards wanting to believe that on account of their religious beliefs.
As for the Pope, I am not aware of the current Pope's point of view vis a vis the age of the Earth. However perhaps you are referencing the work of the long-since-deceased Archbishop Ussher, who calculated that the Earth was only around 2,000 years old. --Captain Ref Desk (talk) 19:32, 19 March 2008 (UTC)[reply]
To be slightly more accurate than 'insignificant', Institute for Creation Research claims around 500 scientists (by which I think they mean those with a PhD in a scientific subject) that subscribe to their views. I'm not sure how many scientists there are in the US, but my guess is that this is substantially less than 1%. Interestingly it probably means that only a tiny fraction of scientists who are also Christians believe that the earth is less than 100,000 years old. DJ Clayworth (talk) 17:46, 19 March 2008 (UTC)[reply]
And it's the sort of number that gets smaller when you take their list and weed out all of those who 1. have degrees from non-accredited universities, and 2. are actively publishing or doing research in subjects related to things like the age of the Earth. I'm betting you'd cut the number down by quite a lot if you just took out the number of engineers—no offense made to engineers, but they seem to always predominate in the "people who think they really understand science deeply but actually have a fairly superficial working knowledge, and as such often overestimate their own competency in subjects they have not really studied seriously" category. --Captain Ref Desk (talk) 19:32, 19 March 2008 (UTC)[reply]
The precise stand of the Vatican on the Big Bang, Creation vs. Evolution, etc., can be found here: [[17]], at the Holy See's website; scroll down to paragraph 62. Other Christian denominations are not nearly so centralized, but in general the Anglican Communion has distanced itself from the notion of a 10,000 year old earth. In fact, in the West, the remaining "literalists" are largely Protestant denominations, many of whom have no fantastic claim to antiquity themselves. Vance.mcpherson (talk) 20:38, 19 March 2008 (UTC)[reply]

How do you poison a dress?

(I'm not sure whether I should post this at the Humanities-desk or here, but I figure since it has to do with medicine than history, Science would be most appropriate)

I saw the movie Elizabeth (the one with Cate Blanchett) a while ago, and something struck me: there is a subplot in it about an attempted assassination of Elizabeth using a dress that had been poisoned, so that when she would put it on, she would die (the attempt failed, obviously). That seems to me to be a not uncommon motif in literature, poisoning a dress. The most obvious other example is Medea killing Creon and Glauce by poisoning a dress.

Is this possible? I mean, can you make a dress that otherwise looks (and smells) normal, but when you touch it or wear it, it will poison you? Supposing you could, could you do it in the 16th century? I mean, if you just dunk it in a vat of arsenic-contaminated water (or something), wouldn't it be all discolored and also smell pretty foul? And is this just part of mythology, or has there ever been an attempt of using this method of assassination? --Oskar 14:45, 19 March 2008 (UTC)[reply]

It would be possible, but it seems to be firmly in the territory of soap opera and mythology. - Nunh-huh 15:43, 19 March 2008 (UTC)[reply]
So say you were a 16th century cardinal who didn't like all this Church of England nonsense, how would you do it? --Oskar 17:38, 19 March 2008 (UTC)[reply]
Get out of town? - Nunh-huh 17:49, 19 March 2008 (UTC)[reply]
Maybe it was soaked in cyanide with a nice almond scent. -- MacAddct  1984 (talk • contribs) 16:14, 19 March 2008 (UTC)[reply]
Well, the CIA considered giving Fidel Castro a poisoned wet suit, so why not? --Sean 17:11, 19 March 2008 (UTC)[reply]
Coupla drops of dimethyl mercury would do it (but long-drawn-out, not quick-convulse&die), but not sure anyone in the 1500s would be able to handle that stuff. Heck, even today it is hard to handle safely. DMacks (talk) 19:14, 19 March 2008 (UTC)[reply]
Pyrrole might be a good candidate, as it is a permeator. See [[18]]. Have no idea how they'd get the stuff in Elizabethan times, though, the synthesis of it is complex ([[19]]).Vance.mcpherson (talk) 20:50, 19 March 2008 (UTC)[reply]
Lots of things permeate skin, but need something that is reliably toxic in a dose small enough to be delivered that way. The MSDS you cite lists an acute LD50 of 98 mg/kg (mouse, orally), so even if that's the right ballpark for (human, dermal), we're talking several dozen mL to absorb. In smaller doses, it's not that toxic (see note on pyrrole about its use in cigarettes). DMacks (talk) 21:00, 19 March 2008 (UTC)[reply]

hey

wats is the common and scientific name for sponges,flat worms, earthworms, bugs, frogs, jellyfish, round worms, clams, and starfish —Preceding unsigned comment added by 165.29.165.253 (talk) 15:14, 19 March 2008 (UTC)[reply]

Well, you just listed their common names. Scientific names will depend of each species of the animal you want; out of those groups there are probably millions(?) of different species. The only group they all fall under is the kingdom "Animalia". -- MacAddct  1984 (talk • contribs) 15:27, 19 March 2008 (UTC)[reply]
Simple enough eg type 'sponge' into the search box gives the sponge page which tells that they are all of the phylum 'Porifera'. You can do the same for the rest. type 'insects' for 'bugs'.83.100.183.180 (talk) 15:50, 19 March 2008 (UTC)[reply]

Tantalum hafnium carbide

We don't have any reference in the article Tantalum hafnium carbide, the substance with the highest melting point known (?). Where is the publication on the 4488 K melting point? Thanks in advance. Icek (talk) 15:55, 19 March 2008 (UTC)[reply]

This Britannica article is a very reliable source. --Bowlhover (talk) 16:00, 19 March 2008 (UTC)[reply]
Thank you, but does anyone know where to find the original publication? Icek (talk) 16:12, 19 March 2008 (UTC)[reply]
Google told me that the phrase "The alloy tantalum hafnium carbide. (Ta. 4. HfC. 5. ), with a melting point of 42158C,. is one of the most refractory substances" appears in "11 Hafnium" of doi:10.1002/9783527619634.ch32, but I don't have access to the actual text from here. A publication like that is probably well-footnoted to primary and/or reputable secondary sources. DMacks (talk) 19:12, 19 March 2008 (UTC)[reply]

New car smell

I just found out today that the new car smell is toxic and carcinogenic. So what should we do to avoid it? Would opening the windows help? --Lazar Taxon (talk) 17:26, 19 March 2008 (UTC)[reply]

Do you have a reference for this? It would surprise me (at least, in the US, with all our emphasis on car safety) that manufacturers would sell a toxic product. But, assuming you do want to avoid it.. the easiest way would seem to be, don't get a new car. Letting it air out sure seems like it wouldn't hurt either. Friday (talk) 17:36, 19 March 2008 (UTC)[reply]
I think it's more about the regulatory bodies not allowing a product to be sold that's clearly and undeniably toxic. If a company can plausibly deny a product's harmfulness (see Crisco, Dupont), that's good enough for the feds. Vranak (talk) 20:04, 19 March 2008 (UTC)[reply]
Found a couple sources: From a few years ago, saying it might be toxic, and more recently saying it's not toxic. This is what I get for not checking the wiki first- we have a decent article on this at New car smell. Friday (talk) 17:40, 19 March 2008 (UTC)[reply]

New Solar Chip

I found an interesting article and forgot to bookmark it. Now I cannot find it again.

I was looking at a new type of photovoltaic device that converts sunlight directly to electricity using a photochemical nano-gate composed of niobium [columbium], gold and silver, interchangeably, on one side of the gate. There was a separate and different word to describe this type of PV cell in its own page, and I cannot remember the word.

I have tried every combination of "solar power energy photovoltaic [columbium niobium] gold silver" that I can think of in your search engine, and in Google's search engine with wiki as the first word, and I can not find the article again.

Please help me find that page again.

The above was posted to the Help Desk recently and responses have not been helpful. Does anyone have any info on the above described chip, now undergoing testing, that was described on a Wikipedia page? External links welcomed.

was it a gratzel cell? 131.111.100.44 (talk) 18:05, 19 March 2008 (UTC)[reply]

Gravity

I was just reading an article on howstuffworks.com and i though came to me, if all the gravity on earth magically disappeared, would everything float up, surely it would just stay put, coz there wouldn't be a force acting on it?Vagery (talk) 19:20, 19 March 2008 (UTC)[reply]

Not exactly. Objects would slowly move away from the Earth and slightly east as the Earth rotated away from them. Drawing a picture of the rotating Earth and the object on it may help. It isn't that the object accelerates up, it's that the Earth underneath accelerates away from the object. anonymous6494 19:27, 19 March 2008 (UTC)[reply]
Thats what i thought would happen... just checking, thnx alot manVagery (talk) 19:29, 19 March 2008 (UTC)[reply]
Um, a person at the equator is traveling with the Earth's rotation, at a rate of 40,000km (earth's circumference) per 24 hours (length of one rotation), or, if my quick math is right, about 360m/s. Initially, said person will appear to stay put, but will in reality start moving 360m/s in a straight line, as will the ground under his feet. It wouldn't take long for things to start spreading out at that speed. Naturally, the velocity will be lower for people not on the equator, but you can do the math for that yourself, multiplying that velocity by a trig function that gives you 1 for the equator and 0 for the poles. -SandyJax (talk) 19:58, 19 March 2008 (UTC)[reply]
Would the entire Earth fly apart? The plates of the crust aren't firmly stuck together, so there'd be nothing holding them down, right? And then the layers underneath are mostly solid only because of pressure, so once the layers above are gone, they'd become fluid again and spin off? Or would they cool down fast enough to remain solid? --Allen (talk) 20:38, 19 March 2008 (UTC)[reply]
If gravity disappeared only in a sphere equal to the size of Earth, then everything within that sphere would move in the same direction relative to the distant stars that it was moving at the instant that gravity disappeared, so yes, it would all fly apart pretty quickly. There would be no more reason to rotate and everything would move in straight (geodesic) lines instead. Electromagnetic forces would still keep stuff stuck together, i.e. rocks would still be rocks and magma would quickly solidify into rocks, all those rocks would continue in a straight line. That's how I see it anyway. Franamax (talk) 21:14, 19 March 2008 (UTC)[reply]

Battleships in space

Years ago I read somewhere that steel retrieved from pre-1945 battleships was an essential component of satellites. The reason given was that all the steel smelted since 1945 (or shortly thereafter) was contaminated with radioactive elements from nuclear explosions to sufficient degree that it would affect the satellite instrumentation.

I'm not sure whether this was only for the Pioneer/Voyager probes and I'm not sure whether it was just one particular battleship, but I know I read it once and I've never been able to find a reference since. Can anyone help me out? Franamax (talk) 20:58, 19 March 2008 (UTC)[reply]

Operation Deadlight83.100.183.180 (talk) 21:15, 19 March 2008 (UTC)[reply]

Wiki Myth Busters...

So when you pull the ol' snap your friend in the butt with a towel prank, the snapping noise is caused by the end of the towel breaking the sound barrier. True or false? Beekone (talk) 21:09, 19 March 2008 (UTC)[reply]